Imo Problems Theorems Methods Algebra
Imo Problems Theorems Methods Algebra
and Methods
Algebra
Mathematical Olympiad Series
ISSN: 1793-8570
Published
The complete list of the published volumes in the series can be found at
http://www.worldscientific.com/series/mos
Vol. 25 Mathematical
Olympiad
Series
Authors
Jinhua Chen
Bin Xiong
East China Normal University, China
Proofreader
Jiu Ding
School of Mathematics and Natural Sciences,
University of Southern Mississippi, USA
Copy Editors
Lingzhi Kong, Liyu Zhang, and Ming Ni
East China Normal University Press, China
and
For photocopying of material in this volume, please pay a copying fee through the Copyright Clearance
Center, Inc., 222 Rosewood Drive, Danvers, MA 01923, USA. In this case permission to photocopy
is not required from the publisher.
Printed in Singapore
Preface
v
vi IMO Problems, Theorems, and Methods: Algebra
the host country. This committee narrows down the submissions to approx-
imately 30 Shortlist problems, covering algebra, geometry, number theory,
and combinatorics, with about seven to eight problems on each topic. These
are then presented to the Jury Meeting, composed of team leaders, who dis-
cuss and vote to decide on the six problems that will constitute the official
competition paper. The host country does not provide any problems.
This event has played a significant role in promoting the exchange of
mathematical education among nations, enhancing the level of mathemati-
cal education, facilitating mutual learning and understanding among young
students worldwide, stimulating a broad interest in mathematics among
secondary school students, and identifying and nurturing mathematically
gifted students.
The development over more than 60 years is the result of the collective
efforts of mathematicians, organizers, and contestants, and is worthy of
reflection and study. Particularly deserving of study are the evolution of
the competition problems, the mathematical ideas, and methods involved.
Indeed, several colleagues from the International Mathematical Olympiad
Research Center at East China Normal University had envisioned research
and publication before the 60th IMO. For this purpose, we initiated sev-
eral seminars involving over ten people. For special reasons, this work was
delayed. Based on the mathematical domains covered by the IMO prob-
lems — algebra, geometry, number theory, and combinatorics — we planned
to compile the work into four volumes, with the general title IMO Problems,
Theorems, and Methods, to be included in the “IMO Study Series.”
Each volume begins with an introduction that provides an overview of
the IMO. Subsequent chapters introduce relevant foundational knowledge
and methods, followed by a reclassification and organization of past IMO
problems. For some problems, multiple solutions are provided, along with
a difficulty analysis. It is worth noting that some problems do not fit neatly
into a single topic, as they may involve both algebra and number theory,
or algebra and combinatorics. We primarily categorize them based on the
topic under which they were placed on the Shortlist.
The four volumes titled IMO Problems, Theorems, and Methods were
conceived with an overall writing plan proposed by myself, with the authors
collectively discussing and refining the plan. The majority of the initial
drafts were completed by Jinhua Chen (Algebra), Tianqi Lin (Geome-
try), Gengyu Zhang (Number Theory), and Guangyu Xu (Combinatorics).
The first three volumes were supplemented, consolidated, and finalized by
Preface vii
Bin Xiong
June 2024
This page intentionally left blank
About the Authors
ix
This page intentionally left blank
Contents
Preface v
xi
xii IMO Problems, Theorems, and Methods: Algebra
1
2 IMO Problems, Theorems, and Methods: Algebra
Garrett Birkhoff, the son of George David Birkhoff, and his colleagues in
the Harvard University Department of Mathematics.
Many participants in the Putnam Competition have gone on to become
prominent mathematicians and scientists. John Milnor, David Mumford,
Daniel Quillen, Paul Cohen, John G. Thompson, and Manjul Bhargava
have been recipients of Fields Medal. Richard Feynman, Kenneth Ged-
des Wilson, Steven Weinberg, and Murray Gell-Mann have received Nobel
Prize in Physics, while John Nash was awarded the Nobel Prize in Eco-
nomic Sciences. Additionally, numerous Putnam Fellows have been elected
as members of the National Academy of Sciences in the United States.
Building upon the foundation of existing mathematics competitions in
many countries, particularly in Eastern European nations, Romania pro-
posed in 1956 the organization of an international mathematics competition
involving seven Eastern European countries. This proposal led to the inau-
gural IMO held in 1959.
The first IMO was held in Braşov, Romania, in 1959. As of 2023, the
IMO has been successfully held 64 times, with the exception of the year 1980
when it was not held for specific reasons. Apart from the 61st IMO, which
was postponed to September in 2020 due to the impact of the COVID-19
pandemic, the IMO typically takes place in July each year.
The IMO has emerged as the most influential secondary school math-
ematics competition at present. In recent years, the number of countries
and regions participating in this event has exceeded 100.
700
Number of the Participating Countries and Regions
600
Number of Contestants
500
400
300
200
100
0
1 4 7 10 13 16 19 22 25 28 31 34 37 40 43 46 49 52 55 58 61 64
increased by 17 countries and regions, reaching a total of 73, with 413 con-
testants. By the 40th IMO, which was still hosted by Romania in 1999,
the number of participating countries and regions had reached 81, with 450
contestants.
The first instance of the number of participating countries and regions
surpassing one hundred occurred during the 50th IMO, held in Germany in
2009, with a total of 565 contestants. Among the first 64 IMOs, the biggest
number of participating countries and regions, as well as the largest number
of contestants, was observed in the 60th IMO, hosted by the United King-
dom in 2019, where 621 contestants from 112 countries and regions took
part. In the 64th IMO held in Japan in 2023, there were 618 contestants
from 112 countries and regions.
As evident from Appendix A, the IMO is primarily hosted by European
countries. Moreover, as the number of participating countries and regions
in the IMO has increased, it is no longer confined to the seven founding
member countries, and many new participating countries and regions have
also begun to organize the IMO.
spans two days, with each day featuring three problems to be solved within
a four-and-a-half-hour timeframe. Each problem carries a maximum score
of 7 points, resulting in a total maximum score of 42 points, while the total
maximum score of the team is 252 points.
In early IMOs, the number of problems and their individual point values
varied from one session to another. For instance, the 2nd and 4th IMOs
featured seven problems, while all other IMOs had six problems each. Addi-
tionally, in the 13th IMO, although the total score remained at 42 points,
the six problems were allocated point values of 5, 7, 9, 6, 7, and 8, respec-
tively. It was only from the 22nd IMO, held in the United States in 1981,
that the IMO problems have become standardized, with each problem car-
rying 7 points and a total of six problems.
The number of contestants in each delegation has also become stable at
six individuals starting from the 24th IMO held in France in 1983.
Topic
1–10 20 6 29 7
11–20 20 12 18 10
21–30 14 16 18 12
31–40 13 16 15 16
41–50 15 11 20 14
51–60 13 19 18 11
61–64 6 7 6 5
Total 101 87 123 75
6 IMO Problems, Theorems, and Methods: Algebra
Remarkably, in the first 64 IMOs, there were five sessions when three
algebra problems were presented, specifically in the 3rd, 5th, 7th, 15th,
and 18th IMOs. In 29 IMOs, two algebra problems were featured, while
in 28 IMOs, only one algebra problem was included. In two IMOs, algebra
problem was absent, namely the 30th and 39th IMOs.
Problem Mean
Problem 1 0 0 5 13 23
Problem 2 2 6 18 9 6
Problem 3 22 12 4 3 0
Problem 4 0 0 7 17 17
Problem 5 4 10 14 10 3
Problem 6 24 11 6 0 0
Total 52 39 54 52 49
The 246 problems are categorized into four topics: algebra, combina-
torics, geometry, and number theory, as shown in Table 3. Notably, there
is a relatively large representation of combinatorics and geometry problems.
Combining this information with Table 1, it is evident that in the first 23
IMOs, algebra and geometry problems were predominant.
Furthermore, in early IMOs, geometry problems predominantly
appeared in the 1st/4th and 2nd/5th positions. However, starting from
Introduction to the IMO 7
Topic
Session 1, 4 2, 5 3, 6 1, 4 2, 5 3, 6 1, 4 2, 5 3, 6 1, 4 2, 5 3, 6
24–30 4 1 5 4 3 5 5 7 0 1 3 4
31–40 4 5 4 5 4 7 6 9 0 5 2 9
41–50 3 8 4 3 2 6 10 6 5 5 4 5
51–60 5 6 2 4 7 8 7 3 6 3 4 4
61–64 1 3 2 2 2 3 3 1 2 2 2 1
Total 17 23 17 18 18 29 31 26 13 16 15 23
57 65 70 54
the 41st to 50th IMOs, geometry problems were more commonly found in
the 1st/4th and 3rd/6th positions. Similarly, algebra problems were more
frequent in the 1st/4th and 2nd/5th positions, combinatorics problems were
more prevalent in the 2nd/5th and 3rd/6th positions, while the quantity of
number theory problems across different problem numbers does not differ
significantly.
From Table 4, it can be observed that among the four topics, the num-
bers of problems with an average score ranging from 2 to 4 points are quite
similar. However, in the combinatorics topic, there is a higher quantity of
challenging problems, with 31 problems having an average score between
0 and 2 points. Conversely, the geometry topic has the largest number
of relatively easy problems, with 23 problems scoring above 4 points.
This discrepancy is largely due to the fact that there are 29 combinatorics
problems in the 3rd/6th positions, and 31 geometry problems in the 1st/4th
positions.
Problem Mean
Algebra 7 14 19 6 11 57
Combinatorics 20 11 12 14 8 65
Geometry 13 5 14 15 23 70
Number theory 12 9 9 17 7 54
Total 52 39 54 52 49 246
8 IMO Problems, Theorems, and Methods: Algebra
an = max{ak + an−k |1 ≤ k ≤ n − 1}
for all n > s. Prove that there exist positive integers l and N with l ≤ s,
such that an = al + an−l for all n ≥ N .
n
n n
n
|xi − xj | ≤ |xi + xj |
i=1 j=1 i=1 j=1
Subject
1–10 11 1 1 5 2
11–20 3 5 3 7 2
21–30 1 6 2 4 1
31–40 0 5 3 4 1
41–50 0 3 1 8 3
51–60 1 7 3 2 0
61–64 0 1 1 3 1
Total 16 28 14 33 10
Introduction to the IMO 11
Problem Number
Number of Problems
Equation Problem 1, 4 2, 5 3, 6 in the First 64 IMOs
Finding solutions 0 0 0 7
Proving relationships 0 0 0 4
Investigating conditions 0 1 0 5
Total 0 1 0 16
Problem Number
Number of Problems
Function Problem 1, 4 2, 5 3, 6 in the First 64 IMOs
Proving properties 1 2 1 9
Determining values 0 0 1 4
Deriving expressions 6 8 1 15
Total 7 10 3 28
12 IMO Problems, Theorems, and Methods: Algebra
Problem Number
Number of Problems
Sequence Problem 1, 4 2, 5 3, 6 in the First 64 IMOs
Determining values 2 1 0 4
Existence problems 0 0 2 5
Proving quantitative relationships 0 0 4 5
Total 2 1 6 14
Problem Number
Number of Problems
Inequality Problem 1, 4 2, 5 3, 6 in the First 64 IMOs
Solving inequalities 1 0 0 5
Proving inequalities 6 8 4 25
Determining value ranges 0 1 1 3
Total 7 9 5 33
two problems; (3) proving properties of polynomials and sets, totaling six
problems.
As shown in Table 11, in the 24th–64th IMOs, there were a total of
six other algebra problems, primarily focusing on proving properties of
polynomials and sets.
Problem Number
Number of Problems
Others 1, 4 2, 5 3, 6 in the First 64 IMOs
Problem Mean
Equation Problems 0 1 0 0 0 1
Function problems 0 4 6 5 5 20
Sequence problems 1 3 3 0 2 9
Inequality problems 3 5 8 1 4 21
Others 3 1 2 0 0 6
Total 7 14 19 6 11 57
21 25 11
50
45
40
35
30
25
20
15
10
5
0
1 4 7 10 13 16 19 22 25 28 31 34 37 40 43 46 49 52 55 58 61 64
the 14th IMO (1972, Poland) with a perfect score of 40 points and eight
gold medalists; and the 28th IMO (1988, Cuba) with a perfect score of 42
points and 22 gold medalists.
3.1 Participation
In the first 64 IMOs, a total of 269 contestants took part in four or more
IMOs. Among them, two contestants attended seven IMOs, four contes-
tants attended six IMOs, 42 contestants attended five IMOs, and 221 con-
testants attended four IMOs, as shown in Table 13.
Coincidentally, in the 43rd IMO held in 2002, the gold medal cutoff
was set at 29 points, and David Kunszenti-Kovács achieved a total score of
exactly 29 points.
In the 44th IMO held in 2003, the gold medal cutoff was set at 29 points,
whereas Alexey Entin attained a total score of exactly 28 points.
In the 51st IMO held in 2010, the bronze medal cutoff was set at 15
points, and Zhuo Qun (Alex) Song achieved a total score of exactly 15
points.
In the 58th IMO held in 2017, the silver medal cutoff was set at 19
points, and Yeoh Zi Song achieved a total score of exactly 19 points. In the
59th IMO held in 2018, the silver medal cutoff was set at 25 points, and
Yeoh Zi Song’s total score was exactly 24 points. In the 61st IMO held in
2020, the gold medal cutoff was set at 31 points, and Yeoh Zi Song’s total
score was exactly 31 points.
Additionally, from 2002 to 2005 and in 2007, Sherry Gong participated
in the IMO, earning one bronze, two silver, and one gold medal. In the
48th IMO held in 2007, she ranked 7th individually. Notably, from 2002 to
2004, she was a member of the Puerto Rico IMO team, while in 2005 and
2007, she was a member of the United States IMO team.
From 2020 to 2023, Alex Chui participated in the IMO, securing two
gold and two silver medals. However, in 2020 and 2021, he was a member
of the Chinese Hong Kong IMO team, while in 2022 and 2023, he was a
member of the United Kingdom IMO team.
Other than Sherry Gong and Alex Chui, the remaining 267 contestants
hailed from 75 different countries and regions. Among them, there were 12
contestants from Cyprus and Moldova each, 11 from Malaysia, eight from
Trinidad and Tobago, seven from each of Estonia, Germany, Sri Lanka, and
North Macedonia, and six from Japan and Philippines each.
Table 14 Contestants with Four or More Gold Medals in the First 64 IMOs
In the 41st IMO held in 2000, the top four contestants all achieved
perfect scores, and Reid Barton ranked fifth with a total score of 39. In the
43rd IMO held in 2002, the top three contestants all achieved perfect scores,
and Christian Reiher ranked fourth with a total score of 36. Furthermore,
in the 50th IMO held in 2009, Lisa Sauermann achieved a total score of 41,
securing the third position.
Moreover, Oleg Golberg participated in the IMO from 2002 to 2004,
achieving three gold medals. He consistently ranked within the top 10 in
terms of total scores. Notably, in 2002 and 2003, he was a member of the
Russia IMO team, and in 2004, he was a member of the United States IMO
team.
Apart from Oleg Golberg, the remaining 48 contestants hailed from
22 different countries and regions. Among them, there were four contes-
tants from each of Russia, Bulgaria, Germany, Hungary, Romania, and
the United States. Both South Korea and the United Kingdom had three
contestants, while Canada, Japan, Singapore, and the Soviet Union were
represented by two contestants each.
As shown in Table 15, John Rickard, Imre Ruzsa, and Marc van Leeuwen
all achieved two special prizes in one IMO for their elegant solutions. Fur-
thermore, John Rickard, Imre Ruzsa, and László Lovász have all earned a
perfect score twice.
Perfect
Participation Special Gold Score
Contestant Country Year Prize Year Year Year
John Rickard The United 1975–1977 1976, 1977 (2) 1975–1977 1975, 1977
Kingdom
József Hungary 1963–1966 1965, 1966 1964–1966 1966
Pelikán
László Lovász Hungary 1963–1966 1965, 1966 1964–1966 1965, 1966
László Babai Hungary 1966–1968 1966, 1968 1968 1968
Simon The United 1967–1969 1967, 1969 1967–1969 1969
Phillips Kingdom
Norton
Wolfgang The German 1967–1971 1970, 1971 1968, 1970, 1971 1970
Burmeister Democratic
Republic
Imre Ruzsa Hungary 1969–1971 1971 (2) 1970, 1971 1970, 1971
Marc van The Nether- 1977–1978 1978 (2)
Leeuwen lands
Coincidentally, in the 11th IMO held in 1969, only three gold medals
were awarded, with Imre Ruzsa ranking 4th and receiving a silver medal.
Similarly, in the 19th IMO held in 1977, which also resulted in only 13 gold
medals, Marc van Leeuwen ranked 14th and earned a silver medal.
Additionally, these 44 contestants hailed from 16 different countries and
regions. Among them, there were seven contestants from each of Hungary
and the United Kingdom, five from the German Democratic Republic, four
from Bulgaria and Poland each, three from each of Czechoslovakia and the
Soviet Union, and two from Finland and the United States each.
Special prizes were more frequently granted in the first 20 IMOs, with
a total of 27 special prizes earned by 22 contestants from the 11th–20th
IMOs. Subsequently, the frequency of special prize presentations declined.
Since Moldovan contestant Iurie Boreico received a special prize for his
brilliant solution to IMO 46-3 (Algebra, proposed by South Korea) in 2005,
no contestant has achieved this accolade to date.
From 2003 to 2007, Iurie Boreico consistently participated in the IMO,
earning three gold and two silver medals. He achieved a perfect score in
2005 and 2006. It’s noteworthy that the 44th IMO held in 2003 only yielded
20 IMO Problems, Theorems, and Methods: Algebra
37 gold medals, with Iurie Boreico placing 38th individually and receiving
a silver medal.
x5 − x2 y5 − y2 z5 − z2
+ + ≥ 0.
x5 + y 2 + z 2 y 5 + z 2 + x2 z 5 + x2 + y 2
4 Summary
The IMO stands as a distinguished intellectual competition for young
minds. According to a study by Agarwal R. and Gaule P., statistical anal-
ysis reveals that among contestants in the IMO (including those who did
not secure medals), 22% choose to pursue further studies in mathemat-
ics, ultimately obtaining doctoral degrees in the field. Additionally, 1% of
these contestants become presenters at the International Congress of Math-
ematicians, and 0.2% attain the Fields Medal. These statistics underscore
the vital role of the IMO in identifying and nurturing mathematical talent.
It’s essential not to perceive the IMO as a mere selection exam. Rather
than focusing solely on the brief two-day competition, the crucial aspect
lies in the learning and preparation undertaken before participating. As
the mathematician Paul Halmos aptly put it, what mathematics really con-
sists of is problems and solutions. Contestants, through their exploration
of Olympiad problems, not only enhance their mathematical abilities but
also experience the joy and satisfaction of problem-solving. This experience
plants the seeds of a future career in mathematics.
However, it’s important to acknowledge that Olympiad problems and
research problems in mathematics differ. Research problems often lack
readily available answers and may require the investment of countless days
and nights. Hence, the IMO is just one pathway in the growth of math-
ematical talents, and success in the IMO is not the sole qualification for
becoming an outstanding mathematician.
Although every contestant aims for a gold medal, their aspirations go
far beyond accolades. On this stage, they have the opportunity to showcase
their intellectual capabilities, revel in the mathematical exploration, and
relish competing with talented young minds from around the world, all
without the narrow goal of proving their superiority over others. While the
Introduction to the IMO 21
competition results may vary, each contestant stands as a victor in their own
right and becomes a companion and witness to one another’s life journeys.
In contrast to the Olympics, where athletes’ careers are closely inter-
twined with the Games, the IMO is merely a chapter in the growth of these
gifted young individuals. Following the IMO, the door to a new mathemat-
ical world has already swung wide open for them.
This page intentionally left blank
Chapter 1
Equation Problems
23
24 IMO Problems, Theorems, and Methods: Algebra
who introduced the concept of groups and thereby initiated a new branch of
mathematics called “Abstract Algebra.” This not only elevated the study
of algebra to new heights but also had a significant impact on modern
mathematics, physics, chemistry, and other fields.
Today, the knowledge and applications of equations are integral to all
stages of mathematical education, ranging from elementary puzzles like the
classic chicken-rabbit cage problem to complex university-level differential
equations. Equations have evolved to become essential tools in mathemat-
ics, enabling the exploration and understanding of relationships between
various mathematical elements and uncovering the fundamental principles
that govern these relationships.
In the first 64 IMOs, there had been a total of 16 equation problems,
accounting for approximately 15.8% of all algebra problems. These prob-
lems can be primarily categorized into three types: (1) finding solutions of
equations and systems of equations, totaling seven problems; (2) proving
relationships satisfied by equations and systems of equations, totaling four
problems; (3) investigating conditions under which equations and systems
of equations have solutions, totaling five problems. The statistical distri-
bution of these three types of problems in the previous IMOs is presented
in Table 1.1.
Session
Finding solutions 7 0 0 0 0 0 0 7
Proving 2 2 0 0 0 0 0 4
relationships
Investigating 2 1 1 0 0 1 0 5
conditions
Algebra problems 20 20 14 13 15 13 6 101
The percentage 55.0% 15.0% 7.1% 0.0% 0.0% 7.7% 0.0% 15.8%
of equation
problems among
the algebra
problems
equation problems among all algebra problems was very high, exceeding
50%, but rapidly decreased thereafter.
This can partly be attributed to the fact that the study of equations
spans various stages of primary and secondary school mathematics educa-
tion. The ability to formulate and solve appropriate equations has become
an essential skill and method in dealing with mathematical problems.
Therefore, in recent years, algebra problems rarely specifically address equa-
tion problems, with the focus shifting towards more flexible topics such as
solving functional equations and proving inequalities.
Problems involving finding solutions are generally not very difficult, and
commonly used methods include factoring, the method of undetermined
coefficients, trial and error, etc. It is important to note the verification of
extraneous roots during the solving process. As for problems about proving
relationships, the relationship between the unknowns and coefficients of the
equations or systems is somehow clear, and the direction from conditions
to conclusions is often more apparent. In contrast, problems investigating
conditions tend to be more open-ended.
This chapter will be divided into three parts. The first part introduces
common equations and methods, followed by presentations of important
theorems related to equations, mainly focusing on roots and coefficients.
These theorems are highly applicable when solving equations or proving
relationships satisfied by equations. Polynomials and equations are closely
related, with additional contents related to polynomials planned for the
fifth chapter, “Other Algebra Problems.”
The second part revolves around three types of problems: “finding solu-
tions of equations and systems of equations,” “proving relationships satis-
fied by equations and systems of equations,” and “investigating conditions
under which equations and systems of equations have solutions.” These
problems are presented in chronological order, and some problems include
various solutions, generalizations, and similar problems.
It is important to note that for each problem, the solutions are fol-
lowed by information on the scores, including the number of contestants in
each score range, the average score, and the scores of the top five teams.
However, early IMOs often lacked information on contestant scores, so the
number of contestants in each score range only represents the counted num-
ber of contestants, and some problems lack scores of the top five teams.
The third part provides a brief summary of this chapter.
26 IMO Problems, Theorems, and Methods: Algebra
Some of the equation problems are quite innovative, and readers are
encouraged to contemplate the solutions before referring to the answers.
(i) When Δ > 0, the equation has two distinct real roots.
(ii) When Δ = 0, the equation has two identical real roots.
(iii) When Δ < 0, the equation has no real roots but two complex conjugate
roots.
an xn + · · · + ai xi + · · · + a1 x + a0 = 0,
(iii) When Δ < 0, there are three distinct real roots given by
⎧ √3
⎪
⎨x1 = 2 √r cos θ,
x2 = 2 3 r cos(θ + 120◦ ),
⎪
⎩ √
x3 = 2 3 r cos(θ + 240◦ ),
3
where r = − p3 and θ = 13 arccos − 2r q
.
(4) Fractional equations
Equations with unknowns in the numerators and denominators are termed
fractional (or rational) equations. Typically, methods such as clear-
ing denominators, substitution, and basic identity transformations are
employed to transform fractional equations into polynomial equations for
solving. It is important to note that this process may introduce extraneous
solutions.
(5) Irrational equations
An irrational equation, also known as a radical equation, is an equation
containing an unknown under a radical root symbol. The fundamental
approach to solving irrational equations involves transforming the “irra-
tional” into “rational.” Typically, techniques such as factoring, substitu-
tion, and completing the square are employed to convert the equation.
When dealing with irrational equations, caution must be exercised regard-
ing the possibility of extraneous solutions during the process.
(6) Systems of equations
A system of equations refers to a combination of two or more equations with
multiple unknowns. In secondary education, the most commonly encoun-
tered type is the n-variable linear system:
⎧
⎪a11 x1 + a12 x2 + · · · + a1n xn = b1 ,
⎪
⎪
⎪
⎨a x + a x + · · · + a x = b ,
21 1 22 2 2n n 2
⎪
⎪ ·································
⎪
⎪
⎩
as1 x1 + as2 x2 + · · · + asn xn = bs ,
where a11 , a12 , . . . , asn are coefficients, and b1 , b2 , . . . , bs are constants. The
number of equations may be equal to, less than, or greater than the number
of unknowns. There are three possible scenarios for the solutions of an n-
variable linear system: (i) no solution; (ii) a unique solution; (iii) an infinite
number of solutions.
28 IMO Problems, Theorems, and Methods: Algebra
√ √ 1
x+ y−1+ z − 2 = (x + y + z)
2
within the real number domain, completing the square can be employed to
transform the equation into
√ 2 √ 2
2
x−1 + y−1−1 + z−2−1 = 0.
3x + 7y = 2,
2x + y = 7,
assume
an xn + · · · + a1 x + a0 = 0(an = 0),
by listing the divisors of the leading coefficient an and the constant term
a0 , denoted as q and p, respectively, and dividing them separately, poten-
tial rational roots pq for the equation can be obtained. Subsequently, these
values are substituted back into the equation for verification.
For example, for solving the equation 2x3 − 6x2 + 3x + 2 = 0, using the
trial-and-error method yields potential rational roots as ±1, ±2, ± 12 . Upon
testing, x = 2 is found to be a rational root, leading to the factorization of
2x3 − 6x2 + 3x + 2 = (x − 2)(2x2 − 2x − 1).
(6) Substitution method
When faced with a complex equation structure, considering certain parts
as a whole and then replacing them with a new variable is a com-
mon approach. This method is often employed to reduce the number of
unknowns or decrease the degree of the equation, simplifying complex
problems.
For example, for solving the equation (x2 − 2x)2 − 3(x2 − 2x) − 4 = 0,
using substitution, let y = x2 − 2x. The original equation is then simplified
to y 2 − 3y − 4 = 0, and solving this quadratic equation yields y1 = 4
and y2 = −1. Substituting the value of y into y = x2 − 2x provides the
corresponding values for x.
30 IMO Problems, Theorems, and Methods: Algebra
···························
(−1)n a0
x1 x2 · · · xn = an .
Score Situation This particular problem saw the following distribution of scores among
contestants: 1 contestant scored 8 points, 2 contestants scored 7 points, 4 contestants scored
6 points, 1 contestant scored 5 points, 1 contestant scored 4 points, no contestant scored 3
points, no contestant scored 2 points, no contestant scored 1 point, and no contestant scored
0 point. The average score of this problem is 6.111, indicating that it was simple.
Among the top five teams in the team scores, the Romania team achieved a total score
of 249 points, the Hungary team achieved a total score of 233 points, the Czechoslovakia
team achieved a total score of 192 points, the Bulgaria team achieved a total score of 131
points, and the Poland team achieved a total score of 122 points.
The gold medal cutoff for this IMO was set at 37 points (with 3 contestants earning gold
medals), the silver medal cutoff was 36 points (with 3 contestants earning silver medals), and
the bronze medal cutoff was 33 points (with 5 contestants earning bronze medals).
In this IMO, only one contestant achieved a perfect score of 40 points, namely Bohuslav
Diviš from Czechoslovakia.
π kπ π kπ π
x1 = kπ + , x2 = + , x3 = + .
2 2 4 3 6
Here, k takes all integers, and it is easy to see that the first solution x1
is included in the third solution x3 , so it can be omitted.
(z + z −1 )2 + (z 2 + z −2 )2 + (z 3 + z −3 )2 = 4.
Equation Problems 33
Problem 1.3 (IMO 7-4, proposed by the Soviet Union). Find all
sets of four real numbers x1 , x2 , x3 , x4 such that the sum of any one and
the product of the other three is equal to 2.
Solution 1. For the system of equations
⎧
⎪
⎪ x1 + x2 x3 x4 = 2, (1)
⎪
⎪
⎪
⎨ x2 + x3 x4 x1 = 2, (2)
⎪
⎪ x3 + x4 x1 x2 = 2, (3)
⎪
⎪
⎪
⎩
x4 + x1 x2 x3 = 2, (4)
34 IMO Problems, Theorems, and Methods: Algebra
x1 = x2 = x3 = x4 .
P
xi + = 2 (i = 1, 2, 3, 4),
xi
√
which implies xi = 1 ± 1 − P (i = 1, 2, 3, 4). It is easy to see that P ≤ 1.
Case 1: If P = 1, then x1 = x2 = x3 = x4 = 1.
Case 2: If P < 1 and x1 , x2 , x3 , x4 are not all equal, then we discuss the
following three cases.
√ √
(i) xi (i = 1, 2, 3, 4) include two 1 + 1 − P and two 1 − 1 − P . Then
P = x1 x2 x3 x4
√ √
= (1 + 1 − P )2 (1 − 1 − P )2
= P 2.
P = x1 x2 x3 x4
√ √
= (1 + 1 − P )3 (1 − 1 − P )
√
= P (1 + 1 − P )2 .
√ √
(iii) xi (i = 1, 2, 3, 4) include one 1 + 1 − P and three 1 − 1 − P . Then
P = x1 x2 x3 x4
√ √
= (1 + 1 − P )(1 − 1 − P )3
√
= P (1 − 1 − P )2 .
Score Situation This particular problem saw the following distribution of scores among
contestants: 37 contestants scored 6 points, 6 contestants scored 5 points, 7 contestants
scored 4 points, 4 contestants scored 3 points, 19 contestants scored 2 points, 5 contestants
scored 1 point, and 2 contestants scored 0 point. The average score for this problem is 4.188,
indicating that it was simple.
Among the top five teams in the team scores, the scores of this problem are as follows:
the Soviet Union team scored 43 points (with a total team score of 281 points), the Hungary
team scored 38 points (with a total team score of 244 points), the Romania team scored 39
points (with a total team score of 222 points), the Poland team scored 44 points (with a
total team score of 178 points), and the German Democratic Republic team scored 34 points
(with a total team score of 175 points).
The gold medal cutoff for this IMO was set at 38 points (with 8 contestants earning gold
medals), the silver medal cutoff was 30 points (with 12 contestants earning silver medals),
and the bronze medal cutoff was 20 points (with 17 contestants earning bronze medals).
In this IMO, only two contestants achieved a perfect score of 40 points, namely László
Lovász from Hungary and Pavel Bleher from the Soviet Union.
38 IMO Problems, Theorems, and Methods: Algebra
cos2m x = 1 + sin2m x.
that the equality holds if and only if cos x = 1 or sin x = −1. Consequently,
x = 2kπ or x = 2kπ − π2 , where k is an integer.
When n = 1, the original equation
√ becomes cos x − sin x = 1, which can
be rewritten as cos(x + 4 ) = 2 . This also leads to x = 2kπ or x = 2kπ− π2 ,
π 2
where k is an integer.
Note. There is a similar problem:
Score Situation This particular problem saw the following distribution of scores among
contestants: 1 contestant scored 7 points, 1 contestant scored 6 points, 1 contestant scored
5 points, 1 contestant scored 4 points, no contestant scored 3 points, no contestant scored
2 points, 1 contestant scored 1 point, and 4 contestants scored 0 point. The average score
of this problem is 2.556, indicating that it had a certain level of difficulty.
Among the top five teams in the team scores, the Hungary team achieved a total score of
270 points, the Poland team achieved a total score of 203 points, the Romania team achieved
a total score of 197 points, the Czechoslovakia team achieved a total score of 159 points,
and the German Democratic Republic team achieved a total score of 146 points.
The gold medal cutoff for this IMO was set at 37 points (with 3 contestants earning gold
medals), the silver medal cutoff was 34 points (with 4 contestants earning silver medals), and
the bronze medal cutoff was 30 points (with 4 contestants earning bronze medals).
Equation Problems 39
In this IMO, only one contestant achieved a perfect score of 40 points, namely Béla
Bollobás from Hungary.
x2 − p + 2 x2 − 1 = x,
x2 − p + 2 x2 − 1 ≥ x2 − p > x.
2 x2 − 1 = x − x2 − p.
2x2 + p − 4 = −2x x2 − p,
It’s easy to see that p must satisfy 0 ≤ p < 2, and in this case, there is
a solution
4−p
x= .
8(2 − p)
Substituting this into the original equation and simplifying, we get
|3p − 4| = 4 − 3p, which implies p ≤ 43 .
In conclusion, the equation has a√ unique solution if and only if
0 ≤ p ≤ 43 , and the solution is x = (4−p) 4−2p
8−4p .
Note. There is a similar problem:
Score Situation This particular problem saw the following distribution of scores among
contestants: 2 contestants scored 6 points, 3 contestants scored 5 points, 4 contestants
scored 4 points, 1 contestant scored 3 points, 5 contestants scored 2 points, no contestant
scored 1 point, and 1 contestant scored 0 point. The average score of this problem is 3.500,
indicating that it was relatively straightforward.
Among the top five teams in the team scores, the Soviet Union team achieved a total
score of 271 points, the Hungary team achieved a total score of 234 points, the Romania
team achieved a total score of 191 points, the Yugoslavia team achieved a total score of 162
points, and the Czechoslovakia team achieved a total score of 151 points.
The gold medal cutoff for this IMO was set at 35 points (with 7 contestants earning gold
medals), the silver medal cutoff was 28 points (with 11 contestants earning silver medals),
and the bronze medal cutoff was 21 points (with 17 contestants earning bronze medals).
In this IMO, no contestant achieved a perfect score of 40 points.
Problem 1.6 (IMO 5-4, proposed by the Soviet Union). Find all
solutions x1 , x2 , x3 , x4 , x5 of the system
⎧
⎪ x + x2 = yx1 ,
⎪ 5
⎪
⎪
⎪
⎪
⎨x1 + x3 = yx2 ,
x2 + x4 = yx3 ,
⎪
⎪
⎪
⎪
⎪
⎪
x3 + x5 = yx4 ,
⎩
x4 + x1 = yx5 ,
where y is a parameter.
Solution. First, we sum up all five equations to obtain
(x1 + x2 + x3 + x4 + x5 )(y − 2) = 0,
which implies that either x1 + x2 + x3 + x4 + x5 = 0 or y = 2.
If y = 2, then we can deduce from the original equation:
(x1 + x3 ) + (x2 + x4 ) = 2x2 + 2x3 ⇒ x1 + x4 = x2 + x3 ,
and x1 + x4 = 2x5 . Consequently,
x1 + x2 + x3 + x4 + x5 = 5x5 .
Furthermore, x1 + x2 + x3 + x4 + x5 = 5xi (i = 1, 2, 3, 4, 5), which implies
that
x1 = x2 = x3 = x4 = x5 = k,
where k can be any real number.
Equation Problems 41
If y = 2, then
yx1 − x2 + x3 − y 2 x3 + yx2 = 0,
(1 − y 2 )x1 + yx2 + yx3 − x2 = 0,
(y 2 + y − 1)(x1 + (1 − y)x2 ) = 0,
(y 2 + y − 1)(x2 − x1 ) = 0.
The gold medal cutoff for this IMO was set at 35 points (with 7 contestants earning gold
medals), the silver medal cutoff was 28 points (with 11 contestants earning silver medals),
and the bronze medal cutoff was 21 points (with 17 contestants earning bronze medals).
In this IMO, no contestant achieved a perfect score of 40 points.
Problem 1.7 (IMO 8-5, proposed by Czechoslovakia). Solve the
system of equations
⎧
⎪
⎪ |a1 − a2 |x2 + |a1 − a3 |x3 + |a1 − a4 |x4 = 1,
⎪
⎪
⎨|a − a |x + |a − a |x + |a − a |x = 1,
2 1 1 2 3 3 2 4 4
⎪
⎪ |a3 − a1 |x1 + |a3 − a2 |x2 + |a3 − a4 |x4 = 1,
⎪
⎪
⎩
|a4 − a1 |x1 + |a4 − a2 |x2 + |a4 − a3 |x3 = 1,
where a1 , a2 , a3 , a4 are four different real numbers.
Solution. If we swap the subscripts of ai and aj , then the original system
remains unchanged. Therefore, we can assume a1 > a2 > a3 > a4 , and the
original system becomes
⎧
⎪ (a1 − a2 )x2 + (a1 − a3 )x3 + (a1 − a4 )x4 = 1, (1)
⎪
⎪
⎪
⎪
⎨ (a1 − a2 )x1 + (a2 − a3 )x3 + (a2 − a4 )x4 = 1, (2)
⎪
⎪ (a1 − a3 )x1 + (a2 − a3 )x2 + (a3 − a4 )x4 = 1, (3)
⎪
⎪
⎪
⎩
(a1 − a4 )x1 + (a2 − a4 )x2 + (a3 − a4 )x3 = 1. (4)
(1)–(2) yields (a1 − a2 )(x3 + x2 + x4 − x1 ) = 0.
(2)–(3) yields (a2 − a3 )(x3 + x4 − x1 − x2 ) = 0.
(3)–(4) yields (a3 − a4 )(x4 − x1 − x2 − x3 ) = 0.
Since ai are all distinct, we have
⎧
⎨x2 + x3 + x4 − x1 = 0,
⎪
x3 + x4 − x1 − x2 = 0,
⎪
⎩
x4 − x1 − x2 − x3 = 0.
1
Solving this system results in x1 = x4 = a1 −a4 and x2 = x3 = 0.
Note. Using the same method, we can solve a system of n equations:
⎧
⎪
⎪ |a1 − a2 |x2 + |a1 − a3 |x3 + · · · + |a1 − an |xn = 1,
⎪
⎪
⎨|a2 − a1 |x1 + |a2 − a3 |x3 + · · · + |a2 − an |xn = 1,
⎪· · · · · · · · · · · · · · · · · · · · · · · · · · · · · · · · · · · · · · · · · · · · · · · · · · ·
⎪
⎪
⎪
⎩
|an − a1 |x1 + |an − a2 |x2 + · · · + |an − an−1 |xn−1 = 1.
1
If a1 > a2 > · · · > an , then the solution is x1 = xn = a1 −an and x2 = x3 =
· · · = xn−1 = 0.
Equation Problems 43
Score Situation This particular problem saw the following distribution of scores among
contestants: 15 contestants scored 7 points, no contestant scored 6 points, 1 contestant
scored 5 points, 3 contestants scored 4 points, 1 contestant scored 3 points, 4 contestants
scored 2 points, 2 contestants scored 1 point, and 1 contestant scored 0 point. The average
score of this problem is 5.000, indicating that it was simple.
Among the top five teams in the team scores, the Soviet Union team achieved a total
score of 293 points, the Hungary team achieved a total score of 281 points, the German
Democratic Republic team achieved a total score of 280 points, the Poland team achieved a
total score of 269 points, and the Romania team achieved a total score of 257 points.
The gold medal cutoff for this IMO was set at 39 points (with 13 contestants earning gold
medals), the silver medal cutoff was 34 points (with 15 contestants earning silver medals),
and the bronze medal cutoff was 31 points (with 11 contestants earning bronze medals).
In this IMO, a total of 11 contestants achieved a perfect score of 40 points.
The last column can be replaced with the sum of all three columns,
resulting in
a11 a12 S1
D = a21 a22 S2 ,
a a32 S3
31
Note that the first two terms in this expression are positive, from con-
ditions (a) and (b). It is also apparent that from conditions (b) and (c),
a11 + a12 > a11 + a12 + a13 > 0 ⇒ a11 > −a12 = |a12 |,
a22 + a21 > a22 + a21 + a23 > 0 ⇒ a22 > −a21 = |a21 |.
Therefore a11 a22 > a12 a21 , making the third term in the above expres-
sion positive. Hence, D > 0, and as a result, the original system of equa-
tions has only the trivial solution x1 = x2 = x3 = 0.
• (Germany Team Selection Test 2004, Problem 7). Let aij , where
i, j ∈ {1, 2, 3}, be real numbers such that aij is positive for i = j and
negative for i = j. Prove the existence of positive real numbers c1 , c2 , c3
such that the numbers
Score Situation This particular problem saw the following distribution of scores among
contestants: 19 contestants scored 6 points, 6 contestants scored 5 points, 4 contestants
scored 4 points, 6 contestants scored 3 points, 10 contestants scored 2 points, 18 contestants
scored 1 point, and 17 contestants scored 0 point. The average score for this problem is
2.700, indicating that it had a certain level of difficulty.
Among the top five teams in the team scores, the scores of this problem are as follows:
the Soviet Union team scored 46 points (with a total team score of 281 points), the Hungary
team scored 36 points (with a total team score of 244 points), the Romania team scored 35
points (with a total team score of 222 points), the Poland team scored 25 points (with a
total team score of 178 points), and the German Democratic Republic team scored 17 points
(with a total team score of 175 points).
The gold medal cutoff for this IMO was set at 38 points (with 8 contestants earning gold
medals), the silver medal cutoff was 30 points (with 12 contestants earning silver medals),
and the bronze medal cutoff was 20 points (with 17 contestants earning bronze medals).
In this IMO, only two contestants achieved a perfect score of 40 points, namely László
Lovász from Hungary and Pavel Bleher from the Soviet Union.
xk + xk+1 = x2k+2 , k = 1, 2, . . . , n,
where xn+i = xi .
• (Polish Mathematical Olympiad 2000, Problem 1). For a given
integer n ≥ 2, find the number of non-negative real solutions of the
system of equations:
⎧
⎪ x1 + x2n = 4xn ,
⎪
⎪
⎪
⎪ 2
⎪
⎨x2 + x1 = 4x1 ,
x3 + x22 = 4x2 ,
⎪
⎪
⎪
⎪ ··················
⎪
⎪
⎩
xn + x2n−1 = 4xn−1 .
• (Turkey Team Selection Test 1995, Problem 1). Given real num-
bers b ≥ a > 0, find all solutions of the system
⎧ 2
⎪ x1 + 2ax1 + b2 = x2 ,
⎪
⎪
⎪
⎪ 2 2
⎪
⎨x2 + 2ax2 + b = x3 ,
························
⎪
⎪
⎪
⎪ x2n−1 + 2axn−1 + b2 = xn ,
⎪
⎪
⎩ 2
xn + 2axn + b2 = x1 .
48 IMO Problems, Theorems, and Methods: Algebra
Score Situation This particular problem saw the following distribution of scores among
contestants: 38 contestants scored 7 points, 9 contestants scored 6 points, 1 contestant
scored 5 points, 2 contestants scored 4 points, no contestant scored 3 points, 7 contestants
scored 2 points, 21 contestants scored 1 point, and 13 contestants scored 0 point. The
average score of this problem is 4.044, indicating that it was simple.
Among the top five teams in the team scores, the German Democratic Republic team
achieved a total score of 304 points, the Soviet Union team achieved a total score of 298
points, the Hungary team achieved a total score of 291 points, the United Kingdom team
achieved a total score of 263 points, and the Poland team achieved a total score of 262
points.
The gold medal cutoff for this IMO was set at 39 points (with 22 contestants earning gold
medals), the silver medal cutoff was 33 points (with 22 contestants earning silver medals),
and the bronze medal cutoff was 26 points (with 20 contestants earning bronze medals).
In this IMO, a total of 16 contestants achieved a perfect score of 40 points.
Since the cosine function is strictly monotonic on [0, π], the values
2 cos tk (k = 0, 1, . . . , 2n−1 − 1)
2n − 1 k
= .
2n + 1 l
Since k < 2n − 1 and l < 2n + 1, it indicates that the fraction on the left
side of the equation has not been reduced to its simplest form. However,
2n − 1 and 2n + 1 are consecutive odd numbers and coprime, meaning they
cannot have a common factor greater than 1. This implies that 2 cos tk and
2 cos sl are all 2n distinct real roots of the equation pn (x) = x.
y y=x
2
p2
1
p1
–2 –1 O 1 2 x
–1
Score Situation This particular problem saw the following distribution of scores among
contestants: 25 contestants scored 7 points, 5 contestants scored 6 points, 4 contestants
scored 5 points, no contestant scored 4 points, 4 contestants scored 3 points, 5 contestants
scored 2 points, 22 contestants scored 1 point, and 74 contestants scored 0 point. The
average score for this problem is 1.935, indicating that it was relatively challenging.
Among the top five teams in the team scores, the scores of this problem are as follows:
the Soviet Union team scored 46 points (with a total team score of 250 points), the United
Kingdom team scored 35 points (with a total team score of 214 points), the United States
team scored 18 points (with a total team score of 188 points), the Bulgaria team scored 18
points (with a total team score of 174 points), and the Austria team scored 16 points (with
a total team score of 167 points).
The gold medal cutoff for this IMO was set at 34 points (with 9 contestants earning gold
medals), the silver medal cutoff was 23 points (with 28 contestants earning silver medals),
and the bronze medal cutoff was 15 points (with 45 contestants earning bronze medals).
In this IMO, only one contestant achieved a perfect score of 40 points, namely Laurent
Pierre from France.
Equation Problems 51
with every coefficient aij a member of the set {−1, 0, 1}. Prove that the
system has a solution (x1 , x2 , . . . , xq ) satisfying:
Thus, ai1 x1 +ai2 x2 +· · ·+aiq xq can take at most 2pq +1 possible values.
Consequently, the values corresponding to the left side of the system can
take at most (2pq + 1)p possible sets of integers (b1 , b2 , . . . , bp ).
For each xj , let it take 2p + 1 values ranging from −p to p, resulting in
(2p + 1)q possible sets of integers (b1 , b2 , . . . , bp ). Since
Note. This problem can also be solved from a matrix perspective. Let A
represent the coefficient matrix of the system of equations, 0 represent the
constant term vector, and X the unknown vector, that is, let
⎛ ⎞ ⎛ ⎞ ⎛ ⎞
a11 a12 · · · a1q 0 x1
⎜ ⎟ ⎜ ⎟ ⎜ ⎟
⎜a21 a22 · · · a2q ⎟ ⎜0 ⎟ ⎜x2 ⎟
⎜ ⎟ ⎜ ⎟ ⎜ ⎟
A=⎜ . . . ⎟, 0 = ⎜ . ⎟, X = ⎜ . ⎟.
⎜ .. .. .. ⎟ ⎜ .. ⎟ ⎜ .. ⎟
⎝ ⎠ ⎝ ⎠ ⎝ ⎠
ap1 ap2 · · · apq 0 xq
a cos2 x + b cos x + c = 0.
a cos2 x + c = −b cos x,
simplifying to
4 cos2 2x + 2 cos 2x − 1 = 0.
This quadratic equation shares the same coefficients as the given equa-
tion involving cos x.
Score Situation This particular problem saw the following distribution of scores among
contestants: 4 contestants scored 7 points, no contestant scored 6 points, 2 contestants
scored 5 points, no contestant scored 4 points, no contestant scored 3 points, no contestant
scored 2 points, no contestant scored 1 point, and 3 contestants scored 0 point. The average
score of this problem is 4.222, indicating that it was simple.
Among the top five teams in the team scores, the Romania team achieved a total score
of 249 points, the Hungary team achieved a total score of 233 points, the Czechoslovakia
team achieved a total score of 192 points, the Bulgaria team achieved a total score of 131
points, and the Poland team achieved a total score of 122 points.
The gold medal cutoff for this IMO was set at 37 points (with 3 contestants earning gold
medals), the silver medal cutoff was 36 points (with 3 contestants earning silver medals), and
the bronze medal cutoff was 33 points (with 5 contestants earning bronze medals).
In this IMO, only one contestant achieved a perfect score of 40 points, namely Bohuslav
Diviš from Czechoslovakia.
where a and b are constants. Give conditions that a and b must satisfy, so
that x, y, z (the solutions of the system) are distinct positive numbers.
Equation Problems 55
Solution. Since the solutions of the system of equations are distinct pos-
itive numbers, it is evident that a > 0. Additionally, it is easy to observe
that
b2 = x2 + y 2 + z 2 = x2 + y 2 + 2xy − z 2 = (x + y + z)(x + y − z),
which leads to
b2
x+y−z = ,
a
(x + y + z) − (x + y − z) a2 − b 2
z= = ,
2 2a
a2 + b 2
x+y = ,
2a
(a2 − b2 )2
xy = .
4a2
a2 +b2 (a2 −b2 )2
Consider the quadratic equation f (x) = x2 − 2a x + 4a2 . Then,
1
Δ= ((a2 + b2 )2 − 4(a2 − b2 )2 )
4a2
1
= 2 (3a2 − b2 )(3b2 − a2 ) ≥ 0,
4a
and in this case, the system of equations has solutions
⎧ 1
⎪
⎪ x = 4a (a2 + b2 ± (3a2 − b2 )(3b2 − a2 )),
⎨
1
y = 4a (a2 + b2 ∓ (3a2 − b2 )(3b2 − a2 )),
⎪
⎪
⎩ 1
z = 2a (a2 − b2 ).
To ensure that the solutions are positive, z > 0, implying a2 > b2 .
Since x = y, we have Δ > 0. Additionally, as 3a2 − b2 > a2 − b2 > 0,
there holds 3b2 > a2 .
Therefore,
√
0 < |b| < a < 3|b|.
2 2
√
It is evident that a 4a
+b
> 2Δ . Thus, x > 0 and y > 0. Furthermore,
√
since x, y are distinct positive numbers, and z = xy, it is clear that x, y, z
are distinct positive numbers.
Note. From the given conditions, it is known that the distance from the
plane x + y + z = a > 0 to the origin is √a3 > 0. The radius of the sphere
56 IMO Problems, Theorems, and Methods: Algebra
a total score of 197 points, the Czechoslovakia team achieved a total score of 159 points,
and the German Democratic Republic team achieved a total score of 146 points.
The gold medal cutoff for this IMO was set at 37 points (with 3 contestants earning gold
medals), the silver medal cutoff was 34 points (with 4 contestants earning silver medals), and
the bronze medal cutoff was 30 points (with 4 contestants earning bronze medals).
In this IMO, only one contestant achieved a perfect score of 40 points, namely Béla
Bollobás from Hungary.
x4 + ax3 + bx2 + ax + 1 = 0
has at least one real solution. For all such pairs (a, b), find the minimum
value of a2 + b2 .
−a ± a2 − 4(b − 2)
y= ,
2
where at least one of the roots has an absolute value not less than 2. This
is equivalent to
|a| + a2 − 4(b − 2) ≥ 4,
or
a2 − 4(b − 2) ≥ 4 − |a|.
2|a| ≥ 2 + b.
58 IMO Problems, Theorems, and Methods: Algebra
Score Situation This particular problem saw the following distribution of scores among con-
testants: 42 contestants scored 8 points, 4 contestants scored 7 points, 8 contestants scored
6 points, 4 contestants scored 5 points, 4 contestants scored 4 points, 6 contestants scored
Equation Problems 59
Problem 1.15 (IMO 21-5, proposed by Israel). Find all real numbers
a for which there exist non-negative real numbers x1 , x2 , x3 , x4 , x5 satisfying
the relations
5 5 5
kxk = a, k 3 xk = a2 , k 5 xk = a3 .
k=1 k=1 k=1
and hence
5
k(a − k 2 )2 xk = 0.
k=1
This implies that each of the five non-negative terms is zero, i.e.,
k(a − k 2 )2 xk = 0, k = 1, 2, 3, 4, 5.
5
2
(a2 )2 = k 3 xk
k=1
5
5
5
≤ kxk k xk
k=1 k=1
= a4 .
√
The equality implies the existence of a real number λ such that k 5 xk =
√
λ kxk , or k 4 xk = λ2 xk . In other words,
(k 4 − λ2 )xk = 0, k = 1, 2, 3, 4, 5.
Score Situation This particular problem saw the following distribution of scores among
contestants: 46 contestants scored 7 points, 2 contestants scored 6 points, 7 contestants
scored 5 points, 4 contestants scored 4 points, 10 contestants scored 3 points, 31 contestants
scored 2 points, 42 contestants scored 1 point, and 24 contestants scored 0 point. The
average score for this problem is 3.127, indicating that it was relatively straightforward.
Among the top five teams in the team scores, the scores of this problem are as follows:
the Soviet Union team scored 41 points (with a total team score of 267 points), the Romania
team scored 40 points (with a total team score of 240 points), the Germany team scored 17
points (with a total team score of 235 points), the United Kingdom team scored 35 points
(with a total team score of 218 points), and the United States team scored 38 points (with
a total team score of 199 points).
The gold medal cutoff for this IMO was set at 37 points (with 8 contestants earning gold
medals), the silver medal cutoff was 29 points (with 32 contestants earning silver medals),
and the bronze medal cutoff was 20 points (with 42 contestants earning bronze medals).
In this IMO, a total of four contestants achieved a perfect score of 40 points.
Equation Problems 61
Multiplying these inequalities results in the left side of (1) being less
than the right side, making (1) false.
Case 4: x ∈ (4k + 2, 4k + 3), where k ∈ {0, 1, 2, . . . , 503}.
For j ∈ {1, 2, . . . , 503}, the inequalities
are true.
Additionally, x − 1 > x − 2 > 0 > x − 2015 > x − 2016. Multiplying
these inequalities results in
503
503
(x − 4j − 1)(x − 4j − 4) < (x − 4j − 2)(x − 4j − 3) < 0.
j=0 j=0
• (Turkey Tean Selection Test 2023, Problem 8). Initially, the equa-
tion
1 1 1 1
··· =0
x−1 x−2 x−4 x − 22023
is written on the board. In each turn Asli and Zehra deletes one of the
stars in the equation and writes + or − instead. The first move is per-
formed by Asli and continues in order. What is the maximum number
of real solutions Asli can guarantee after all the stars have been replaced
by signs?
• (Chinese Team Selection Test 2014, Problem 15). Show that there
are no 2-tuples (x, y) of positive integers satisfying the equation
|x − a1 | + |x − a2 | + · · · + |x − a50 | = |x − b1 | + |x − b2 | + · · · + |x − b50 |
Score Situation This particular problem saw the following distribution of scores among
contestants: 81 contestants scored 7 points, 4 contestants scored 6 points, 2 contestants
scored 5 points, 50 contestants scored 4 points, 21 contestants scored 3 points, 55 contestants
scored 2 points, 36 contestants scored 1 point, and 353 contestants scored 0 point. The
average score for this problem is 1.678, indicating that it was relatively challenging.
Among the top five teams in the team scores, the scores of this problem are as follows:
the United States team scored 41 points (with a total team score of 214 points), the South
Korea team scored 33 points (with a total team score of 207 points), the China team scored
42 points (with a total team score of 204 points), the Singapore team scored 42 points (with
a total team score of 196 points), and the Chinese Taiwan team scored 30 points (with a
total team score of 175 points).
The gold medal cutoff for this IMO was set at 29 points (with 44 contestants earning gold
medals), the silver medal cutoff was 22 points (with 101 contestants earning silver medals),
and the bronze medal cutoff was 16 points (with 135 contestants earning bronze medals).
In this IMO, a total of six contestants achieved a perfect score of 42 points.
1.3 Summary
In the first 64 IMOs, there were a total of 16 equation problems. These
problems can be broadly categorized into three types, as depicted in
Figure 1.2. The score details for these problems are presented in Table 1.2.
Due to the smaller number of participating teams and missing contestant
score information in early IMOs, there are several blanks in Table 1.2.
Problems 1.1–1.7 focus on “finding solutions of equations and systems
of equations;” among these seven problems, the one with the lowest average
64 IMO Problems, Theorems, and Methods: Algebra
8
7
Finding Solutions Proving Relationships
6
Investigating Conditions
5
4
3
2
1
0
1–10 11–20 21–30 31–40 41–50 51–60 61–64
score is Problem 1.6 (IMO 5-4), proposed by the Soviet Union. Problems
1.8–1.11 deal with “proving relationships satisfied by equations and systems
of equations;” among these four problems, the one with the lowest average
score is Problem 1.11 (IMO 18-5), proposed by the Netherlands. Problems
1.12–1.16 are about “investigating conditions under which equations and
systems of equations have solutions;” among these five problems, the one
with the lowest average score is Problem 1.16 (IMO 57-5), proposed by
Russia.
These 16 problems were proposed by 11 countries. The Soviet Union,
Romania, Bulgaria, Czechoslovakia, and Hungary each contributed two
problems.
From Table 1.2, it can be observed that in the first 64 IMOs, there were
four equation problems with an average score of 1–2 points; two problems
with an average score of 2–3 points; five problems with an average score
of 3–4 points; five problems with an average score above 4 points. Overall,
the equation problems were relatively simple, and the contestants scored
relatively high.
In the 24th–64th IMOs, there was only one equation problem, specifi-
cally Problem 1.16 (IMO 57-5), which had an average score of 1.678 points.
This indicates that equation problems were primarily featured in the first
23 IMOs, as shown in Table 1.3.
From Table 1.2, it can be observed that, excluding Problem 1.16 (IMO
57-5), the average score of the top five teams is typically about 1 point
higher than the average score of the problem. However, the average score
Table 1.2 Score Details of Equation Problems in the First 64 IMOs
Full points 8.000 5.000 6.000 7.000 6.000 6.000 7.000 6.000
Average score 6.111 3.941 4.188 2.556 3.500 1.375 5.000 2.700
Top five mean 4.950 3.975
6th–15th mean
16th–25th mean
Problem number 1-2 4-4 7-4 3-3 5-1 5-4 8-5 7-2
in IMO
Equation Problems
Proposing Romania Romania The Soviet Bulgaria Czecho- The Soviet Czecho- Poland
country Union slovakia Union slovakia
Full points 7.000 7.000 7.000 7.000 6.000 8.000 7.000 7.000
Average score 4.044 1.935 1.094 4.222 3.222 3.992 3.127 1.678
Top five mean 3.325 2.050 4.700 4.275 6.267
6th–15th mean 1.563 0.863 3.850 3.434 4.485
16th–25th mean 2.636
Problem number 10-3 18-2 18-5 1-3 3-1 15-3 21-5 57-5
in IMO
Proposing Bulgaria Finland The Netherlands Hungary Hungary Sweden Israel Russia
country
Note. Top five mean = Total score of the top five teams ÷ Total number of contestants from the top five teams,
6th–15th mean = Total score of the 6th–15th teams ÷ Total number of contestants from the 6th–15th teams,
16th–25th mean = Total score of the 16th–25th teams ÷ Total number of contestants from the 16th–25th teams.
65
66 IMO Problems, Theorems, and Methods: Algebra
Problem Number
Number of Problems in
Equation Problem 1, 4 2, 5 3, 6 the First 64 IMOs
Finding solutions 0 0 0 7
Proving relationships 0 0 0 4
Investigating conditions 0 1 0 5
Total 0 1 0 16
of the 6th–15th teams tends to be close to or below the average score of the
problem, as seen in Problem 1.10 (IMO 18-2), Problem 1.11 (IMO 18-5),
Problem 1.14 (IMO 15-3), and Problem 1.15 (IMO 21-5).
This phenomenon is due to the smaller number of participating teams
in early IMOs. It was not until the 22nd IMO in 1981 that the number of
participating teams exceeded 25. Furthermore, the relatively low level of
difficulty associated with equation problems during this period suggests a
minimal variation in the average scores between the overall, top five, and
6th–15th teams.
Chapter 2
Function Problems
67
68 IMO Problems, Theorems, and Methods: Algebra
Session
Proving properties 1 4 1 1 0 2 0 9
Determining values 0 1 3 0 0 0 0 4
Deriving expressions 0 0 2 4 3 5 1 15
Algebra problems 20 20 14 13 15 13 6 101
The percentage of 5.0% 25.0% 42.9% 38.5% 20.0% 53.8% 16.7% 27.7%
function problems
among the algebra
problems
with z as any real number, but his derivation was not rigorous. Cauchy
further refined Newton’s work. On this basis, Cauchy defined the function
and proved that f (z + w) = f (z)f (w) for any real numbers z and w, known
as Cauchy’s exponential functional equation.
Later, many renowned mathematicians, such as Jean Le Rond
d’Alembert, Léonard Euler, Carl Friedrich Gauss, Adrien-Marie Legendre,
Niels Henrik Abel, and David Hilbert, contributed to the study of func-
tional equations, leading to equations named after them. Today, func-
tional equations remain an actively developing branch of mathematics, with
widespread applications in various scientific and technical fields.
Function Problems 69
denoted as
y = f (x), x ∈ A.
f −1 : {f (x)|x ∈ A} → A,
or
z = f (y), where y ∈ E,
y = g(x), where x ∈ D.
If the set D ∗ = {x ∈ D|g(x) ∈ E} is non-empty, then a new function
can be defined with elements from D∗ as the independent variable, passing
through the successive actions of g and f , and elements from the range of
f as the function value, denoted as
z = f (g(x)), x ∈ D∗ .
Proposition 2.1. Only the zero function can be both odd and even.
Proposition 2.3. The sum and difference of two odd (even) functions are
odd (even) functions, and an odd (even) function multiplied by a non-zero
constant remains odd (even).
Proposition 2.4. The product and quotient of two odd (even) functions
are even functions, while the product and quotient of an odd function and
an even function are odd functions.
More generally, for a function f (x) with domain R, the following holds:
(2) Monotonicity
Let the domain of a function f (x) be D. If for any x1 , x2 ∈ D1 ⊂ D,
it holds that f (x1 ) < f (x2 ) whenever x1 < x2 , then f (x) is said to be
(strictly) increasing on D1 ; if f (x1 ) > f (x2 ) whenever x1 < x2 , then f (x)
is said to be (strictly) decreasing on D1 .
If the function f (x) is increasing or decreasing on a subset of its domain
(usually an interval), then f (x) is said to be (strictly) monotonic on this
subset. This subset is referred to as the monotonic interval of f (x).
It should be noted that the monotonicity of a function pertains to a
subset of its domain and is a “local” property. Even if f (x) is increasing
(decreasing) on subintervals D1 and D2 of its domain, it cannot be directly
inferred that f (x) is increasing (decreasing) on D1 ∪ D2 .
Furthermore, in advanced mathematics, the definition of monotonicity
is slightly weaker, indicating that f (x1 ) ≤ f (x2 ) or f (x1 ) ≥ f (x2 ) for
x1 < x2 .
The following propositions are relatively straightforward to prove by
using definitions.
Proposition 2.11. If two positive-valued functions f (x) and g(x) are both
increasing (decreasing) on a common interval I, then the function f (x)g(x)
is also increasing (decreasing) on I.
Function Problems 75
(3) Periodicity
Let the domain of a function f (x) be D. If there exists a non-zero constant
T such that x + T ∈ D and f (x + T ) = f (x) for every x ∈ D, then f (x) is
termed a periodic function, and T is known as a period of the function f (x).
If among all positive periods of the function f (x), there exists the small-
est value T0 , then T0 is called the fundamental period of the periodic func-
tion f (x).
However, not every periodic function has a fundamental period. For
instance, a constant function can have any real number as a period, and
the function
1, x ∈ Q,
d(x) =
0, x ∈/Q
can have any rational number as a period.
The following propositions are relatively straightforward to prove by
using definitions.
Proposition 2.14. If a function f (x) with domain R is symmetric about
the lines x = a and x = b, where a = b, then f (x) is periodic with a period
T = 2|a − b|.
In particular, f (x) being an even function is equivalent to f (x) being
symmetric about the line x = 0.
Proposition 2.15. If a function f (x) with domain R is symmetric about
the points (a, 0) and (b, 0), where a = b, then f (x) is periodic with a period
T = 2|a − b|.
76 IMO Problems, Theorems, and Methods: Algebra
Proposition 2.18. If f (x) and g(x) are both convex functions defined on I,
then h1 (x) = f (x) + g(x) and h2 (x) = max{f (x), g(x)} are both convex
functions.
Proposition 2.19. If f (x) and g(x) are both convex functions and g(x) is
increasing, then g(f (x)) is a convex function.
(5) Continuity
A continuous function is one in which a sufficiently small change in the inde-
pendent variable results in a correspondingly small change in the function
value. However, this is not the rigorous definition of a continuous function,
which relies on the concept of limits in advanced mathematics.
Let x0 be a point in the domain D of the function f (x). If for any
sequence {xn } ⊂ D converging to x0 , the sequence {f (xn )} tends to f (x0 ),
then f (x) is said to be continuous at x0 .
Suppose the domain of the function f (x) is D, and the interval I is a
subset of D. If f (x) is continuous at every point x in I, then f (x) is said
to be continuous on the interval I.
Intuitively, functions whose graphs are unbroken curves are continuous.
For example, polynomial function, power function, exponential function,
logarithmic function, trigonometric function, and the absolute value func-
tion are all continuous on subintervals of their domains.
The following propositions are relatively straightforward to prove by
using definitions.
Let f (x) be a bijection from D to D. Define f (−1) (x) = f −1 (x), i.e., the
inverse function of f (x). Then, for any positive integer n, define f (−n) (x)
as the nth iteration of f (−1) (x).
Common forms of function iterations include:
(i) The nth iteration of the function f (x) = x + c is f (n) (x) = x + nc.
n
(ii) The nth iteration of the function f (x) = xm is f (n) (x) = xm .
(iii) The nth iteration of the function f (x) = ax + b is
b b
f (n) (x) = an x − + .
1−a 1−a
x
(iv) The nth iteration of the function f (x) = a+bx is
x
f (n) (x) = 1−an .
an + bx · 1−a
f (x) = ax + b,
Function Problems 79
implying that the proposition also holds for n = k + 1, and thus for any
positive integer n.
This implies that if functions f (x) and g(x) are similar, then finding the
nth iteration of f (x) is essentially equivalent to finding the nth iteration
of g(x). If the nth iteration of g(x) is easier to determine, then the problem
becomes more straightforward.
x
Example 2.3. Let f (x) = 1+ax and find f (n) (x).
(i) If x0 is a fixed point of f (x), then x0 is also a fixed point of f (n) (x).
(ii) If f ∼ϕ g and x0 is a fixed point of f (x), then ϕ(x0 ) is a fixed point of
g(x).
Note that when using the similarity method, g(x) is often chosen as
x + 1, ax, ax2 , ax3 , etc., where the fixed points of g(x) are 0 or +∞.
Therefore, based on the properties of fixed points, the chosen bridge func-
tion ϕ(x) should map the fixed points of f (x) to 0 or +∞.
If f (x) has only one fixed point x0 , then consider ϕ(x) = x − x0 or
1
ϕ(x) = x−x 0
. If f (x) has two distinct fixed points α and β, then consider
x−α
ϕ(x) = x−β .
Function Problems 81
x+6
Example 2.4. Let f (x) = x+2 and find f (n) (x).
Solution. Setting f (x) = x, we find that the fixed points of f (x) are
2 and −3. Let ϕ(x) = x−2 x+3 and g(x) = − 4 x, and it is easy to verify
1
(i) For functions like f (x) = a(x − h)k + h(a = 0, k ∈/ {0, 1}), a bridge
function can be ϕ(x) = x − h.
(ii) For functions like
√
k x k
axk + b
f1 (x) = ax + b, f2 (x) = √
k
k
, f 3 (x) = ,
k
ax + b x
Note that the expression of the nth iteration of most functions f (x) is
not easy to determine and can be quite complex. In such cases, it is more
common to study the properties of f (n) (x), such as given x0 , examining the
sequence
f (x + y) = f (x) · f (y),
f (x + y) = g(x) + h(y),
Find f (x).
Example 2.7. Find all functions f : R+ ∪ {0} → R+ ∪ {0} such that for
any x ≥ 0,
Solution. For any fixed x0 ≥ 0, let an = f (n) (x0 ), thus obtaining a non-
negative real number sequence {an }. This sequence satisfies the recurrence
relation an+2 = −3an+1 + 10an, with its characteristic equation being x2 +
3x − 10 = 0, and roots 2 and −5. Therefore, an = 2n A + (−5)n B, where
A, B are constants. n
Since an = 2n A + B − 52 , if B = 0, then an can be negative when
n is a sufficiently large odd integer. Hence, B must be zero, and an = 2n A.
Given a0 = x0 = A, we find f (n) (x0 ) = an = 2n x0 . In particular,
f (x0 ) = 2x0 , which holds true for any x0 ≥ 0. Upon verification, the only
function satisfying the conditions is f (x) = 2x.
On the other hand, the recursive relation can be used to determine a
series of function values. For instance, if the values of f (1), f (2), . . . , f (n)
are known and the value of f (n+1) is uniquely determined by the recurrence
relation, then one can also employ the characteristic equation.
(4) Fixed point method
This method is commonly used to solve functional equations involving func-
tion composition. If the function in the functional equation has a fixed
point, then the solutions can be derived by using this fixed point and the
properties of the function itself.
making xk+1 = x2k + 1 also a fixed point of P (x). Thus, there exists an
infinite sequence
0, 1, 2, 5, 26, . . .
such that each term is a fixed point of P (x), i.e., roots of P (x) − x = 0.
However, since P (x) is a polynomial, P (x)−x = 0 can only have a finite
number of roots, implying P (x) − x is identically zero, i.e., P (x) = x.
Example 2.10. Find all functions f : R+ → R+ such that for any positive
real numbers x and y,
x + f (y) 1 1
=f +f .
xf (y) y x
Solution. It is easy to see that f (x) = x is a solution of the functional
equation.
Suppose there exists x0 > 0 such that x10 > f x10 . Let x = x0 and
1
y= 1
. Then
x0 −f x1
0
x + f (y) 1 1 1 1
= + = + ,
xf (y) x f (y) x0 f (y)
1 1 1 1 1 1
f +f =f −f +f =f .
y x x0 x0 x0 x0
1 1 1 1 1
Thus, x0 + f (y) =f x0 , i.e., x0 <f x0 , a contradiction.
Therefore, for any positive real number x0 , there is 1
x0 ≤f 1
x0 , i.e.,
x ≤ f (x).
1 1
Also, x1 + 1
f (y) =f 1
y +f x ≥ 1
y +f x ≥ 1
y + x1 , i.e., y ≥ f (y).
Consequently, the solution to the functional equation is f (x) = x.
(7) Calculus method
For some types of functional equations, it is sometimes possible to trans-
form the functional equation into a form like f (x) = g(x) (where g(x) is
integrable). Then integrate to get
f (x) = g(x)dx + C,
and finally determine the expression of f (x) based on the given conditions.
Example 2.11. Find the real coefficient polynomial f (x) such that for any
real coefficient polynomial g(x), the equality f (g(x)) = g(f (x)) is true.
Function Problems 89
f (x + y) + f (x − y) = 2f (x) cos y.
f (π + t) + f (t) = 0. (II)
π π
Set x = 2 and y = 2 + t to get
π
f (π + t) + f (−t) = −2f sin t. (III)
2
π
From (I) + (II) − (III), f (t) = f (0) cos πt+ f 2 sin t, i.e., f (x) =
a cos x + b sin x, where a = f (0) and b = f 2 are constants. Upon verifi-
cation, f (x) = a cos x + b sin x satisfies the conditions.
(iii) Mathematical Induction
This method is suitable for functional equations whose domain is the set of
positive integers. It begins with concrete calculations of f (1), f (2), f (3), . . .,
leading to a conjecture about the expression of f (n), and then use mathe-
matical induction to prove the conjecture.
Example 2.14. Solve the functional equation: f (n+1) = cos θ+f (n) sin θ,
where f (1) = cos θ with θ ∈ [0, 2π) and n ∈ N+ .
f (f (x) + y) = f (x + y) + f (0)
(a) Prove that the function f is periodic (i.e., there exists a positive number
b such that f (x + b) = f (x) for all x).
(b) For a = 1, give an example of a non-constant function with the required
properties.
Proof. (a) Since the equation (*) holds for any x ∈ R, we have
f (x + 2a) = f ((x + a) + a)
1
= + f (x + a) − (f (x + a))2
2
1
+ f (x) − (f (x))2
1 2
= +
2 1
− + f (x) − (f (x))2 + f (x) − (f (x))2
4
1 1
= + − f (x) + (f (x))2
2 4
1 1
= + − f (x).
2 2
Function Problems 93
Since f (x + a) ≥ 1
2 for any x ∈ R, we see that f (x) ≥ 12 . Therefore
1 1
f (x + 2a) = + f (x) − = f (x).
2 2
Thus, f (x) is a periodic function with a period of 2a.
(b) When a = 1, the following is a periodic function with the prescribed
property:
1 1 πx
f (x) = + sin .
2 2 2
Its period is 2.
The gold medal cutoff for this IMO was set at 39 points (with 22 contestants earning gold
medals), the silver medal cutoff was 33 points (with 22 contestants earning silver medals),
and the bronze medal cutoff was 26 points (with 20 contestants earning bronze medals).
In this IMO, a total of 16 contestants achieved a perfect score of 40 points.
1 1
f (x) = cos(a1 + x) + cos(a2 + x) + cos(a3 + x)
2 4
1
+ ···+ cos(an + x).
2n−1
Given that f (x1 ) = f (x2 ) = 0, prove that x2 −x1 = mπ for some integer m.
1 1 1
f (−a1 ) = 1 + cos(a2 − a1 ) + cos(a3 − a1 ) + · · · + n−1 cos(an − a1 )
2 4 2
1 1 1
≥ 1 − − − · · · − n−1
2 4 2
1
= n−1 > 0.
2
= A cos x − B sin x,
n 1 n 1
where A = k=1 2k−1 cos ak and B = k=1 2k−1 sin ak , and they cannot
both be zero. Otherwise, f (x) would be identically zero, leading to a con-
tradiction.
If A = 0, then
Prove that
Score Situation This particular problem saw the following distribution of scores among
contestants: 34 contestants scored 7 points, 5 contestants scored 6 points, 12 contestants
scored 5 points, 32 contestants scored 4 points, 6 contestants scored 3 points, 5 contestants
scored 2 points, 5 contestants scored 1 point, and 13 contestants scored 0 point. The average
score of this problem is 4.366, indicating that it was simple.
Among the top five teams in the team scores, the scores of this problem are as follows:
the Hungary team scored 53 points (with a total team score of 247 points), the German
Democratic Republic team scored 45 points (with a total team score of 240 points), the
Soviet Union team scored 52 points (with a total team score of 231 points), the Romania
team scored 36 points (with a total team score of 219 points), and the United Kingdom team
scored 43 points (with a total team score of 193 points).
96 IMO Problems, Theorems, and Methods: Algebra
The gold medal cutoff for this IMO was set at 40 points (with 3 contestants earning gold
medals), the silver medal cutoff was 30 points (with 20 contestants earning silver medals),
and the bronze medal cutoff was 24 points (with 21 contestants earning bronze medals).
In this IMO, only three contestants achieved a perfect score of 40 points, namely Tibor
Fiala from Hungary, Vladimir Drinfeld from the Soviet Union, and Simon Phillips Norton from
the United Kingdom.
Therefore, the solution to the recurrence relation (2) is fn = b1 r1n +b2 r2n .
Clearly, r1 > 1, and if b1 = 0, then limn→+∞ fn = ∞, which contradicts
the assumption. If b1 = 0, then b2 = 0, otherwise fn would be identically
zero. Furthermore, 0 < r2 < 1, so limn→−∞ fn = limn→−∞ b2 r2n = ∞,
which also contradicts the assumption.
In conclusion, |g(y)| ≤ 1 for all y ∈ R.
Note. It is evident that the recurrence relation fn+1 − 2gfn + fn−1 = 0 is
satisfied by the functions fn = cos nx and g = cos x. This is related to the
Chebyshev polynomials
Tn (cos θ) = cos nθ,
which will be presented in Chapter 5.
Furthermore, there are several similar problems:
Score Situation This particular problem saw the following distribution of scores among
contestants: 9 contestants scored 7 points, 2 contestants scored 6 points, 2 contestants
scored 5 points, no contestant scored 4 points, 1 contestant scored 3 points, 1 contestant
scored 2 points, 3 contestants scored 1 point, and 15 contestants scored 0 point. The average
score of this problem is 2.818, indicating that it had a certain level of difficulty.
98 IMO Problems, Theorems, and Methods: Algebra
Among the top five teams in the team scores, the scores of this problem are as follows:
the Soviet Union team scored 54 points (with a total team score of 270 points), the Hungary
team scored 40 points (with a total team score of 263 points), the German Democratic
Republic team scored 46 points (with a total team score of 239 points), the Romania team
scored 36 points (with a total team score of 208 points), and the United Kingdom team
scored 31 points (with a total team score of 179 points).
The gold medal cutoff for this IMO was set at 40 points (with 8 contestants earning gold
medals), the silver medal cutoff was 30 points (with 16 contestants earning silver medals),
and the bronze medal cutoff was 19 points (with 30 contestants earning bronze medals).
In this IMO, a total of eight contestants achieved a perfect score of 40 points.
Prove that there exists a real number k such that f (k) = k for all f in G.
Proof 1. First, we prove that
f (x) = x + b ∈ G ⇒ b = 0. (1)
Since f (x) and g(x) are arbitrary functions in G other than the identity
b
element, by setting k = 1−a , the number k is a common fixed point for all
functions in G.
Score Situation This particular problem saw the following distribution of scores among
contestants: 62 contestants scored 6 points, 3 contestants scored 5 points, 2 contestants
scored 4 points, 2 contestants scored 3 points, 2 contestants scored 2 points, 17 contestants
scored 1 point, and 37 contestants scored 0 point. The average score of this problem is 3.376,
indicating that it was relatively straightforward.
Among the top five teams in the team scores, the scores of this problem are as follows:
the Soviet Union team scored 48 points (with a total team score of 254 points), the Hungary
team scored 45 points (with a total team score of 215 points), the German Democratic
Republic team scored 40 points (with a total team score of 188 points), the Poland team
scored 42 points (with a total team score of 174 points), and the United Kingdom team
scored 24 points (with a total team score of 164 points).
The gold medal cutoff for this IMO was set at 35 points (with 5 contestants earning gold
medals), the silver medal cutoff was 27 points (with 15 contestants earning silver medals),
and the bronze medal cutoff was 17 points (with 48 contestants earning bronze medals).
In this IMO, only one contestant achieved a perfect score of 40 points, namely Sergei
Konyagin from the Soviet Union.
f (n + 1) > f (f (n))
• (Belarus Team Selection Test 2000, Problem 4). Does there exist
a function f : N → N such that
f (f (n − 1)) = f (n + 1) − f (n)
for all n ≥ 2?
Score Situation This particular problem saw the following distribution of scores among con-
testants: 10 contestants scored 8 points, 1 contestant scored 7 points, no contestant scored
6 points, no contestant scored 5 points, 3 contestants scored 4 points, 3 contestants scored
3 points, no contestant scored 2 points, 5 contestants scored 1 point, and 15 contestants
scored 0 point. The average score for this problem is 3.054, indicating that it was relatively
straightforward.
Among the top five teams in the team scores, the United States team achieved a total
score of 202 points, the Soviet Union team achieved a total score of 192 points, the Hungary
team achieved a total score of 190 points, the United Kingdom team achieved a total score
of 190 points, and the Netherlands team achieved a total score of 185 points.
The gold medal cutoff for this IMO was set at 34 points (with 13 contestants earning gold
medals), the silver medal cutoff was 24 points (with 29 contestants earning silver medals),
and the bronze medal cutoff was 17 points (with 35 contestants earning bronze medals).
In this IMO, a total of five contestants achieved a perfect score of 40 points.
implying
indicating r = l.
If k = 0, then from (1) and (2),
indicating r = l.
Hence, integers in the set {0, 1, 2, . . . , 1986} always pair up such that
with r = l. However, the set {0, 1, 2, . . . , 1986} has 1987 integers and 1987
is odd, so they cannot be paired up.
Therefore, such a function f does not exist.
Proof 2. By contradiction, suppose such a function f exists.
It is evident that f is injective: If f (n1 ) = f (n2 ), then f (f (n1 )) =
f (f (n2 )), implying n1 + 1987 = n2 + 1987, thus n1 = n2 . Therefore, f (0),
f (1), . . . , f (1986) are 1987 distinct integers. Let M = {f (0), f (1), . . . ,
f (1986)}.
Function Problems 103
Score Situation This particular problem saw the following distribution of scores among
contestants: 91 contestants scored 7 points, 7 contestants scored 6 points, 12 contestants
scored 5 points, 1 contestant scored 4 points, 10 contestants scored 3 points, 21 contestants
scored 2 points, 20 contestants scored 1 point, and 75 contestants scored 0 point. The
average score for this problem is 3.523, indicating that it was relatively straightforward.
Among the top five teams in the team scores, the scores of this problem are as follows:
the Romania team scored 42 points (with a total team score of 250 points), the Germany
team scored 42 points (with a total team score of 248 points), the Soviet Union team scored
42 points (with a total team score of 235 points), the German Democratic Republic team
scored 38 points (with a total team score of 231 points), and the United States team scored
36 points (with a total team score of 220 points).
The gold medal cutoff for this IMO was set at 42 points (with 22 contestants earning gold
medals), the silver medal cutoff was 32 points (with 42 contestants earning silver medals),
and the bronze medal cutoff was 18 points (with 56 contestants earning bronze medals).
In this IMO, a total of 22 contestants achieved a perfect score of 42 points.
Problem 2.7 (IMO 34-5, proposed by Germany). Does there exist a
function f : N+ → N+ such that:
(a) f (1) = 2;
(b) f (f (n)) = f (n) + n for all n ∈ N+ ;
(c) f (n) < f (n + 1) for all n ∈ N+ ?
Solution.
√ Such a function
√ exists. Let f (n) = αn + β for n ∈ N+ , where
α = 2 and β = 2 , and x denotes the greatest integer not exceed-
5+1 5−1
1
|f (n) − qn| <
q
• (Estonia Team Selection Test 2000, Problem 4). Find all functions
f : N → N such that
for all n ∈ N.
• (Estonia Team Selection Test 2000, Problem 6). We call an infi-
nite sequence of positive integers an F -sequence if every term of this
sequence (starting from the third term) equals the sum of the two pre-
ceding terms. Is it possible to decompose the set of all positive integers
into
(a) a finite;
(b) an infinite
number of F -sequences having no common members?
• (William Lowell Putnam Mathematical Competition 1988, A5).
Prove that there exists a unique function f from the set R+ of positive
real numbers to R+ such that
Score Situation This particular problem saw the following distribution of scores among
contestants: 91 contestants scored 7 points, 27 contestants scored 6 points, 19 contestants
scored 5 points, 32 contestants scored 4 points, 65 contestants scored 3 points, 44 contestants
scored 2 points, 92 contestants scored 1 point, and 43 contestants scored 0 point. The
average score for this problem is 3.383, indicating that it was relatively straightforward.
Among the top five teams in the team scores, the scores of this problem are as follows:
the China team scored 39 points (with a total team score of 215 points), the Germany team
scored 40 points (with a total team score of 189 points), the Bulgaria team scored 42 points
(with a total team score of 178 points), the Russia team scored 38 points (with a total
team score of 177 points), and the Chinese Taiwan team scored 33 points (with a total team
score of 162 points).
The gold medal cutoff for this IMO was set at 30 points (with 35 contestants earning gold
medals), the silver medal cutoff was 20 points (with 66 contestants earning silver medals),
and the bronze medal cutoff was 11 points (with 97 contestants earning bronze medals).
In this IMO, only two contestants achieved a perfect score of 42 points, namely Hong
Zhou from China and Hung-Wu Wu from Chinese Taiwan.
Function Problems 107
any x, y ∈ R, which implies f (0) ≤ −xf (x). For x < 0, it follows that
f (x) ≥ 0, leading to f (x) = 0 for x ≤ 0.
If there exists a z0 such that f (f (z0 )) < 0, then from (1) it is known
that f (z0 ) = f (f (z0 )) < 0. Therefore, there exists a t = f (z0 ) < 0 such
that f (t) = t. And if there exist t1 , t2 both satisfying f (t) = t, where
t1 < t2 < 0, then
t2 = f (t2 ) ≤ (t2 − t1 )f (t1 ) + f (f (t1 )) = (t2 − t1 )t1 + t1 ,
leading to (t2 − t1 )(1 − t1 ) ≤ 0, a contradiction. Thus, such t is unique, and
further, there exists an x1 ∈ R such that f (x1 ) = t.
(Otherwise, f (x) = t for all x ∈ R, implying t ≤ yt + t for any y ∈ R,
but it is not true for y > 0.)
For this x1 , we have f (f (x1 )) = f (x1 ), and from (1), f (f (x1 )) ≥ 0 > t.
Thus f (f (f (x1 ))) ≥ 0.
(Otherwise, from (1), 0 ≤ f (f (x1 )) = f (f (f (x1 ))) < 0, a contradiction.)
Combining with (2) implies
0 = f (f (x1 )) = f (f (f (x1 ))) = f (0).
Thus, it still holds that f (x + y) ≤ yf (x), leading to f (0) = 0 ≤ −xf (x).
Therefore, f (x) ≥ 0 for x < 0, which implies f (x) = 0 for x ≤ 0.
In conclusion, f (x) = 0 for any x ≤ 0.
Proof 2. Set y = t − x, and then the given inequality can be rewritten as
f (t) ≤ tf (x) − xf (x) + f (f (x)). (3)
In (3), by setting t = f (a), x = b and t = f (b), x = a, we have
f (f (a)) − f (f (b)) ≤ f (a)f (b) − bf (b),
f (f (b)) − f (f (a)) ≤ f (a)f (b) − af (a).
Adding the above two inequalities yields 2f (a)f (b) ≥ af (a) + bf (b).
Setting b = 2f (a) gives bf (b) ≥ af (a) + bf (b), so af (a) ≤ 0. Therefore,
f (a) ≤ 0 when a > 0, and f (a) ≥ 0 when a < 0.
Suppose there exists a real number x0 such that f (x0 ) > 0. Then
from
x0 f (x0 )−f (f (x0 ))
(3), it is known that f (t) < 0 for every t < min f (x0 ) ,0 , which
contradicts that f (t) ≥ 0 for t < 0.
Therefore, f (x) ≤ 0 for all real numbers x. Combined with f (x) ≥ 0
for x < 0, it follows that f (x) = 0 for any real number x < 0. Setting t = x
in (3), we have f (x) ≤ f (f (x)), and by taking x = −1, we obtain 0 ≤ f (0).
Since f (x) ≤ 0 for any real number x, it follows that f (0) = 0.
Function Problems 109
f (x + y) ≥ f (x) + yf (f (x))
Score Situation This particular problem saw the following distribution of scores among
contestants: 51 contestants scored 7 points, 5 contestants scored 6 points, 3 contestants
scored 5 points, 7 contestants scored 4 points, 13 contestants scored 3 points, 34 contestants
scored 2 points, 57 contestants scored 1 point, and 393 contestants scored 0 point. The
average score for this problem is 1.055, indicating that it was relatively challenging.
Among the top five teams in the team scores, the scores of this problem are as follows:
the China team scored 42 points (with a total team score of 189 points), the United States
team scored 35 points (with a total team score of 184 points), the Singapore team scored
23 points (with a total team score of 179 points), the Russia team scored 23 points (with a
total team score of 161 points), and the Thailand team scored 23 points (with a total team
score of 160 points).
The gold medal cutoff for this IMO was set at 28 points (with 54 contestants earning gold
medals), the silver medal cutoff was 22 points (with 90 contestants earning silver medals),
and the bronze medal cutoff was 16 points (with 137 contestants earning bronze medals).
In this IMO, only one contestant achieved a perfect score of 42 points, namely Lisa
Sauermann from Germany.
f (1) = 1,
g(1) = 2,
f (2) = 3,
f (f (2)) = f (3) = f (2) + 2 − 1 = 4,
f (f (3)) = f (4) = f (3) + 3 − 1 = 6,
f (f (4)) = f (6) = f (4) + 4 − 1 = 9,
f (f (6)) = f (9) = f (6) + 6 − 1 = 14,
f (f (9)) = f (14) = f (9) + 9 − 1 = 22,
f (f (14)) = f (22) = f (14) + 14 − 1 = 35,
f (f (22)) = f (35) = f (22) + 22 − 1 = 56,
f (f (35)) = f (56) = f (35) + 35 − 1 = 90,
g(35) = f (f (35)) + 1 = 91,
f (57) = 92,
f (f (57)) = f (92) = f (57) + 57 − 1 = 148,
f (f (92)) = f (148) = f (92) + 92 − 1 = 239,
f (f (148)) = f (239) = f (148) + 148 − 1 = 386,
g(148) = f (f (148)) + 1 = f (239) + 1 = 387,
f (240) = 388.
Score Situation This particular problem saw the following distribution of scores among
contestants: 8 contestants scored 8 points, 2 contestants scored 7 points, 1 contestant scored
6 points, 3 contestants scored 5 points, 7 contestants scored 4 points, 4 contestants scored
3 points, 6 contestants scored 2 points, 8 contestants scored 1 point, and 9 contestants
scored 0 point. The average score for this problem is 3.313, indicating that it was relatively
straightforward.
Among the top five teams in the team scores, the scores of this problem are as follows:
the Romania team scored 40 points (with a total team score of 237 points), the United States
team scored 47 points (with a total team score of 225 points), the United Kingdom team
scored 45 points (with a total team score of 201 points), the Vietnam team scored 45 points
(with a total team score of 200 points), and the Czechoslovakia team scored 24 points (with
a total team score of 195 points).
The gold medal cutoff for this IMO was set at 35 points (with 5 contestants earning gold
medals), the silver medal cutoff was 27 points (with 20 contestants earning silver medals),
and the bronze medal cutoff was 22 points (with 38 contestants earning bronze medals).
In this IMO, only one contestant achieved a perfect score of 40 points, namely Mark
Kleiman from the United States.
Among the top five teams in the team scores, the scores of this problem are as follows:
the United States team scored 55 points (with a total team score of 314 points), the Germany
team scored 56 points (with a total team score of 312 points), the United Kingdom team
scored 55 points (with a total team score of 301 points), the Austria team scored 55 points
(with a total team score of 290 points), and the Bulgaria team scored 55 points (with a total
team score of 287 points).
The gold medal cutoff for this IMO was set at 41 points (with 36 contestants earning gold
medals), the silver medal cutoff was 34 points (with 37 contestants earning silver medals),
and the bronze medal cutoff was 26 points (with 30 contestants earning bronze medals).
In this IMO, a total of 26 contestants achieved a perfect score of 42 points.
Problem 2.12 (IMO 23-1, proposed by the United Kingdom). A
function f (n) is defined for all positive integers n and takes on non-negative
integer values. Also, for all m and n,
f (m + n) − f (m) − f (n) = 0 or 1,
f (2) = 0, f (3) > 0, and f (9999) = 3333.
Determine f (1982).
Solution. From the given condition,
f (m + n) ≥ f (m) + f (n). (1)
Setting m = n = 1, we obtain f (2) ≥ 2f (1). However, since f (2) = 0
and f (1) is a non-negative integer, it follows that f (1) = 0. Setting m = 2
and n = 1, we get
f (3) − f (2) + f (1) = 0 or 1.
Given that f (3) > 0, it implies f (3) = 1. Now, we prove that f (3k) = k
for k ≤ 3333. From (1),
f (3k) ≥ f (3k − 3) + f (3)
≥ f (3k − 6) + 2f (3)
≥ ············
≥ kf (3)
= k.
If f (3k) > k, then f (3k) ≥ k + 1, leading to
f (9999) ≥ f (9999 − 3k) + f (3k)
≥ 3333 − k + k + 1
> 3333,
which contradicts the given conditions. Therefore, f (3k) = k.
Function Problems 117
Note.
n
There exists a function that meets the conditions, such as f (n) =
3 .
Furthermore, there are several similar problems:
• (Korea Winter Program Test 2018, Problem 1). Find all functions
f : R → R satisfying the following conditions:
Score Situation This particular problem saw the following distribution of scores among
contestants: 54 contestants scored 7 points, 4 contestants scored 6 points, no contestant
scored 5 points, 20 contestants scored 4 points, 11 contestants scored 3 points, 8 contestants
scored 2 points, 18 contestants scored 1 point, and 4 contestants scored 0 point. The average
score for this problem is 4.613, indicating that it was simple.
Among the top five teams in the team scores, the scores of this problem are as follows:
the Germany team scored 28 points (with a total team score of 145 points), the Soviet Union
team scored 28 points (with a total team score of 137 points), the German Democratic
Republic team scored 28 points (with a total team score of 136 points), the United States
team scored 25 points (with a total team score of 136 points), and the Vietnam team scored
25 points (with a total team score of 133 points).
The gold medal cutoff for this IMO was set at 37 points (with 10 contestants earn-
ing gold medals), the silver medal cutoff was 30 points (with 20 contestants earning silver
medals), and the bronze medal cutoff was 21 points (with 31 contestants earning bronze
medals).
In this IMO, only three contestants achieved a perfect score of 42 points, namely Bruno
Haible from Germany, Grigori Perelman from the Soviet Union, and Le Tu QuocThang from
Vietnam.
118 IMO Problems, Theorems, and Methods: Algebra
For n ∈ {1, 2, 3}, it is evident that (1) holds. Since f (2n) = f (n), it
suffices to consider the case where n is odd. Assume (1) holds for n < k.
For n = k, we consider f (k).
(i) Suppose k = 4m + 1. Let 4m + 1 = (at at−1 · · · a0 )2 , where a0 = 1
and a1 = 0. Then
leading to
confirming (1).
(ii) Let k = 4m + 3 with 4m + 3 = (at at−1 · · · a0 )2 , where a0 = a1 = 1.
Then
Score Situation This particular problem saw the following distribution of scores among
contestants: 31 contestants scored 7 points, 9 contestants scored 6 points, 4 contestants
scored 5 points, 3 contestants scored 4 points, 1 contestant scored 3 points, 13 contestants
scored 2 points, 132 contestants scored 1 point, and 75 contestants scored 0 point. The
average score for this problem is 1.731, indicating that it was relatively challenging.
Among the top five teams in the team scores, the scores of this problem are as follows:
the Soviet Union team scored 36 points (with a total team score of 217 points), the Romania
team scored 35 points (with a total team score of 201 points), the China team scored 17 points
(with a total team score of 201 points), the Germany team scored 25 points (with a total team
score of 174 points), and the Vietnam team scored 12 points (with a total team score of
166 points).
The gold medal cutoff for this IMO was set at 32 points (with 17 contestants earning gold
medals), the silver medal cutoff was 23 points (with 48 contestants earning silver medals),
and the bronze medal cutoff was 14 points (with 65 contestants earning bronze medals).
In this IMO, a total of five contestants achieved a perfect score of 42 points.
Solution. First, we prove that 1 is in the range of f . For any x0 > 1, set
y0 = f (x1 0 ) and substitute it into (a) to obtain
f (x) = y0 f (x),
For any positive real number x, let a = xf (x) and substitute y = x into
(a) to yield f (a) = a. If a > 1, then
f (a2 ) = f (af (a)) = af (a) = a2 .
n n
By mathematical induction, as n → +∞, f (a2 ) = a2 → +∞, contra-
dicting (b).
If a < 1, then
1 1
1 = f (1) = f f (a) = af ,
a a
implying f a1 = a1 . However, since a1 > 1, following the case for a > 1 by
setting a = a1 , we also come to a contradiction.
Therefore, a = 1, implying f (x) = x1 . It is easy to verify that f (x) = x1
meets the conditions.
Note. Another approach to deriving f (1) = 1 involves substituting values
into the condition. Setting x = y = 1 gives f (f (1)) = f (1), and setting x =
1 and y = f (1) yields f (f (f (1))) = f (1)f (1). Therefore, f (1) = f (1)f (1).
Since f (1) > 0, we have f (1) = 1.
Furthermore, there are several similar problems:
Score Situation This particular problem saw the following distribution of scores among
contestants: 12 contestants scored 7 points, 1 contestant scored 6 points, 1 contestant
scored 5 points, no contestant scored 4 points, 9 contestants scored 3 points, 4 contestants
scored 2 points, 13 contestants scored 1 point, and no contestant scored 0 point. The average
score of this problem is 3.575, indicating that it was relatively straightforward.
Among the top five teams in the team scores, the Germany team achieved a total score
of 212 points, the United States team achieved a total score of 171 points, the Hungary
team achieved a total score of 170 points, the Soviet Union team achieved a total score of
169 points, and the Romania team achieved a total score of 161 points.
The gold medal cutoff for this IMO was set at 38 points (with 9 contestants earning gold
medals), the silver medal cutoff was 26 points (with 27 contestants earning silver medals),
and the bronze medal cutoff was 15 points (with 57 contestants earning bronze medals).
In this IMO, a total of four contestants achieved a perfect score of 42 points.
122 IMO Problems, Theorems, and Methods: Algebra
Note. This problem is the same as the 20th problem in 2004 Germany
Team Selection Test. Furthermore, there is a similar problem:
Score Situation This particular problem saw the following distribution of scores among
contestants: 57 contestants scored 7 points, 41 contestants scored 6 points, 8 contestants
scored 5 points, 13 contestants scored 4 points, 30 contestants scored 3 points, 14 contestants
scored 2 points, 17 contestants scored 1 point, and 30 contestants scored 0 point. The
average score for this problem is 4.152, indicating that it was simple.
Function Problems 123
Among the top five teams in the team scores, the scores of this problem are as follows:
the Soviet Union team scored 41 points (with a total team score of 203 points), the United
States team scored 39 points (with a total team score of 203 points), the Germany team
scored 40 points (with a total team score of 196 points), the China team scored 30 points
(with a total team score of 177 points), and the German Democratic Republic team scored
42 points (with a total team score of 172 points).
The gold medal cutoff for this IMO was set at 34 points (with 18 contestants earning gold
medals), the silver medal cutoff was 26 points (with 41 contestants earning silver medals),
and the bronze medal cutoff was 17 points (with 48 contestants earning bronze medals).
In this IMO, only three contestants achieved a perfect score of 42 points, namely Vladimir
Roganov and Stanislav Smirnov from the Soviet Union, and Géza Kós from Hungary.
f (x)
f (xf (y)) =
y
f (x)
f (xf (y)) = , where x, y ∈ Q+ , (1)
y
f (x) f (x)
= f (xf (y1 )) = f (xf (y2 )) = ,
y1 y2
f (1) 1
f (f (y)) = = . (2)
y y
f (x)
f (xf (f (t))) = ,
f (t)
124 IMO Problems, Theorems, and Methods: Algebra
x f (x)
and from (2), f (xf (f (t))) = f t = f (t) . Thus
It is easy to verify that the function f defined by (4) and (5) satisfies
(2), and therefore such a defined f satisfies (1).
Score Situation This particular problem saw the following distribution of scores among
contestants: 69 contestants scored 7 points, 3 contestants scored 6 points, 3 contestants
scored 5 points, 10 contestants scored 4 points, 46 contestants scored 3 points, 70 contestants
scored 2 points, 76 contestants scored 1 point, and 31 contestants scored 0 point. The
average score for this problem is 2.955, indicating that it had a certain level of difficulty.
Among the top five teams in the team scores, the scores of this problem are as follows:
the China team scored 42 points (with a total team score of 230 points), the Soviet Union
team scored 35 points (with a total team score of 193 points), the United States team scored
37 points (with a total team score of 174 points), the Romania team scored 30 points (with
a total team score of 171 points), and the France team scored 38 points (with a total team
score of 168 points).
The gold medal cutoff for this IMO was set at 34 points (with 23 contestants earning gold
medals), the silver medal cutoff was 23 points (with 56 contestants earning silver medals),
and the bronze medal cutoff was 16 points (with 76 contestants earning bronze medals).
In this IMO, a total of four contestants achieved a perfect score of 42 points.
Function Problems 125
Problem 2.17 (IMO 33-2, proposed by India). Let R denote the set
of all real numbers. Find all functions f : R → R such that
f (x2 + f (y)) = y + (f (x))2 for all x, y ∈ R.
Solution 1. Suppose
f (x2 + f (y)) = y + (f (x))2 . (1)
First, we prove that f (0) = 0. Set x = 0 and t = (f (0))2 . Then
f (f (y)) = y + t. (2)
Replacing y with x2 + f (f (y)) in (2), we get
f (f (x2 + f (f (y)))) = x2 + f (f (y)) + t. (3)
From (1),
f (f (x2 + f (f (y)))) = f ((f (x))2 + f (y))
= y + (f (f (x)))2 . (4)
Combining (2), (3), and (4), we see that
x2 + y + 2t = y + (x + t)2 and 2t = t2 + 2tx
for any real number x. It follows that t = 0, i.e., f (0) = 0. Thus, (2)
becomes
f (f (y)) = y. (5)
For x ≥ 0, from (1) and (5),
√
f (x + y) = f (x + f (f (y))) = f (y) + (f ( x))2 ≥ f (y),
indicating that f (x) is an increasing function on R, i.e., f (x) ≥ f (y) for
x ≥ y.
If there exists an x0 such that f (x0 ) > x0 , then f (f (x0 )) ≥ f (x0 ) > x0 ,
which is a contradiction. Similarly, if there exists an x0 such that f (x0 ) <
x0 , then f (f (x0 )) ≤ f (x0 ) < x0 , which is also a contradiction.
Therefore, f (x) = x. Clearly, f (x) = x satisfies the given conditions.
Solution 2. From the conditions and the fact that y can take any real
number, f is surjective. Moreover, if f (y1 ) = f (y2 ), then
y1 + (f (x))2 = f (x2 + f (y1 )) = f (x2 + f (y2 )) = y2 + (f (x))2 ,
implying y1 = y2 , and thus f is injective. Therefore, f is bijective.
126 IMO Problems, Theorems, and Methods: Algebra
(a) except for n = 1 and l = 0, if and only if both m, n are odd, then there
are two solutions f (x) = x and f (x) = −x;
(b) except for n = 1 and l = 0, if m, n are not both odd, then there is only
one solution f (x) = x;
(c) for n = 1 and l = 0, the solutions are not yet fully determined.
Here, l ∈ {0, 1, 2, 3, . . .}, m, n ∈ {1, 2, 3, . . .}, and f (m) (x) represents the
mth iteration of f .
Furthermore, there are several similar problems:
• (Japan Team Selection Test 2023, Problem 5). Let R be the set
of real numbers and F be the set of all functions f : R → R such that
Find all rational numbers q such that for every function f ∈ F , there
exists some z ∈ R satisfying f (z) = qz.
• (Chinese Team Selection Test 2021, Problem 17). Determine all
f : R → R such that
f (x + f (y)) = x + f (f (y))
for all real numbers x and y, with the additional constraint f (2004) =
2005.
• (Japan Mathematical Olympiad 2004, Final Round,
Problem 2). Find all functions f : R → R such that
f (xf (x) + f (y)) = y + (f (x))2 for all x, y ∈ R.
Score Situation This particular problem saw the following distribution of scores among
contestants: 67 contestants scored 7 points, 3 contestants scored 6 points, 18 contestants
scored 5 points, 34 contestants scored 4 points, 43 contestants scored 3 points, 41 contestants
scored 2 points, 113 contestants scored 1 point, and 31 contestants scored 0 point. The
average score for this problem is 2.963, indicating that it had a certain level of difficulty.
Among the top five teams in the team scores, the scores of this problem are as follows:
the China team scored 42 points (with a total team score of 240 points), the United States
team scored 32 points (with a total team score of 181 points), the Romania team scored
42 points (with a total team score of 177 points), the Commonwealth of Independent States
team scored 30 points (with a total team score of 176 points), and the United Kingdom team
scored 24 points (with a total team score of 168 points).
128 IMO Problems, Theorems, and Methods: Algebra
The gold medal cutoff for this IMO was set at 32 points (with 26 contestants earning gold
medals), the silver medal cutoff was 24 points (with 55 contestants earning silver medals),
and the bronze medal cutoff was 14 points (with 74 contestants earning bronze medals).
In this IMO, a total of four contestants achieved a perfect score of 42 points.
u2 + 2u = u,
Score Situation This particular problem saw the following distribution of scores among
contestants: 84 contestants scored 7 points, 38 contestants scored 6 points, 23 contestants
scored 5 points, 22 contestants scored 4 points, 29 contestants scored 3 points, 40 contestants
scored 2 points, 54 contestants scored 1 point, and 95 contestants scored 0 point. The
average score for this problem is 3.221, indicating that it was relatively straightforward.
Among the top five teams in the team scores, the scores of this problem are as follows:
the United States team scored 42 points (with a total team score of 252 points), the China
team scored 40 points (with a total team score of 229 points), the Russia team scored
Function Problems 129
40 points (with a total team score of 224 points), the Bulgaria team scored 41 points (with
a total team score of 223 points), and the Hungary team scored 31 points (with a total team
score of 221 points).
The gold medal cutoff for this IMO was set at 40 points (with 30 contestants earning gold
medals), the silver medal cutoff was 30 points (with 64 contestants earning silver medals),
and the bronze medal cutoff was 19 points (with 98 contestants earning bronze medals).
In this IMO, a total of 22 contestants achieved a perfect score of 42 points.
Score Situation This particular problem saw the following distribution of scores among
contestants: 11 contestants scored 7 points, 5 contestants scored 6 points, 7 contestants
scored 5 points, 8 contestants scored 4 points, 21 contestants scored 3 points, 28 contestants
scored 2 points, 225 contestants scored 1 point, and 145 contestants scored 0 point. The
average score for this problem is 1.151, indicating that it was relatively challenging.
Among the top five teams in the team scores, the scores of this problem are as follows:
the China team scored 15 points (with a total team score of 182 points), the Russia team
scored 11 points (with a total team score of 182 points), the Vietnam team scored 18 points
(with a total team score of 177 points), the Romania team scored 23 points (with a total
team score of 173 points), and the Bulgaria team scored 29 points (with a total team score
of 170 points).
The gold medal cutoff for this IMO was set at 28 points (with 38 contestants earning gold
medals), the silver medal cutoff was 19 points (with 70 contestants earning silver medals),
and the bronze medal cutoff was 12 points (with 118 contestants earning bronze medals).
In this IMO, no contestant achieved a perfect score of 42 points.
for all x, y, z, t ∈ R.
Solution 1. Setting x = y = z = 0 in (1) gives
f (x2 ) = (f (x))2 ≥ 0,
which implies
and using (4) and mathematical induction, we see that f (n) = n2 for non-
negative integers n. From f (xy) = f (x)f (y), it follows that f (a) = a2 for
all positive rational numbers a.
Next, we prove that f (x) = x2 for x > 0. Otherwise, there exists x0 ∈ R
such that f (x0 ) = x20 .
If f (x0 ) < x20 , by choosing a rational number a such that f (x0 ) <
a < x0 , then f (x0 ) ≥ f (a) = a2 > f (x0 ), which is a contradiction. If
f (x0 ) > x20 , then there is a similar contradiction. Therefore, f (x) = x2 for
x > 0. Since f (x) is even, f (x) = x2 for x ∈ R.
Upon verification, the solutions satisfying the conditions are f (x) = 0,
f (x) = 12 , and f (x) = x2 .
Solution 2. Setting y = t = 0 and z = x in (1), we obtain 4f (0)f (x) =
2f (0).
• (Bolivia Team Selection Test 2021, Problem 1). Find all functions
f : Q → Q such that
f (x + y) + f (x − y) = 2f (x) + 2f (y)
for all x, y ∈ Q.
• (Korean Mathematical Olympiad 2015, 2nd Round, Prob-
lem 5). Find all functions f : R → R such that for all real numbers
x, y, z,
for all rational numbers x < y < z < t that form an arithmetic progres-
sion.
Function Problems 133
Score Situation This particular problem saw the following distribution of scores among
contestants: 66 contestants scored 7 points, 25 contestants scored 6 points, 10 contestants
scored 5 points, no contestant scored 4 points, 21 contestants scored 3 points, 101 contestants
scored 2 points, 159 contestants scored 1 point, and 97 contestants scored 0 point. The
average score for this problem is 2.267, indicating that it had a certain level of difficulty.
Among the top five teams in the team scores, the scores of this problem are as follows:
the China team scored 42 points (with a total team score of 212 points), the Russia team
scored 42 points (with a total team score of 204 points), the United States team scored
34 points (with a total team score of 171 points), the Bulgaria team scored 36 points (with
a total team score of 167 points), and the Vietnam team scored 40 points (with a total team
score of 166 points).
The gold medal cutoff for this IMO was set at 29 points (with 39 contestants earning gold
medals), the silver medal cutoff was 23 points (with 73 contestants earning silver medals),
and the bronze medal cutoff was 14 points (with 120 contestants earning bronze medals).
In this IMO, only three contestants achieved a perfect score of 42 points, namely Yunhao
Fu and Botong Wang from China, and Andrei Khaliavine from Russia.
Problem 2.21 (IMO 49-4, proposed by South Korea). Find all func-
tions f : (0, +∞) → (0, +∞) such that
(f (w))2 + (f (x))2 w2 + x2
=
f (y 2 ) + f (z 2 ) y2 + z 2
for all positive real numbers w, x, y, z satisfying wx = yz.
Solution. Setting w = x = y = z = 1 gives (f (1))2 = f (1). Hence
f (1) = 1. √
For any t > 0, setting w = t, x = 1, and y = z = t yields
(f (t))2 + 1 t2 + 1
= ,
2f (t) 2t
and after simplifying, we get (tf (t) − 1)(f (t) − t) = 0. Therefore, for each
t > 0,
1
f (t) = t or f (t) = . (1)
t
If there exist b, c ∈ (0, +∞) such that f (b) = b and f (c) = c , then from
1
(1) we know b and c are both not equal to 1, and f (b) = 1b and f (c) = c.
134 IMO Problems, Theorems, and Methods: Algebra
√ 1
+c2 b2 +c2
b2
Setting w = b, x = c, and y = z = bc, we get 2f (bc) = 2bc , i.e.,
c+b2 c3
f (bc) = b(b2 +c2 ) .
1
Since f (bc) = bc or f (bc) = bc , if f (bc) = bc, then
c + b 2 c3
bc = ,
b(b2 + c2 )
1
yielding b4 c = c and b = 1, which is a contradiction. If f (bc) = bc , then
1 c + b 2 c3
= ,
bc b(b2 + c2 )
yielding b2 c4 = b2 and c = 1, which is also a contradiction.
Thus, either f (x) = x for all x ∈ (0, +∞) or f (x) = x1 for all x ∈
(0, +∞). Upon verification, both f (x) = x and f (x) = x1 satisfy the given
conditions.
Score Situation This particular problem saw the following distribution of scores among
contestants: 227 contestants scored 7 points, 27 contestants scored 6 points, no contestant
scored 5 points, 128 contestants scored 4 points, 4 contestants scored 3 points, no contestant
scored 2 points, 80 contestants scored 1 point, and 69 contestants scored 0 point. The
average score for this problem is 4.402, indicating that it was simple.
Among the top five teams in the team scores, the scores of this problem are as follows:
the China team scored 42 points (with a total team score of 217 points), the Russia team
scored 42 points (with a total team score of 199 points), the United States team scored
41 points (with a total team score of 190 points), the South Korea team scored 42 points
(with a total team score of 188 points), and the Iran team scored 42 points (with a total
team score of 181 points).
The gold medal cutoff for this IMO was set at 31 points (with 47 contestants earning gold
medals), the silver medal cutoff was 22 points (with 100 contestants earning silver medals),
and the bronze medal cutoff was 15 points (with 120 contestants earning bronze medals).
In this IMO, only three contestants achieved a perfect score of 42 points, namely
Xiaosheng Mu and Dongyi Wei from China, and Alex Zhai from the United States.
Setting a = 1 in (2) and (3), we find that f (b) = f (b + f (1) − 1) for any
b ∈ N+ . If f (1) = 1, then the above equation implies that f is periodic.
Considering the domain of f , we are sure that f is bounded. Let M be a
positive integer such that M ≥ f (n) for all n ∈ N+ (i.e., M is an upper
bound of f ). Setting a = 2M in (1) leads to a contradiction. Therefore
f (1) = 1.
Setting b = 1 in (1) and (2), we get f (f (n)) = n for n ∈ N+ . If there
exists t ∈ N+ such that f (t) < t, then clearly t ≥ 2 and f (t) ≤ t − 1.
Setting a = f (t) in (2) yields
f (b + t − 1) = f (b + f (a) − 1)
≤ f (b) + a − 1
≤ f (b) + t − 2.
1 ≤ n0 ≤ t − 1, n0 ≡ n(mod(t − 1)).
Then
t−2 M (t − 2)n
f (n) ≤ f (n0 ) + (n − n0 ) ≤ + < n.
t−1 t−1 t−1
Therefore, f (n) < n for integers n > M . Choose n1 ∈ N+ such
that n1 > M and n1 is not equal to any of f (1), f (2), . . . , f (M ). From
f (f (n1 )) = n1 , we know f (n1 ) > M , and from the above conclusion,
n1 > f (n1 ) > f (f (n1 )) = n1 , a contradiction.
This implies that f (t) ≥ t for any t ∈ N+ and further t = f (f (t)) ≥
f (t) ≥ t, implying the equality throughout, i.e., f (n) = n for n ∈ N+ .
Upon verification, f (n) = n for n ∈ N+ satisfies the conditions. There-
fore, the solution is f (n) = n for n ∈ N+ .
136 IMO Problems, Theorems, and Methods: Algebra
f (b + t) − f (b) = ±1.
• (Chinese Team Selection Test 2016, Problem 18). Find all func-
tions f : R+ → R+ satisfying the following condition: for any three dis-
tinct real numbers a, b, c, a triangle can be formed with side lengths a, b, c
if and only if a triangle can be formed with side lengths f (a), f (b), f (c).
• (British Mathematical Olympiad 2014, 1st Round, Problem 6).
Determine all functions f (n) from the positive integers to the positive
integers which satisfy the following condition: whenever a, b, and c are
positive integers such that a1 + 1b = 1c ,
1 1 1
+ = .
f (a) f (b) f (c)
Score Situation This particular problem saw the following distribution of scores among
contestants: 153 contestants scored 7 points, 7 contestants scored 6 points, 4 contestants
scored 5 points, 6 contestants scored 4 points, 33 contestants scored 3 points, 50 contestants
scored 2 points, 42 contestants scored 1 point, and 270 contestants scored 0 point. The
average score for this problem is 2.474, indicating that it had a certain level of difficulty.
Among the top five teams in the team scores, the scores of this problem are as follows:
the China team scored 42 points (with a total team score of 221 points), the Japan team
scored 42 points (with a total team score of 212 points), the Russia team scored 42 points
Function Problems 137
(with a total team score of 203 points), the South Korea team scored 42 points (with a total
team score of 188 points), and the North Korea team scored 36 points (with a total team
score of 183 points).
The gold medal cutoff for this IMO was set at 32 points (with 49 contestants earning gold
medals), the silver medal cutoff was 24 points (with 98 contestants earning silver medals),
and the bronze medal cutoff was 14 points (with 135 contestants earning bronze medals).
In this IMO, only two contestants achieved a perfect score of 42 points, namely Dongyi
Wei from China and Makoto Soejima from Japan.
Score Situation This particular problem saw the following distribution of scores among
contestants: 294 contestants scored 7 points, 54 contestants scored 6 points, 35 contestants
scored 5 points, 34 contestants scored 4 points, 16 contestants scored 3 points, 27 contestants
scored 2 points, 17 contestants scored 1 point, and 39 contestants scored 0 point. The
average score for this problem is 5.450, indicating that it was simple.
Among the top five teams in the team scores, the scores of this problem are as follows:
the China team scored 41 points (with a total team score of 197 points), the Russia team
scored 41 points (with a total team score of 169 points), the United States team scored
40 points (with a total team score of 168 points), the South Korea team scored 42 points
(with a total team score of 156 points), the Kazakhstan team scored 42 points (with a total
team score of 148 points), and the Thailand team scored 42 points (with a total team score
of 148 points).
The gold medal cutoff for this IMO was set at 27 points (with 47 contestants earning gold
medals), the silver medal cutoff was 21 points (with 103 contestants earning silver medals),
and the bronze medal cutoff was 15 points (with 115 contestants earning bronze medals).
In this IMO, only one contestant achieved a perfect score of 42 points, namely Zipei Nie
from China.
Problem 2.24 (IMO 53-4, proposed by South Africa). Find all func-
tions f : Z → Z such that, for all integers a, b, c that satisfy a + b + c = 0,
the following equality holds:
(f (a + r) − f (a))2 = 0,
for all a ∈ Z. This function clearly satisfies the given conditions. Next,
assume f (1) = k = 0.
From (1), we have f (2) = 0 or f (2) = 4k.
If f (2) = 0, then f is a periodic function with period 2. Hence for all
n ∈ Z,
f (2n) = 0 and f (2n + 1) = k.
If f (2) = 4k = 0, then from (1), we know f (4) = 0 or f (4) = 16k.
Suppose f (4) = 0, the function f is periodic with period 4, and
f (3) = f (−1) = f (1) = k.
Hence f (4n) = 0, f (4n + 1) = f (4n + 3) = k, and f (4n + 2) = 4k for all
n ∈ Z.
Suppose f (4) = 16k = 0, setting a = 1, b = 2, and c = −3 gives
(f (3))2 − 10kf (3) + 9k 2 = 0,
implying f (3) ∈ {k, 9k}. Setting a = 1, b = 3, and c = −4 yields
(f (3))2 − 34kf (3) + 225k 2 = 0,
implying f (3) ∈ {9k, 25k}. Thus f (3) = 9k.
Next, we use mathematical induction to prove f (x) = kx2 for x ∈ Z.
For x ∈ {0, 1, 2, 3, 4}, the proposition already holds. Assume the proposi-
tion holds for x ∈ {0, 1, . . . , n}(n ≥ 4).
Setting a = n, b = 1, and c = −n − 1, we get
f (n + 1) ∈ {k(n + 1)2 , k(n − 1)2 }.
Setting a = n − 1, b = 2, and c = −n − 1, we obtain
f (n + 1) ∈ {k(n + 1)2 , k(n − 3)2 }.
Since k(n−1)2 = k(n−3)2 for n = 2, it follows that f (n+1) = k(n+1)2 ,
proving f (x) = kx2 for non-negative integers x. As f is even, f (x) = kx2
for x ∈ Z.
In conclusion, the possible functions are
f1 (x) = 0,
f2 (x) = kx2 ,
0, x ≡ 0 (mod2),
f3 (x) =
k, x ≡ 1 (mod2),
⎧
⎨0, x ≡ 0 (mod4),
f4 (x) = k, x ≡ 1 (mod2),
⎩
4k, x ≡ 2 (mod4),
where k is any non-zero integer.
140 IMO Problems, Theorems, and Methods: Algebra
satisfying the conditions. If a, b, c include one even and two odd integers,
then the left side equals 2k 2 as does the right side, meeting the conditions.
For f4 , by considering the symmetry and a + b + c = 0, it suffices to
consider the cases where (f (a), f (b), f (c)) are (0, k, k), (4k, k, k), (0, 0, 0),
and (0, 4k, 4k), all of which evidently satisfy the given conditions.
Score Situation This particular problem saw the following distribution of scores among
contestants: 143 contestants scored 7 points, 44 contestants scored 6 points, 26 contestants
scored 5 points, 47 contestants scored 4 points, 74 contestants scored 3 points, 95 contestants
scored 2 points, 65 contestants scored 1 point, and 53 contestants scored 0 point. The
average score for this problem is 3.766, indicating that it was relatively straightforward.
Among the top five teams in the team scores, the scores of this problem are as follows:
the South Korea team scored 39 points (with a total team score of 209 points), the China
team scored 31 points (with a total team score of 195 points), the United States team scored
38 points (with a total team score of 194 points), the Russia team scored 41 points (with a
total team score of 177 points), the Canada team scored 39 points (with a total team score of
159 points), and the Thailand team scored 39 points (with a total team score of 159 points).
The gold medal cutoff for this IMO was set at 28 points (with 51 contestants earning gold
medals), the silver medal cutoff was 21 points (with 88 contestants earning silver medals),
and the bronze medal cutoff was 14 points (with 137 contestants earning bronze medals).
In this IMO, only one contestant achieved a perfect score of 42 points, namely Jeck Lim
from Singapore.
Case 1: f (0) = 0.
Considering P (0, y), we have
Case 2: f (0) = 0.
Considering P (x + 1, 0), we have
f (x + f (x + 1) + 1) = x + f (x + 1) + 1. (2)
f (x + f (x + 1) + 2) = x + f (x + 1) + 2.
f (x + f (x − 1)) = x + f (x − 1).
−f (1 + f (y + 1)) + f (y) = −1 − f (y + 1) + y,
and adding this equality to (4), we know f (y) = y for any y ∈ R. It is easy
to verify that f (x) = x is a function that satisfies the conditions.
In conclusion, there are two functions that satisfy the conditions:
f (x) = x and f (x) = 2 − x.
Note. There is a similar problem:
• (Japan Team Selection Test 2022, Problem 9). Find all functions
f : R → R such that f (−1) = −1 and
Score Situation This particular problem saw the following distribution of scores among
contestants: 30 contestants scored 7 points, 3 contestants scored 6 points, 4 contestants
scored 5 points, 8 contestants scored 4 points, 90 contestants scored 3 points, 34 contestants
scored 2 points, 255 contestants scored 1 point, and 153 contestants scored 0 point. The
average score for this problem is 1.513, indicating that it was relatively challenging.
Among the top five teams in the team scores, the scores of this problem are as follows:
the United States team scored 25 points (with a total team score of 185 points), the China
team scored 22 points (with a total team score of 181 points), the South Korea team scored
25 points (with a total team score of 161 points), the North Korea team scored 22 points
(with a total team score of 156 points), and the Vietnam team scored 28 points (with a total
team score of 151 points).
The gold medal cutoff for this IMO was set at 26 points (with 39 contestants earning gold
medals), the silver medal cutoff was 19 points (with 100 contestants earning silver medals),
and the bronze medal cutoff was 14 points (with 143 contestants earning bronze medals).
In this IMO, only one contestant achieved a perfect score of 42 points, namely Zhuo Qun
Alex Song from Canada.
P (0, 0),
f (f (0)2 ) = 0. (1)
f (0) + f (x + 1) = f (x),
f (x + n) = f (x) + n. (3)
Conclusion 2: f is injective.
By contradiction, suppose there exist a = b such that f (a) = f (b). From
(3), for any integer N , we have f (a + N + 1) = f (b + N ) + 1. Choosing
an integer N > −b, we see that there exist x0 , y0 ∈ R satisfying x0 + y0 =
a + N + 1 and x0 y0 = b + N . Since a = b, we have (x0 − 1)(y0 − 1) = 0.
From P (x0 , y0 ),
f (f (x0 )f (y0 )) + f (a + N + 1) = f (b + N ),
For (i), note that g(x + n) = g(x) + n is the same as (3) in Solution 1.
Thus
g(x) = n ⇔ g(x − n) = 0 ⇔ x − n = 0.
n
For (ii), when x = 0, it is obvious. For x = 0, substitute y = x into (6),
yielding
n n n
g g(x)g +g x+ =g n+x+
x x x
n n
⇔ g g(x)g = n ⇔ g(x)g = n,
x x
Function Problems 145
n
1
i.e., g(x) = g( n
for x = 0. Taking n = 1, we get g x = 1
g(x) , and repla-
x)
cing x with nx in the above equation, we obtain g(nx) = g n1 = ng(x).
(x)
Thus proving the conclusion.
Conclusion 4: g is an additive function, i.e., for any a, b ∈ R,
Score Situation This particular problem saw the following distribution of scores among
contestants: 61 contestants scored 7 points, 8 contestants scored 6 points, 10 contestants
scored 5 points, 79 contestants scored 4 points, 138 contestants scored 3 points, 26 contes-
tants scored 2 points, 110 contestants scored 1 point, and 183 contestants scored 0 point.
The average score for this problem is 2.304, indicating that it had a certain level of difficulty.
Among the top five teams in the team scores, the scores of this problem are as follows:
the South Korea team scored 39 points (with a total team score of 170 points), the China
team scored 25 points (with a total team score of 159 points), the Vietnam team scored
36 points (with a total team score of 155 points), the United States team scored 29 points
(with a total team score of 148 points), and the Iran team scored 32 points (with a total
team score of 142 points).
The gold medal cutoff for this IMO was set at 25 points (with 48 contestants earning gold
medals), the silver medal cutoff was 19 points (with 90 contestants earning silver medals),
and the bronze medal cutoff was 16 points (with 153 contestants earning bronze medals).
In this IMO, no contestant achieved a perfect score of 42 points.
Problem 2.27 (IMO 60-1, proposed by South Africa). Let Z be the
set of integers. Determine all functions f : Z → Z such that, for all integers
a and b,
f (2a) + 2f (b) = f (f (a + b)).
Solution. Let’s denote the equation given in the problem as P (a, b). From
P (0, x),
f (0) + 2f (x) = f (f (x)). (1)
From P (x, 0),
f (2x) + 2f (0) = f (f (x)). (2)
Comparing (1) and (2), we get
f (2x) = 2f (x) − f (0). (3)
Substituting (1) and (3) into P (a, b), we have
2f (a) − f (0) + 2f (b) = f (0) + 2f (a + b). (4)
Let g(x) = f (x) − f (0). Then g(0) = 0, and (4) can be rewritten as
g(a + b) = g(a) + g(b). (5)
By Cauchy’s functional equation, g(n) = g(1)·n for any integer n. Thus
f can be expressed as f (x) = kx + c, where k, c ∈ Z. Substituting this back
into P (a, b), we see that for all integers a and b,
2k(a + b) + 3c = k 2 (a + b) + (k + 1)c.
Function Problems 147
• (Japan Team Selection Test 2021, Problem 12). Find all functions
f : Z → Z satisfying
2
+b2 )
f (a (a + b) = af (a) + bf (b) for all a, b ∈ Z.
Score Situation This particular problem saw the following distribution of scores among
contestants: 382 contestants scored 7 points, 52 contestants scored 6 points, 5 contestants
scored 5 points, 14 contestants scored 4 points, 24 contestants scored 3 points, 6 contestants
scored 2 points, 65 contestants scored 1 point, and 73 contestants scored 0 point. The
average score for this problem is 5.179, indicating that it was simple.
Among the top five teams in the team scores, the scores of this problem are as follows:
the China team scored 40 points (with a total team score of 227 points), the United States
team scored 42 points (with a total team score of 227 points), the South Korea team scored
42 points (with a total team score of 226 points), the North Korea team scored 41 points
(with a total team score of 187 points), and the Thailand team scored 42 points (with a total
team score of 185 points).
The gold medal cutoff for this IMO was set at 31 points (with 52 contestants earning gold
medals), the silver medal cutoff was 24 points (with 94 contestants earning silver medals),
and the bronze medal cutoff was 17 points (with 156 contestants earning bronze medals).
In this IMO, a total of six contestants achieved a perfect score of 42 points.
xf (y) + yf (x) ≤ 2.
1
Solution 1. The unique function that satisfies the conditions is f (x) = x
for all x ∈ R+ .
148 IMO Problems, Theorems, and Methods: Algebra
First, we verify that the function f (x) = x1 meets the conditions. For
any x, y ∈ R+ , using the AM-GM inequality, we know
x y
xf (y) + yf (x) = + ≥ 2,
y x
with equality if and only if x = y. This indicates that the only y ∈ R+
satisfying xf (y) + yf (x) ≤ 2 is y = x.
Next, we prove that if f satisfies the conditions, then f (x) = x1 .
If a pair of positive real numbers (x, y) satisfies xf (y) + yf (x) ≤ 2, then
we call it a “good pair.” If (x, y) is a good pair, then (y, x) is also a good
pair.
We assert that for any good pair (x, y), it must be that x = y. Oth-
erwise, suppose x = y. Then the conditions imply that neither (x, x) nor
(y, y) are good pairs, i.e., xf (x) > 1 and yf (y) > 1. Using the AM-GM
inequality, we have
xf (y) + yf (x) ≥ 2 xf (y) · yf (x) = 2 xf (x) · yf (y) > 2,
1 1
xf (y) + yf (x) ≤ x · + · f (x) = xf (x) + 1 ≤ 2,
y f (x)
1
indicating that x, f (x) is also a good pair.
1
By the assertion, x = f (x) , and thus f (x) = x1 .
(i) f is strictly decreasing, i.e., f (y1 ) > f (y2 ) for any y1 < y2 .
Otherwise, suppose f (y1 ) ≤ f (y2 ). Let x = σ(y2 ). Then
Score Situation This particular problem saw the following distribution of scores among
contestants: 303 contestants scored 7 points, 19 contestants scored 6 points, 3 contestants
scored 5 points, 7 contestants scored 4 points, 41 contestants scored 3 points, 23 contestants
scored 2 points, 89 contestants scored 1 point, and 104 contestants scored 0 point. The
average score for this problem is 4.306, indicating that it was simple.
Among the top five teams in the team scores, the scores of this problem are as follows:
the China team scored 42 points (with a total team score of 252 points), the South Korea
team scored 42 points (with a total team score of 208 points), the United States team scored
42 points (with a total team score of 207 points), the Vietnam team scored 42 points (with
a total team score of 196 points), and the Romania team scored 41 points (with a total team
score of 194 points).
150 IMO Problems, Theorems, and Methods: Algebra
The gold medal cutoff for this IMO was set at 34 points (with 44 contestants earning gold
medals), the silver medal cutoff was 29 points (with 101 contestants earning silver medals),
and the bronze medal cutoff was 23 points (with 140 contestants earning bronze medals).
In this IMO, a total of 10 contestants achieved a perfect score of 42 points.
2.3 Summary
Functional equations constitute a rich and broadly applied branch of math-
ematics. In 2000, one of the seven Millennium Prize Problems released by
the Clay Mathematics Institute in the United States concerned the issue
of smoothness and existence of solutions for the three-dimensional Navier-
Stokes equations.
In the first 64 IMOs, there were a total of 28 function problems.
These problems can be broadly categorized into three types, as depicted in
Figure 2.1. The score details for these problems are presented in Table 2.2.
Due to the smaller number of participating teams and missing contestant
score information in early IMOs, there are several blanks in Table 2.2.
6
Proving Properties Determining Values Deriving Expressions
5
0
1–10 11–20 21–30 31–40 41–50 51–60 61–64
Full points 7.000 7.000 7.000 6.000 8.000 7.000 7.000 7.000
Average score 4.593 4.366 2.818 3.376 3.054 3.523 3.383 1.055
Full points 7.000 8.000 7.000 7.000 7.000 7.000 7.000 7.000
Average score 2.452 3.313 6.510 4.613 1.731 3.575 4.152 2.955
Top five mean 6.633 5.025 6.900 6.700 4.167 6.400 6.067
6th–15th mean 5.983 2.921 6.096 5.525 2.864 5.450 3.833
16th–25th mean 3.700 4.492 4.175 1.276 3.655 2.926
Problem number in IMO 54-5 20-3 22-6 23-1 29-3 24-1 27-5 31-4
Proposing country Bulgaria The United Finland The United The United The United The United Turkey
Kingdom Kingdom Kingdom Kingdom Kingdom
(Continued )
Table 2.2 (Continued )
Full points 7.000 7.000 7.000 7.000 7.000 7.000 7.000 7.000
Average score 2.963 3.221 1.151 2.267 4.402 2.474 5.450 3.766
Top five mean 5.667 6.467 3.200 6.467 6.967 6.800 6.889 6.306
6th–15th mean 4.350 5.100 2.083 4.550 6.383 6.212 6.648 5.333
16th–25th mean 3.817 4.067 1.136 3.500 6.417 5.467 6.667 5.450
Problem number in IMO 33-2 35-5 40-6 43-5 49-4 50-5 51-1 53-4
Function Problems
Proposing country India The United Japan India South France France South
Kingdom Korea Africa
Note. Top five mean = Total score of the top five teams ÷ Total number of contestants from the top five teams,
6th–15th mean = Total score of the 6th–15th teams ÷ Total number of contestants from the 6th–15th teams,
16th–25th mean = Total score of the 16th–25th teams ÷ Total number of contestants from the 16th–25th teams.
153
154 IMO Problems, Theorems, and Methods: Algebra
Problem Number
Number of Problems in
Function Problem 1, 4 2, 5 3, 6 the First 64 IMOs
Proving properties 1 2 1 9
Determining values 0 0 1 4
Deriving expressions 6 8 1 15
Total 7 10 3 28
0
0 2 4 6 8 10 12 14 16 18 20
(IMO 31-4), Problem 2.6 (IMO 28-4), and Problem 2.15 (IMO 27-5). This
phenomenon is due to the smaller number of participating teams in early
IMOs. It was not until the 30th IMO in 1989 that the number of partici-
pating teams exceeded 50. Therefore, it is common to see situations where
the average score is close to or even higher than the average score of the
16th–25th teams during this period.
Chapter 3
Sequence Problems
155
156 IMO Problems, Theorems, and Methods: Algebra
k × (k + 1) × · · · × (k + p − 1) p 1
, k , and .
1 × 2 × ···× p k × (k + 1) × · · · × (k + p − 1)
Session
Content 1–10 11–20 21–30 31–40 41–50 51–60 61–64 Total
Determining values 1 0 0 1 0 2 0 4
Existence problems 0 2 1 1 0 0 1 5
Proving quantitative 0 1 1 1 1 1 0 5
relationships
Algebra problems 20 20 14 13 15 13 6 101
The percentage of sequence 5.0% 15.0% 14.3 % 23.1% 6.7% 23.1% 16.7% 13.8%
problems among the
algebra problems
quite challenging. For example, the average score of Problem 3.14 (IMO
51-6) is merely 0.368 points, with as many as 470 contestants scoring
zero.
Addressing sequence problems typically involves two primary methods.
The first is the recursive method, such as the use of mathematical induction.
The second is the algebraic method, which demands proficiency in tech-
niques including substitution, identity transformations, and finding the
general term of a sequence. In mathematics competitions, many sequence
problems often start with algebraic transformations.
This chapter will be divided into three parts. The first part intro-
duces some common types and properties of sequences. Although peri-
odic sequences are usually associated with divisibility and congruence, their
properties are also reviewed here. Subsequently, commonly used methods
for finding the general term and sum of sequences are discussed, along with
several theorems that are easily overlooked.
The second part revolves around three types of problems: “determining
the value of a specific term or the number of terms,” “addressing problems
related to the existence of sequences,” and “proving quantitative relation-
ships satisfied by sequences.” These problems are presented in chronolog-
ical order, and some problems include various solutions, generalizations,
and similar problems.
It is important to note that for each problem, the solutions are fol-
lowed by information on the scores, including the number of contestants in
each score range, the average score, and the scores of the top five teams.
However, early IMOs often lacked information on contestant scores, so the
158 IMO Problems, Theorems, and Methods: Algebra
number of contestants in each score range only represents the counted num-
ber of contestants, and some problems lack scores of the top five teams.
The third part provides a brief summary of this chapter.
Proposition 3.8. If {an } and {bn } are both arithmetic sequences, then
{kan + b} and {man + nbn } are also arithmetic sequences, where k, b, m, n
are constants.
Proposition 3.9. Let An and Bn denote the sums of the first n terms of
arithmetic sequences {an } and {bn } respectively. Then abnn = A 2n−1
B2n−1 .
Proposition 3.10. If sequences {an } and {bn } satisfy the condition that
for n ≥ 1,
a1 + 2a2 + · · · + nan
bn = ,
1 + 2 + ··· + n
then {bn } is an arithmetic sequence if and only if {an } is an arithmetic
sequence.
a + d, a + 2d, . . . , a + nd, . . .
a
has three terms in a geometric progression if and only if d is a rational
number.
a2 − a1 , a3 − a2 , . . . , an+1 − an , . . . ;
b2 − b1 , b3 − b2 , . . . , bn+1 − bn , . . . ;
this sequence is termed the second order difference sequence of the original
sequence {an }.
Following this pattern, for any p ∈ N+ , the pth order difference sequence
of {an } can be defined. If the pth order difference sequence of {an } is a
non-zero constant sequence, then {an } is referred to as a pth order arith-
metic sequence. Particularly, a first order arithmetic sequence is commonly
known as an arithmetic sequence, while second and higher-order arithmetic
sequences are collectively termed high-order arithmetic sequences. The fol-
lowing propositions hold:
Proposition 3.18. If {an } is a pth order arithmetic sequence, then its first
order difference sequence is a (p − 1)th order arithmetic sequence.
Sequence Problems 161
and the sum of its first n terms is denoted as Sn , then the sequence {Sn }
is a (k + 1)th order linear recursive sequence with the recurrence relation
xn = axn−1 + byn−1 ,
yn = cxn−1 + dyn−1 ,
If the characteristic equation has two distinct roots λ1 and λ2 , then the
general term formulas for {xn } and {yn } are
xn = Aλn1 + Bλn2 ,
yn = Cλn1 + Dλn2 ,
where the constants A, B, C, D are determined by the initial conditions:
x1 = Aλ1 + Bλ2 , x2 = Aλ21 + Bλ22 ,
and
y1 = Cλ1 + Dλ2 y2 = Cλ21 + Dλ22 .
If the characteristic equation has a double root λ, then the general term
formulas for {xn } and {yn } become:
xn = (An + B)λn ,
yn = (Cn + D)λn ,
where the constants A, B, C, D are determined by the initial conditions:
x1 = (A + B) λ, x2 = (2A + B) λ2 ,
and
y1 = (C + D) λ y2 = (2C + D) λ2 .
xn −axn−1
Actually, by substituting yn−1 = b into yn = cxn−1 + dyn−1 ,
we obtain
xn+1 = (a + d)xn + (bc − ad)xn−1 .
αan +β
(iii) an+1 = an +γ , where α, β, γ are constants.
αx+β
Consider the fixed points of the function f (x) = x+γ , i.e., the solu-
αx+β
tions of the equation x = x+γ are x1 , x2 .
n−1
an −x1
• If x1 = x2 , then an −x2 = aa11 −x α−x1
−x2 · α−x2
1
.
• If x1 = x2 , then 1
an −x1 = 1
a1 −x1 + (n − 1) · 1
α−x1 .
2x+6
Solution. Find the fixed points by setting x = x+1 , yielding x1 = 3 and
x2 = −2. Therefore,
2an + 6 −an + 3 2an + 6 4an + 8
an+1 − 3 = −3= , an+1 + 2 = +2= .
an + 1 an + 1 an + 1 an + 1
Dividing the two equations, we get
an+1 − 3 an − 3
=− ,
an+1 + 2 4(an + 2)
which implies that aan+1 −3
n+1 +2
is a geometric sequence with a common
a1 −3 an −3
ratio of − 14 and an initial term of a1 +2 = − 4 .
1
Consequently, an +2 =
n−1 n n
− 14 − 41 = − 14 . Thus, an = 3(−4) +2
(−4)n −1 .
• If α = β, then an = 1
α−β (a2 − βa1 )αn−1 − (a2 − αa1 )β n−1 .
a1 a2 −αa1
• If α = β, then an = α + (n − 1) · α2 αn .
x2 − 2pan x + a2n − 1 = 0.
(viii) 1
k2 < 4
4k2 −1 < 1
k2 −1 ≤ 1
k2 −k < 1
k2 −2k .
1 1 1 1
(ix) k2 > k2 +k > k2 +2k > k2 +3k+2 .
√ √
(x) √1
k
= 2
√
2 k
< √ 2
√
k+ k−1
= 2( k − k − 1).
Sequence Problems 169
√ √
(xi) √1
k
= 2
√
2 k
> √ = 2( k + 1 − k).
2
√
k+ k+1
1
(xii) k√ k
< √1
k k−1
= √ √ 1√
k· k· k−1
= √1
k
√1
k−1
− √1
k
√ 1
√
k− k−1
< 2 √k−11
− √1k .
1
(xiii) k√ > √1 = √ √ 1√ = √1 √1 − √ 1 √ 1 √
k k k+1 k· k· k+1 k k k+1 k+1− k
> 2 √1k − √k+11
.
• P (1) is true;
• assuming P (k) is true, one can deduce that P (k + 1) is also true;
• P (1) is true;
• assuming that P (k) holds for all positive integers k less than n, one can
deduce that P (n) is also true;
Corollary 3.2. Any finite set of real numbers necessarily contains both a
minimum element and a maximum element.
C1 = a1 + a2 + · · · + a8 ,
C2 = a21 + a22 + · · · + a28 ,
························
Cn = an1 + an2 + · · · + an8 ,
························
in which a1 , a2 , . . . , a8 are real numbers not all equal to zero. Suppose that
an infinite number of terms of the sequence {Cn } are equal to zero. Find
all positive integers n for which Cn = 0.
Solution. Assume that a1 has the largest absolute value. Then there must
= 1) such that ai = −a1 . Otherwise, for sufficiently large n,
exist some ai (i
n
ai
it holds that a1 either are constantly 1 or tend to zero (i = 1). Thus,
n n
n a2 a8
|Cn | = |a1 | 1 + + ···+ > 0,
a1 a1
which contradicts the given conditions.
Without loss of generality, let a2 = −a1 . Since n that makes Cn = 0
must be odd, an1 + an2 = 0. Removing ak1 and ak2 from Ck , the remaining
{Cn } still has infinitely many terms equal to zero. Following the above
172 IMO Problems, Theorems, and Methods: Algebra
a1 + 2k a2 + 3k a3 + · · · + mk am = 0,
Score Situation This particular problem saw the following distribution of scores among
contestants: 13 contestants scored 7 points, no contestant scored 6 points, 2 contestants
Sequence Problems 173
Therefore, xi = xi−12
1
or xi = xi−1 .
Suppose, starting from x0 , there are k − t transformations of the first
kind and t transformations of the second kind to reach xk . It is evident
(−1)t
that xk = 2s x0 , where s ≡ k − t(mod2).
Now consider the case k = 1995. If t is even, then x0 = x1995 = 2s x0 .
Since x0 > 0, it follows that s = 0, implying 0 ≡ 1995 − t(mod2), which
contradicts the assumption that t is even. Therefore, t must be odd, and
s
x0 = x1995 = 2s x−10 , yielding x0 = 2 .
2
Score Situation This particular problem saw the following distribution of scores among
contestants: 168 contestants scored 7 points, 48 contestants scored 6 points, 37 contestants
scored 5 points, 25 contestants scored 4 points, 18 contestants scored 3 points, 16 contestants
scored 2 points, 57 contestants scored 1 point, and 43 contestants scored 0 point. The
average score for this problem is 4.592, indicating that it was simple.
Among the top five teams in the team scores, the scores of this problem are as follows:
the China team scored 36 points (with a total team score of 236 points), the Romania team
scored 42 points (with a total team score of 230 points), the Russia team scored 42 points
(with a total team score of 227 points), the Vietnam team scored 42 points (with a total
team score of 220 points), and the Hungary team scored 42 points (with a total team score
of 210 points).
The gold medal cutoff for this IMO was set at 37 points (with 30 contestants earning gold
medals), the silver medal cutoff was 29 points (with 71 contestants earning silver medals),
and the bronze medal cutoff was 19 points (with 100 contestants earning bronze medals).
In this IMO, a total of 14 contestants achieved a perfect score of 42 points.
term greater than N 2 , and take the smallest k such that ak > N 2 . Since
3|ak , we have ak ≥ N 2 + 3. However, ak−1 ≤ N 2 and ak − ak−1 ≤ 3, so
ak−1 = N 2 . According to the definition, ak = N , leading to a contradic-
tion. Therefore, there is no ak > N 2 , and the sequence {an } is bounded,
satisfying the conditions for such a0 .
Lastly, consider the case a0 ≡ 1(mod 3). It is easy to see that if ak is not
divisible by 3 and ak > 1, then 3 does not divide ak+1 and ak+1 > 1. Thus,
every term in the sequence {an } is not divisible by 3 and greater than 1.
Suppose {an } is bounded, i.e., it eventually becomes periodic. Then each
term in the sequence is congruent to 1 modulo 3. Taking the largest term
ak in the period, we see that ak is a perfect square; otherwise, ak+1 =
a k + 3 > ak .
Let ak = N 2 . Then ak+1 = N , where N ≡ 1(mod3) and N > 1, i.e.,
N ≥ 4. As there exists j > k such that aj = ak = N 2 > (N − 2)2
and ak+1 = N ≤ (N − 2)2 , take the smallest l > k + 1 such that al >
(N − 2)2 , and then al−1 ≤ (N − 2)2 < al , implying al = al−1 + 3. Since
al ≡ al−1 ≡ 1(mod3), it follows that al−1 = (N − 2)2 ≡ 1(mod3), and by
definition, al = N − 2 ≡ 2(mod3), resulting in a contradiction. Therefore,
a0 ≡ 1(mod3) does not satisfy the conditions.
In conclusion, the values of a0 satisfying the conditions are all multiples
of 3.
Note. There are several similar problems:
• (Indian National Mathematical Olympiad 2016, Problem 3).
Let N denote the set of all natural numbers. Define a function T : N →
N by T (2k) = k and T (2k + 1) = 2k + 2. We write T 2 (n) = T (T (n))
and in general T k (n) = T k−1 (T (n)) for any k > 1.
(a) Show that for each n ∈ N, there exists k such that T k (n) = 1.
(b) For k ∈ N, let ck denote the number of elements in the set
{n|T k (n) = 1}. Prove that ck+2 = ck+1 + ck , for k ≥ 1.
• (IMO Shortlist 2015, Number Theory, Problem 4). Suppose that
a0 , a1 , . . . and b0 , b1 , . . . are two sequences of positive integers satisfying
a0 , b0 ≥ 2 and
an+1 = gcd (an , bn ) + 1, bn+1 = lcm(an , bn ) − 1
for all n ≥ 0. Prove that the sequence {an } is eventually periodic; in
other words, there exist integers N ≥ 0 and t > 0 such that an+t = an
for all n ≥ N .
176 IMO Problems, Theorems, and Methods: Algebra
is perodic (there exists an integer d > 0 such that an+d = an for all n).
• (All-Russian Mathematical Olympiad 2000, 4th Round, Grade
10, Problem 6). Given a positive integer a0 , construct a sequence {an }
as follows:
a2n − 5, if an is odd,
an+1 = an
2 , if an is even.
Prove that for any odd integer a0 > 5 and real number M > 0, there is
a positive integer k such that ak > M .
• (Baltic Way Mathematical Contests 1997, Problem 5). In a
sequence u0 , u1 , . . . of positive integers, u0 is arbitrary, and for any non-
negative integer n,
un
2 , for even un ,
un+1 =
a + un , for odd un ,
where a is a fixed odd positive integer. Prove that the sequence is periodic
from a certain step.
Score Situation This particular problem saw the following distribution of scores among
contestants: 446 contestants scored 7 points, 25 contestants scored 6 points, 54 contestants
scored 5 points, 12 contestants scored 4 points, 5 contestants scored 3 points, 17 contestants
scored 2 points, 16 contestants scored 1 point, and 40 contestants scored 0 point. The
average score for this problem is 5.943, indicating that it was simple.
Among the top five teams in the team scores, the scores of this problem are as follows:
the South Korea team scored 42 points (with a total team score of 170 points), the China
team scored 42 points (with a total team score of 159 points), the Vietnam team scored 42
points (with a total team score of 155 points), the United States team scored 42 points (with
a total team score of 148 points), and the Iran team scored 42 points (with a total team
score of 142 points).
Sequence Problems 177
The gold medal cutoff for this IMO was set at 25 points (with 48 contestants earning gold
medals), the silver medal cutoff was 19 points (with 90 contestants earning silver medals),
and the bronze medal cutoff was 16 points (with 153 contestants earning bronze medals).
In this IMO, no contestant achieved a perfect score of 42 points.
Particularly, ai+1 > ai+1−2n , which contradicts the fact that the
defined sequence {am } (m ∈ Z) has n as its period.
Consequently, it is known from (i), (ii), (iii), and (iv) that in any con-
secutive three terms ai , ai+1 , ai+2 , exactly one term is positive, and the
other two are negative. Thus, the sign of this sequence follows a period of
3. Since there is at least one positive term, say ai > 0, then ai+n > 0,
leading to 3|n.
Solution 2. The example part follows Solution 1. Here, we present an
alternative proof for 3|n. Extend a1 , a2 , . . . , an into an infinite sequence
with period n at both ends. Then, for any i,
n
which is equivalent to i=1 (ai − ai+3 )2 = 0.
Therefore, ai = ai+3 for any i. Hence, the sequence {ai } also has a
period of 3. If n is not a multiple of 3, then the sequence {ai } has a period
of 1, making it a constant sequence. However, the equation x2 + 1 = x
has no real solution, and there is no constant sequence satisfying the given
conditions. Thus, 3|n.
Sequence Problems 179
1 1
a2n−1 = + , a2n = a2n−1 + a1 .
a2n−2 a2n
• (British Mathematical Olympiad 2020, 2nd Round, Problem 4).
A sequence b1 , b2 , b3 . . . of nonzero real numbers has the property that
b2n+1 − 1
bn+2 =
bn
for all positive integers n.
Suppose that b1 = 1 and b2 = k, where 1 < k < 2. Show that there
is some constant B, depending on k, such that −B ≤ bn ≤ B for all
n. Also show that, for some 1 < k < 2, there is a value of n such that
bn > 2020.
Score Situation This particular problem saw the following distribution of scores among
contestants: 157 contestants scored 7 points, 7 contestants scored 6 points, 16 contestants
scored 5 points, 18 contestants scored 4 points, 66 contestants scored 3 points, 87 contestants
scored 2 points, 85 contestants scored 1 point, and 158 contestants scored 0 point. The
average score for this problem is 2.946, indicating that it had a certain level of difficulty.
180 IMO Problems, Theorems, and Methods: Algebra
Among the top five teams in the team scores, the scores of this problem are as follows:
the United States team scored 41 points (with a total team score of 212 points), the Russia
team scored 42 points (with a total team score of 201 points), the China team scored 37 points
(with a total team score of 199 points), the Ukraine team scored 34 points (with a total team
score of 186 points), and the Thailand team scored 38 points (with a total team score of 183
points).
The gold medal cutoff for this IMO was set at 31 points (with 48 contestants earning gold
medals), the silver medal cutoff was 25 points (with 98 contestants earning silver medals),
and the bronze medal cutoff was 16 points (with 143 contestants earning bronze medals).
In this IMO, only two contestants achieved a perfect score of 42 points, namely Agnijo
Banerjee from the United Kingdom and James Lin from the United States.
1 = a0 ≤ a1 ≤ a2 ≤ · · · ≤ an ≤ · · · .
n
ak−1 1
bn = 1− √ .
ak ak
k=1
ak−1
Proof. (a) It is evident that ak ≤ 1, implying bn ≥ 0. Moreover,
√ √
ak−1 1 ak−1 ak−1 1
1− √ = 1+ √ 1− √ √
ak ak ak ak ak
√ √
ak−1 ak−1 1 1
= √ 1+ √ √ −√
ak ak ak−1 ak
1 1
≤2 √ −√ .
ak−1 ak
Sequence Problems 181
Thus,
n n
ak−1 1 1 1
bn = 1− √ ≤2 √ −√
ak ak ak−1 ak
k=1 k=1
1 1 1
=2 √ −√ =2 1− √ < 2.
a0 an an
(b) Choose an appropriate positive number d ∈ (0, 1) such that
ak = d−2k , yielding √1ak = dk . It follows that.
n
n
ak−1 1
bn = 1− √ = 1 − d2 dk
ak ak
k=1 k=1
n
1−d
= d 1 − d2 = d (1 + d) (1 − dn ) .
1−d
c
For any positive number c < 2, let d = 2 , and it is evident that
d(1 + d) > c. Since 0 < d < 1, we have lim (1 − dn ) = 1, i.e., for n
n→+∞
sufficiently large,
√
n 2 c c
1−d > √ √ = .
2+ c d(1 + d)
Therefore, there exist infinitely many n such that bn > c.
Note. There are several similar problems:
• (Turkey Mathematical Olympiad 1999, 2nd Round, Problem 4).
Find all sequences a1 , a2 , . . . , a2000 of real numbers with 2000
n=1 an = 1999
such that
1
< an < 1 and an = an−1 (2 − an−1 ) for all n > 1.
2
• (Chinese Northern Mathematical Olympiad 2006, Problem 8).
Given a sequence {an } such that a0 = 12 and an+1 = an + 2006
1
a2n for
n ∈ N, prove that 1 − 1
2008 < a2006 < 1.
Score Situation This particular problem saw the following distribution of scores among
contestants: 5 contestants scored 8 points, no contestant scored 7 points, 1 contestant
scored 6 points, no contestant scored 5 points, 1 contestant scored 4 points, no contestant
scored 3 points, 1 contestant scored 2 points, 1 contestant scored 1 point, and 26 contestants
scored 0 point. The average score of this problem is 1.514, indicating that it was relatively
challenging.
182 IMO Problems, Theorems, and Methods: Algebra
Among the top five teams in the team scores, the Hungary team achieved a total score
of 233 points, the German Democratic Republic team achieved a total score of 221 points,
the Soviet Union team achieved a total score of 221 points, the Yugoslavia team achieved a
total score of 209 points, and the Romania team achieved a total score of 208 points.
The gold medal cutoff for this IMO was set at 37 points (with 7 contestants earning gold
medals), the silver medal cutoff was 30 points (with 11 contestants earning silver medals),
and the bronze medal cutoff was 19 points (with 40 contestants earning bronze medals).
In this IMO, only three contestants achieved a perfect score of 40 points, namely Wolfgang
Burmeister from the German Democratic Republic, Imre Ruzsa from Hungary, and Andrei
Hodulev from the Soviet Union.
and
bk+1
so q < bk < 1q .
(c) Note that
n
b1 + b2 + · · · + bn = (a1 q k−1 + a2 q k−2 + · · · + ak + ak+1 q
k=1
+ · · · + an q n−k )
Sequence Problems 183
u 1 = (1, 0, 0, . . . , 0) ,
u 2 = (0, 1, 0, . . . , 0) ,
·····················
u n = (0, 0, 0, . . . , 1) ,
Score Situation This particular problem saw the following distribution of scores among con-
testants: 10 contestants scored 8 points, 2 contestants scored 7 points, 2 contestants scored
6 points, 3 contestants scored 5 points, 3 contestants scored 4 points, 1 contestant scored
3 points, 2 contestants scored 2 points, 1 contestant scored 1 point, and 101 contestants
scored 0 point. The average score of this problem is 1.128, indicating that it was relatively
challenging.
Among the top five teams in the team scores, the scores of this problem are as follows:
the Soviet Union team scored 30 points (with a total team score of 254 points), the Hungary
team scored 11 points (with a total team score of 215 points), the German Democratic
Republic team scored 14 points (with a total team score of 188 points), the Poland team
scored 8 points (with a total team score of 174 points), and the United Kingdom team scored
25 points (with a total team score of 164 points).
184 IMO Problems, Theorems, and Methods: Algebra
The gold medal cutoff for this IMO was set at 35 points (with 5 contestants earning gold
medals), the silver medal cutoff was 27 points (with 15 contestants earning silver medals),
and the bronze medal cutoff was 17 points (with 48 contestants earning bronze medals).
In this IMO, only one contestant achieved a perfect score of 40 points, namely Sergei
Konyagin from the Soviet Union.
(a) Prove that for every such sequence, there is an n ≥ 1 such that
x20 x2 x2
+ 1 + · · · + n−1 ≥ 3.999.
x1 x2 xn
x20 x2 x2
+ 1 + · · · + n−1 < 4.
x1 x2 xn
x2i
implying xi+1 ≥ 4(xi − xi+1 ). Thus,
x20 x2 x2
+ 1 + · · · + n−1 ≥ 4(x0 − x1 ) + · · · + 4(xn−1 − xn ) = 4 − 4xn .
x1 x2 xn
1
(b) Choose xi = 2i , as
x20 x2 x2 1 1 1
+ 1 + · · · + n−1 = 2 + 1 + + · · · + n−2 = 4 − n−2 < 4.
x1 x2 xn 2 2 2
Solution 2. We construct a positive sequence {Cn } such that for each
sequence satisfying
x0 ≥ x1 ≥ · · · ≥ xn ≥ · · · > 0,
the following inequality holds:
x20 x2 x2
+ 1 + · · · + n−1 ≥ Cn x0 .
x1 x2 xn
Hence,
x20 x21 x2 x2 x2
+ + · · · + n−1 + n ≥ 0 + Cn x1 ≥ 2 Cn x0 ,
x1 x2 xn xn+1 x1
√
implying Cn+1 = 2 Cn .
x2
Given that x01 ≥ x0 , we can choose C1 = 1. Consequently, the general
1
term of {Cn } is given by Cn = 4 × 2− 2n−2 for n = 1, 2, . . ..
To show that Cn ≥ 3.999, we need to find an n ≥ 1 such that
2n−2
4
3.999 ≥ 2. Since
2n−2 2n−2
4 1 2n−2
> 1+ >1+ ,
3.999 4000 4000
212 4096
when n = 14, we have 1 + 4000 =1+ 4000 > 2.
x20 x21 x2n−1
Therefore, + x1 + · · · + xn ≥ Cn x0 = Cn > 2 for n ≥ 14. The
x2
construction for (b) is the same as in Solution 1.
x2 x2 x2
Solution 3. Let Sn = x01 + x12 + · · · + xn−1
n
. Since the sequence {xn } is a
non-increasing sequence of positive real numbers,
x2 x2 x1
S2 = 0 + 1 ≥ 2x0 ≥ 2x0 = 2,
x1 x2 x2
x20 x2 x2 x2
S3 = + 1 + 2 ≥ 0
x1 x2 x3 x1
x2 x2 √ 1
+ 2x1 ≥ 0 + 2x1 ≥ 2 2x0 = 21+ 2 ,
x3 x1
186 IMO Problems, Theorems, and Methods: Algebra
x20 x2 x2 x2 x2 x2
S4 = + 1 + 2 + 3 ≥ 0 + 1 + 2x2
x1 x2 x3 x4 x1 x2
x2 √ √ 1 1
≥ 0 + 2 2x1 ≥ 2 2 2 = 21+ 2 + 4 ,
x1
1 1
and so on, leading to the general inequality Sn ≥ 21+ 2 +···+ 2n−2 .
As limn→∞ 1 + 12 + · · · + 2n−2
1
= 2, there exists a positive integer N
such that Sn > 3.999 for n > N .
The construction for (b) is the same as in Solution 1.
Note. (i) For any positive sequence {xn } with x0 = 1 and xi+1 ≤ xi for
∞ x2i
i ≥ 0, i=0 xi+1 ≥ 4 is valid.
(ii) For a positive sequence {xn } with x0 = 1 and xi+1 ≤ xi for i ≥ 0,
∞ t
xr+t
i r+t r r+t
≥ ·
xt
i=0 i+1
t r
ri
t
is true, where r, t ∈ R+ . The equality is achieved when xi = r+t for
i ≥ 0.
Score Situation This particular problem saw the following distribution of scores among
contestants: 27 contestants scored 7 points, 5 contestants scored 6 points, 2 contes-
tants scored 5 points, 9 contestants scored 4 points, 8 contestants scored 3 points,
36 contestants scored 2 points, 1 contestant scored 1 point, and 31 contestants scored
0 point. The average score for this problem is 3.042, indicating that it was relatively
straightforward.
Among the top five teams in the team scores, the scores of this problem are as follows:
the Germany team scored 27 points (with a total team score of 145 points), the Soviet Union
team scored 23 points (with a total team score of 137 points), the German Democratic
Republic team scored 28 points (with a total team score of 136 points), the United States
team scored 27 points (with a total team score of 136 points), and the Vietnam team scored
20 points (with a total team score of 133 points).
The gold medal cutoff for this IMO was set at 37 points (with 10 contestants earning gold
medals), the silver medal cutoff was 30 points (with 20 contestants earning silver medals),
and the bronze medal cutoff was 21 points (with 31 contestants earning bronze medals).
In this IMO, only three contestants achieved a perfect score of 42 points, namely Bruno
Haible from Germany, Grigori Perelman from the Soviet Union, and Le Tu QuocThang from
Vietnam.
Sequence Problems 187
|xi − xj | ≥ |i − j|−a
i = b0 + b1 · 2 + b2 · 2 2 + · · · + br · 2 r ,
It is evident that |yi | < 1−21−a , making {yi } a bounded sequence. Next,
estimate |yi − yj | for i = j.
Assume j = c0 + c1 · 2 + c2 · 22 + · · · + cs · 2s . Since i = j, there exists
a non-negative integer t0 such that bt0 = ct0 and bt = ct for 0 ≤ t < t0 .
Consequently, |i − j| ≥ 2t0 , and
Solution 2. We first prove that if the following lemma holds, then the
problem is true for a = 1.
where t denotes the greatest integer not exceeding the real number t.
Clearly, 0 ≤ xi < c−1 , making {xi } a bounded sequence. From (1), for
i = j,
which implies that the bounded infinite sequence {xi } satisfies the require-
ments of the given problem when a = 1.
Clearly, |xi − xj | ≥ |i − j|−1 > |i − j|−a for a > 1.
√ √ −1
Next, we prove that when α = 2 and c = 2 + 2 , inequality (1)
holds for any integers p = 0 and q.
(i) Assume 1 ≤ pq ≤ 2. From 2p2 − q 2 = 0,
√ √
1 ≤ |2p2 − q 2 | = 2p + q · 2p − q
q √
√
= |p| · 2 +· 2p − q
p
√ √
≤ 2 + 2 |p| · 2p − q .
q
(ii) Assume p < 1. It is evident that
√
√ q √ √ −1 (2 + 2)−1
2− > 2−1> 2+ 2 ≥ .
p p2
Sequence Problems 189
q
(iii) Assume p > 2. We have
√
q √
−1
√ √ −1 2+ 2
2− > 2 − 2 > 2 + 2 ≥ .
p p2
Solution 3. For a real number a > 1, it is evident that there exists a finite
constant Ca such that
1 1 1
+ a + · · · + a < Ca for n = 1, 2, 3, . . . . (2)
1a 2 n
The definition of the sequence x0 , x1 , x2 , . . . will be inductive. Let x0 =
0, fix an integer k ≥ 1, and assume we have already defined real numbers
x0 , x1 , . . . , xk−1 satisfying the conditions
a
|xi | ≤ Ca , |xi − xj | · |i − j| ≥ 1. (3)
∞
1 1 1 1
p
= 1 + p + p + ···+ p + ···
n=1
n 2 3 n
190 IMO Problems, Theorems, and Methods: Algebra
• (Turkey Team Selection Test 2014, Problem 1). Find the number
of (a1 , a2 , . . . , a2014 ) permutations of the (1, 2, . . . , 2014) such that i+ai ≤
j + aj for all 1 ≤ i < j ≤ 2014.
• (Turkey Team Selection Test 2005, Problem 4). Show that for any
integer n ≥ 2 and all integers a1 , a2 , . . . , an , the product i<j (aj − ai )
is divisible by i<j (j − i).
• (IMO Shortlist 2002, Algebra, Problem 2). Let a1 , a2 , . . . be an
infinite sequence of real numbers, for which there exists a real number c
with 0 ≤ ai ≤ c for all i, such that
1
|ai − aj | ≥ for all i, j with i = j.
i+j
Prove that c ≥ 1.
• (All-Russian Mathematical Olympiad 1998, 4th Round, Grade
11, Problem 8). Given a sequence {an } (n ∈ N) of natural numbers
such that every natural number occurs and for any distinct natural num-
bers m and n,
1 |an − am |
< < 1998,
1998 |n − m|
prove that |an − n| < 2000000 for all natural numbers n.
Sequence Problems 191
√
1
xn − 2 < n for n ≥ 3.
2
Score Situation This particular problem saw the following distribution of scores among
contestants: 50 contestants scored 7 points, 3 contestants scored 6 points, 5 contes-
tants scored 5 points, 10 contestants scored 4 points, 16 contestants scored 3 points,
27 contestants scored 2 points, 29 contestants scored 1 point, and 172 contestants scored
0 point. The average score for this problem is 1.808, indicating that it was relatively
challenging.
Among the top five teams in the team scores, the scores of this problem are as follows:
the Soviet Union team scored 42 points (with a total team score of 241 points), the China
team scored 32 points (with a total team score of 231 points), the Romania team scored 32
points (with a total team score of 225 points), the Germany team scored 36 points (with a
total team score of 222 points), and the United States team scored 31 points (with a total
team score of 212 points).
The gold medal cutoff for this IMO was set at 39 points (with 20 contestants earning gold
medals), the silver medal cutoff was 31 points (with 51 contestants earning silver medals),
and the bronze medal cutoff was 19 points (with 84 contestants earning bronze medals).
In this IMO, a total of nine contestants achieved a perfect score of 42 points.
f (x) = 1 − f (x − a1 )f (x − a2 ) · · · f (x − an )
for each integer x ≥ 0. Show that there exist positive integers s and t
such that f (x + t) = f (x) for any integer x > s.
Score Situation This particular problem saw the following distribution of scores among
contestants: 73 contestants scored 7 points, 3 contestants scored 6 points, 6 contestants
scored 5 points, 8 contestants scored 4 points, 23 contestants scored 3 points, 7 contestants
scored 2 points, 102 contestants scored 1 point, and 396 contestants scored 0 point. The
average score of this problem is 1.256, indicating that it was relatively challenging.
Among the top five teams in the team scores, the scores of this problem are as follows:
the China team scored 42 points (with a total team score of 240 points), the United States
team scored 42 points (with a total team score of 222 points), the South Korea team scored
42 points (with a total team score of 215 points), the Romania team scored 42 points (with
a total team score of 208 points), and the Canada team scored 25 points (with a total team
score of 183 points).
The gold medal cutoff for this IMO was set at 32 points (with 54 contestants earning gold
medals), the silver medal cutoff was 25 points (with 90 contestants earning silver medals),
and the bronze medal cutoff was 18 points (with 170 contestants earning bronze medals).
In this IMO, a total of five contestants achieved a perfect score of 42 points.
5
u0 = 2, u1 = , un+1 = un (u2n−1 − 2) − u1 for n = 1, 2, . . . .
2
Among the top five teams in the team scores, the scores of this problem are as follows:
the Soviet Union team scored 45 points (with a total team score of 250 points), the United
Kingdom team scored 50 points (with a total team score of 214 points), the United States
team scored 43 points (with a total team score of 188 points), the Bulgaria team scored 55
points (with a total team score of 174 points), and the Austria team scored 55 points (with
a total team score of 167 points).
The gold medal cutoff for this IMO was set at 34 points (with 9 contestants earning gold
medals), the silver medal cutoff was 23 points (with 28 contestants earning silver medals),
and the bronze medal cutoff was 15 points (with 45 contestants earning bronze medals).
In this IMO, only one contestant achieved a perfect score of 40 points, namely Laurent
Pierre from France.
for every n.
If 1 − 1
n+1 < xn+1 < 1, then evidently
1 1 1 1 1
xn = xn+1 + 2 − < 1+ + 2 − = 1,
4n 2n n 4n 2n
1 1 1 1 1 1 1
xn > 1 − + 2− > 1− + 2 − =1− .
n + 1 4n 2n n 4n 2n n
Sequence Problems 197
1
1− < Pn (t) < 1, n = 1, 2, . . . .
n
1 an
Pn (an ) = 1 − ≤ .
n bn
Score Situation This particular problem saw the following distribution of scores among
contestants: 23 contestants scored 7 points, 4 contestants scored 6 points, 10 contestants
scored 5 points, 9 contestants scored 4 points, 13 contestants scored 3 points, 19 contestants
scored 2 points, 74 contestants scored 1 point, and 57 contestants scored 0 point. The
average score for this problem is 2.019, indicating that it had a certain level of difficulty.
Among the top five teams in the team scores, the scores of this problem are as follows:
the Romania team scored 34 points (with a total team score of 201 points), the United
States team scored 40 points (with a total team score of 180 points), the Hungary team
scored 25 points (with a total team score of 168 points), the Bulgaria team scored 23 points
Sequence Problems 199
(with a total team score of 165 points), and the Vietnam team scored 12 points (with a total
team score of 144 points).
The gold medal cutoff for this IMO was set at 34 points (with 14 contestants earning gold
medals), the silver medal cutoff was 22 points (with 35 contestants earning silver medals),
and the bronze medal cutoff was 15 points (with 52 contestants earning bronze medals).
In this IMO, only two contestants achieved a perfect score of 42 points, namely Géza
Kós from Hungary and Daniel Tătaru from Romania.
Proof. From the conditions, it is evident that both ss1 , ss2 , ss3 , . . . and
ss1 +1 , ss2 +1 , ss3 +1 , . . . are strictly increasing sequences of positive integers.
Let ssk = a + (k − 1)d1 and ssk +1 = b + (k − 1)d2 for k = 1, 2, . . .,
where a, b, d1 , d2 are positive integers. From sk < sk + 1 ≤ sk+1 and the
monotonicity of {sn }, for any positive integer k,
i.e.,
a − b < (k − 1)(d2 − d1 ) ≤ a + d1 − b.
sa+id − sa+(i−1)d+1 ≥ c0 (d − 1)
be further expressed as the sum of two terms in the sequence, and so on.
Eventually, an can be represented as
Let ai1 , ai2 be the two terms at the last step in expressing an in the form
(2). Then i1 + i2 > s, and (3) becomes
1 ≤ ij ≤ s, i1 + i2 + · · · + ik = n, i1 + i2 > s. (4)
bn = an − mn for n = 1, 2, . . . ,
bn = an − mn
= max {ak + an−k | 1 ≤ k ≤ n − 1 } − mn
= max {bk + bn−k + mn | 1 ≤ k ≤ n − 1 } − mn
= max {bk + bn−k | 1 ≤ k ≤ n − 1 } ≤ 0,
leading to 0 ≥ bn ≥ bn−l ≥ · · · ≥ −M .
For the sequence {bn }, by (2) and (3), each bn belongs to the set
"
T = {bi1 + bi2 + · · · + bik : 1 ≤ i1 , i2 , . . . , ik ≤ s} [−M, 0],
Score Situation This particular problem saw the following distribution of scores among
contestants: 15 contestants scored 7 points, 4 contestants scored 6 points, 6 contestants
scored 5 points, no contestant scored 4 points, 3 contestants scored 3 points, 4 contestants
scored 2 points, 14 contestants scored 1 point, and 470 contestants scored 0 point. The
average score for this problem is 0.368, indicating that it was extremely difficult.
Among the top five teams in the team scores, the scores of this problem are as follows:
the China team scored 25 points (with a total team score of 197 points), the Russia team
scored 20 points (with a total team score of 169 points), the United States team scored
12 points (with a total team score of 168 points), the South Korea team scored 6 points
(with a total team score of 156 points), the Kazakhstan team scored 12 points (with a total
team score of 148 points), and the Thailand team scored 1 point (with a total team score of
148 points).
The gold medal cutoff for this IMO was set at 27 points (with 47 contestants earning gold
medals), the silver medal cutoff was 21 points (with 103 contestants earning silver medals),
and the bronze medal cutoff was 15 points (with 115 contestants earning bronze medals).
Sequence Problems 205
In this IMO, only one contestant achieved a perfect score of 42 points, namely Zipei Nie
from China.
3.3 Summary
Through various definitions, we can derive many interesting sequences that
are not only simple in form but also widely applicable. To a certain extent,
sequences also bridge secondary school mathematics with university-level
mathematics. For instance, the concepts of series and limits in caculus are
closely related to sequences.
In the first 64 IMOs, there were a total of 14 sequence problems.
These problems can be broadly categorized into three types, as depicted in
Figure 3.1. The score details for these problems are presented in Table 3.2.
Due to the smaller number of participating teams and missing contestant
score information in early IMOs, there are several blanks in Table 3.2.
2.5
Determining Values Existence Problems
Proving Quantitative Relationships
2
1.5
0.5
0
1–10 11–20 21–30 31–40 41–50 51–60 61–64
Full points 7.000 7.000 7.000 7.000 8.000 8.000 7.000 7.000
Note. Top five mean = Total score of the top five teams ÷ Total number of contestants from the top five teams,
6th–15th mean = Total score of the 6th–15th teams ÷ Total number of contestants from the 6th–15th teams,
16th–25th mean = Total score of the 16th–25th teams ÷ Total number of contestants from the 16th–25th teams.
Sequence Problems 207
Problem Number
Number of Problems
Sequence Problem 1, 4 2, 5 3, 6 in the First 64 IMOs
Determining values 2 1 0 4
Existence problems 0 0 2 5
Proving quantitative relationships 0 0 4 5
Total 2 1 6 14
From Table 3.2, it can be observed that in the sequence problems, the
average score of the top five teams is generally about 3 points higher than
the average score of the problem. However, in simpler problems, the dif-
ferences in average scores among the top five teams, 6th–15th teams, and
16th–25th teams are not significant, as seen in problems such as Problem
3.2 (IMO 36-4), Problem 3.3 (IMO 58-1), and Problem 3.4 (IMO 59-2).
From Table 3.2, it can also be observed that in the sequence problems
of the first 32 IMOs, the average score of the 16th–25th teams is close to or
below the average score of the problem, such as Problem 3.7 (IMO 23-3),
Problem 3.8 (IMO 32-6), and Problem 3.11 (IMO 26-6). However, after the
32nd IMO, the average score of the 16th–25th teams is about 1.5 points
208 IMO Problems, Theorems, and Methods: Algebra
higher than the average score of the problem. This phenomenon is due to
the smaller number of participating teams in early IMOs. It was not until
the 30th IMO in 1989 that the number of participating teams exceeded
50. Therefore, during this period, it was common to see situations where
the average score was close to or even higher than the average score of the
16th–25th teams.
Chapter 4
Inequality Problems
209
210 IMO Problems, Theorems, and Methods: Algebra
where f (x), g(x), p(x) are integrable functions on [a, b], with p(x) ≥ 0,
and f (x), g(x) are both increasing or decreasing on [a, b]. Chebyshev also
applied this inequality to probability theory.
The other pivotal moment was during the 1928 resignation speech of
renowned mathematician Godfrey Harold Hardy as the president of the
London Mathematical Society. He emphasized the elegance of inequalities
and his fondness for them, leading to a high proportion of inequality-related
papers in the society’s journal. He proposed that mastering elementary
inequalities is one of the necessary skills for the study of function theory.
Along with John Edensor Littlewood and George Pólya, Hardy co-authored
“Inequalities,” which cataloged some of the famous historical inequalities
and their origins, significantly impacting the field’s development.
Subsequently, the study of inequalities gradually formed into a
systematic scientific theory, significantly influencing other mathemati-
cal domains, and led to numerous publications on inequalities. Notably,
the Journal of Inequalities and Applications was founded in 1997, and
Mathematical Inequalities and Applications in 1998. These academic jour-
nals marked a new phase in the development of inequalities.
Due to the significant value and impact of inequalities, they frequently
appear in various mathematics competitions. For example, inequality prob-
lems in the IMO are often special cases of some famous inequalities, with their
difficulty significantly reduced compared to the original famous inequalities.
Moreover, these problems typically have special, simpler solutions.
In the first 64 IMOs, there had been a total of 33 inequality problems,
approximately accounting for 32.7% of all algebra problems. These prob-
lems can be primarily categorized into three types: (1) solving inequalities,
totaling five problems; (2) proving inequalities, totaling 25 problems; (3)
determining value ranges, totaling three problems. The statistical distribu-
tion of these three types of problems in the previous IMOs is presented in
Table 4.1.
In the 11th–20th, 31st–40th, and 41st–50th IMOs, the proportion of
inequality problems among all algebra problems exceeded 30%, especially in
the 41st–50th IMOs, where the proportion of inequality problems surpassed
50%, making it a focal topic of the IMO. It can also be observed from the
table that in early IMOs, there were problems involving solving inequalities,
but this type of problems gradually disappeared over the time. However,
the type of proving inequalities continued to thrive.
Remarkably, in recent IMOs, inequality problems with parameters
have emerged, and this trend is evident in some National Mathematical
Inequality Problems 211
Session
Solving inequalities 3 1 1 0 0 0 0 5
Proving inequalities 2 5 3 3 7 2 3 25
Determining value 0 1 0 1 1 0 0 3
ranges
Algebra problems 20 20 14 13 15 13 6 101
The percentage of 25.0% 35.0% 28.6% 30.8% 53.3% 15.4% 50.0% 32.7%
inequality problems
among the algebra
problems
It is important to note that for each problem, the solutions are fol-
lowed by information on the scores, including the number of contestants in
each score range, the average score, and the scores of the top five teams.
However, early IMOs often lacked information on contestant scores, so
the number of contestants in each score range only represents the counted
number of contestants, and some problems lack scores of the top five teams.
The third part provides a brief summary of this chapter.
Inequalities come in a wide variety, and while this chapter introduces
some methods for proving inequalities, ranging from straightforward basic
approaches to techniques that require manipulation of the inequalities,
this book is not a compendium of problem-solving methods, nor can it
enumerate all possible strategies. Often, the most practical and effective
methods are those discovered and understood by readers through their own
exploration. Therefore, when appreciating problems from past IMOs, read-
ers are encouraged to ponder the problems on their own before consulting
the solutions. It’s possible that the readers’ own solutions may even be
more elegant.
f (x) = (x − a1 )(x − a2 ) · · · (x − an ),
with its n real roots ordered as a1 < a2 < · · · < an . The number line is
divided into n + 1 intervals:
x1 x2 + x2 x3 + · · · + xn x1 ≤ x1 x2 + x2 x3 + x3 x4 + x1 (x4 + x5 + · · · + xn )
= x1 (x2 + x4 + x5 + · · · + xn ) + x3 (x2 + x4 )
≤ (x1 + x3 )(x2 + x4 + x5 + · · · + xn )
2
x1 + x2 + · · · + xn
≤ .
2
Example 4.2. Given an integer n ≥ 3, let a1 , a2 , . . . , a2n and b1 , b2 , . . . , b2n
be 4n non-negative real numbers satisfying
Multiplying these two inequalities and taking the square root, we obtain
6 2
a 1 + a2 + · · · + a6
3 ai ≤ (a1 + a3 + a5 )(a2 + a4 + a6 ) ≤ ,
i=1
2
6
and hence i=1 ai ≥ 12. The equality holds when ai = bi = 2 for
i = 1, 2, 3, 4, 5, 6.
For n ≥ 4, from Corollary 4.1.3,
2n 2n
(a1 + a3 + · · · + a2n−1 )2
≥ a1 a3 + a3 a5 + · · · + a2n−1 a1 ≥ bi = ai ,
4 i=1 i=1
2n
2n
(a2 + a4 + · · · + a2n )2
≥ a2 a4 + a4 a6 + · · · + a2n a2 ≥ bi = ai .
4 i=1 i=1
Multiplying these two inequalities and taking the square root, we get
2n
(a1 + a3 + · · · + a2n−1 )(a2 + a4 + · · · + a2n )
ai ≤
i=1
4
(a1 + a2 + · · · + a2n )2
≤ .
16
2n
Therefore i=1 ai ≥ 16. The equality holds when
This was the second problem in the 2020 Chinese High School Mathe-
matics League, illustrating a flexible application of the AM-GM inequality
216 IMO Problems, Theorems, and Methods: Algebra
a b c 3
+ + ≥ .
1 + b2 1 + c2 1 + a2 2
a b c a b c 3
+ + ≤ + + ≥ .
1 + b2 1 + c2 1 + a2 2b 2c 2a 2
a ab2 ab2 ab
= a − ≥ a − =a− .
1 + b2 1 + b2 2b 2
a b c ab + bc + ca 3
+ + ≥a+b+c− ≥ .
1 + b2 1 + c2 1 + a2 2 2
This method of initially altering the sign of the expression and then
applying the AM-GM inequality to the denominator or similar structures
without changing the direction of the inequality is also known as the
Cauchy Reverse Technique. Similarly, this technique can be used to solve
the following problem:
Given a, b, c, d > 0 with a + b + c + d = 4, prove that
a b c d
+ + + ≥ 2.
1 + b2 c 1 + c2 d 1 + d2 a 1 + a2 b
Corollary 4.4. For any real numbers a1 , a2 , . . . , an and positive real num-
bers b1 , b2 , . . . , bn ,
a21 a2 a2 (a1 + a2 + · · · + an )2
+ 2 + ···+ n ≥ .
b1 b2 bn b1 + b2 + · · · + bn
218 IMO Problems, Theorems, and Methods: Algebra
≥ (a1 + a2 + · · · + an )2 + (b1 + b2 + · · · + bn )2 .
and the equality holds if and only if the following n sequences are
proportional:
(a11 , a12 , . . . , a1m ), (a21 , a22 , . . . , a2m ), . . . , (an1 , an2 , . . . , anm ).
1
(ii) Given ai , bi > 0 for i = 1, 2, . . . , n, and positive p, q satisfying p + 1q = 1,
1 1
a1 b1 + a2 b2 + · · · + an bn ≤ (ap1 + ap2 + · · · + apn ) p (bq1 + bq2 + · · · + bqn ) q ,
where the equality holds if and only if api = λbqi for i = 1, 2, . . . , n.
Furthermore, there are several common corollaries to Hölder’s
inequality:
and the first equality holds if and only if ab ≤ 0 and the second equality
holds if and only if ab ≥ 0.
Theorem 4.1. For real numbers a1 , a2 , . . . , an ,
a1 ≤ a2 ≤ · · · ≤ an and b1 ≤ b2 ≤ · · · ≤ bn ,
it holds that
2(a + b + c + d) ≥ a2 + 3 + b2 + 3 + c2 + 3 + d2 + 3.
1 a2 − 1 a
≥ √ = 0,
cyc
4 a
cyc 2a + a2 + 3
where cyc denotes the cyclic sum, as in this case cyc a = a + b + c + d.
This method of using Chebyshev’s inequality to prove other inequalities
is also known as the Chebyshev Associate Technique.
Hence, the inequality (*) holds, and therefore, the original inequality holds.
1 1 1
n
r n
r n
r
r
(ai + bi ) ≤ ari + bri ,
i=1 i=1 i=1
(1 + a1 )(1 + a2 ) · · · (1 + an ) > 1 + a1 + a2 + · · · + an .
Let f (x) be a continuous function defined on the interval [a, b]. If for
any x1 , x2 ∈ [a, b],
x1 + x2 f (x1 ) + f (x2 )
f ≤ (or ≥) ,
2 2
then f (x) is termed as a convex function (or concave function) on [a, b].
If the second derivative f (x) exists and f (x) ≥ 0(or ≤ 0) for x ∈ [a, b],
then f (x) is a convex function (or concave function) on [a, b].
λ1 , λ2 , . . . , λn satisfying λ1 + λ2 + · · · + λn = 1,
f (λ1 x1 + λ2 x2 + · · · + λn xn )
≤ (or ≥)λ1 f (x1 ) + λ2 f (x2 ) + · · · + λn f (xn ).
which is equivalent to
(i) x3 − cyc x2 (y + z) + 3xyz ≥ 0;
cyc
(ii) ( cyc x)3 − 4 cyc x cyc yz + 9xyz ≥ 0.
Example 4.8. Let x, y, z be positive real numbers such that x+y+z = xyz.
Prove that
x2 + y 2 + z 2 − 2(xy + yz + zx) + 9 ≥ 0.
Inequality Problems 225
xn ≥ x1 + (x2 + · · · + xn−1 )
≥ x1 + (C · 22 + · · · + C · 2n−1 )
≥ C(22 + 22 + 23 + · · · + 2n−1 )
= C · 2n .
x=√ r sin α and x =√r cos α. When conditions or conclusions involve forms
like r2 + x2 and x2 − r2 , consider substitutions like x = r tan α and
x = r sec α. When making substitutions, it is important to note that the
range of values for α is determined by the range of values for the original
variable x.
√
Example 4.15. Given 0◦ ≤ α ≤ 90◦ , prove that 2 ≤ 5 − 4 sin α+sin α ≤
9
4.
√ 2
Proof. Let x = 5√− 4 sin α. Then sin α = 5−x 4 . Since 0 ≤ sin α ≤ 1, it
follows that 1 ≤ x ≤ 5. Define
√
y = 5 − 4 sin α + sin α
5 − x2
= x+
4
1 9
= − (x − 2)2 + .
4 4
√
Given 1 ≤ x ≤ 5, it follows that 2 ≤ y ≤ 94 .
Example 4.16. Let x, y, z be three real numbers, not all zero. Find the
maximum value of x2xy+2yz
+y 2 +z 2 .
terms can facilitate the use of common inequalities such as the AM-GM
inequality, Cauchy–Schwarz inequality, etc.
YZ = a2 + ab + b2 , YW = a2 − ac + c2 , WZ = b2 − bc + c2 .
prove that
(x1 y1 + x2 y2 + · · · + xn yn − 1)2
≥ (x21 + x22 + · · · + x2n − 1)(y12 + y22 + · · · + yn2 − 1).
4(x1 y1 + x2 y2 + · · · + xn yn − 1)2
− 4(x21 + x22 + · · · + x2n − 1)(y12 + y22 + · · · + yn2 − 1) ≥ 0.
there exists a tangent line y = l(x) to f (x) at some point x0 such that the
graph of the function f (x) lies above the tangent line, and l(a) + l(b) +
l(c) ≥ 0, then
1√ 1
Define the function f (x) = 1− x
, and its tangent line at x = 3 is
√
9+6 3 3
y= 4 x + 4. When 0 < x < 1,
√
1 9+6 3 3
√ ≥ x+
1− x 4 4
√ √ √
is equivalent to ( 3x − 1)2 ((3 + 2 3) x + 1) ≥ 0. Therefore,
√ √
1 1 1 9+6 3 9 9+3 3
√ + √ + √ ≥ (a + b + c) + = .
1− a 1− b 1− c 4 4 2
1 1 1 1
+ + ≤ .
5a2 − 4a + 11 5b2 − 4b + 11 5c2 − 4c + 11 4
3 2
fa (a, b, c) = 3 − k(b + c) = 0,
fb (a, b, c) = 4 − k(a + c) = 0,
fc (a, b, c) = 5 − k(a + b) = 0,
3 4 5
yielding k = b+c = c+a = a+b . From ab + bc + ca = 11, we get a = 3,
b = 2, and c = 1, which should be the equality conditions for 3a + 4b + 5c
to attains its minimum value.
We return to the original problem. From c = 11−ab
a+b ,
55 − 5ab
3a + 4b + 5c = 3a + 4b +
a+b
55 + 2ab + 3a2 + 4b2 − 22(a + b)
= + 22
a+b
(a + b − 5)2 + 2(a − 3)2 + 3(b − 2)2
= + 22
a+b
≥ 22.
(i)Sa ≥ 0, Sb ≥ 0, and Sc ≥ 0.
(ii)a ≥ b ≥ c, Sb ≥ 0, Sb + Sa ≥ 0, and Sb + Sc ≥ 0.
(iii)a ≥ b ≥ c, Sa ≥ 0, Sc ≥ 0, Sa + 2Sb ≥ 0, and Sc + 2Sb ≥ 0.
(iv) a ≥ b ≥ c, Sb ≥ 0, Sc ≥ 0, and a2 Sb + b2 Sa ≥ 0.
(v) a ≥ b ≥ c are the side lengths of a triangle, and Sa ≥ 0, Sb ≥ 0, and
b2 Sb + c2 Sc ≥ 0.
(vi) Sa + Sb + Sc ≥ 0 and Sa Sb + Sb Sc + Sc Sa ≥ 0.
234 IMO Problems, Theorems, and Methods: Algebra
The proofs for these conditions are not difficult, and conditions (ii) and
(iv) are illustrated here. For condition (ii), it is easy to see that (c − a)2 ≥
(a − b)2 + (b − c)2 , and thus,
a−c
For condition (iv), it is evident that b−c ≥ ab , from which,
1 1 1 9
+ + ≥ .
(x + y)2 (y + z)2 (z + x)2 4(xy + yz + zx)
1
By the pqr form of the AM-GM inequality, r ≤ 27 , and hence
√
1 9 1 r
q≤ + r< + .
4 4 4 2
(x + y + z)(x3 + y 3 + z 3 ) ≥ (x2 + y 2 + z 2 )2
(x + y + z)2
≥ (x2 + y 2 + z 2 ) .
3
Similarly, the remaining three expressions can be derived, and adding these
four inequalities together suffices.
scored 1 point, and 3 contestants scored 0 point. The average score of this problem is 2.600,
indicating that it had a certain level of difficulty.
Among the top five teams in the team scores, the Czechoslovakia team achieved a total
score of 257 points, the Hungary team achieved a total score of 248 points, the Romania
team achieved a total score of 248 points, the Bulgaria team achieved a total score of 175
points, and the German Democratic Republic team achieved a total score of 38 points.
The gold medal cutoff for this IMO was set at 40 points (with 4 contestants earning gold
medals), the silver medal cutoff was 37 points (with 4 contestants earning silver medals), and
the bronze medal cutoff was 33 points (with 4 contestants earning bronze medals).
In this IMO, no contestant achieved a perfect score of 44 points.
Solution. From
3 − x ≥ 0,
x + 1 ≥ 0,
√ √
we obtain −1 ≤ x ≤ 3. When −1 ≤ x ≤ 3, the value of 3−x− x+1
decreases as x increases. Solve the equation
√ √
3 − x − x + 1 = 12 , (1)
7
√
4 − 2x = x + 1,
64x2 − 128x + 33 = 0,
√ √
yielding x = 8±8 31 . After verification, x = 8−8 31 is a solution to (1). That
√ √ √
is, when x = 8−8 31 , the value of 3 − x − x + 1 equals 12 . Therefore, the
√
solution to the inequality is −1 ≤ x < 8−8 31 .
Score Situation This particular problem saw the following distribution of scores among
contestants: 7 contestants scored 6 points, 3 contestants scored 5 points, 2 contestants
scored 4 points, 3 contestants scored 3 points, 1 contestant scored 2 points, 1 contestant
scored 1 point, and no contestant scored 0 point. The average score of this problem is 4.529,
indicating that it was simple.
Among the top five teams in the team scores, the Hungary team achieved a total score
of 289 points, the Soviet Union team achieved a total score of 263 points, the Romania team
Inequality Problems 239
achieved a total score of 257 points, the Czechoslovakia team achieved a total score of 212
points, and the Poland team achieved a total score of 212 points.
The gold medal cutoff for this IMO was set at 41 points (with 4 contestants earning gold
medals), the silver medal cutoff was 34 points (with 12 contestants earning silver medals),
and the bronze medal cutoff was 29 points (with 15 contestants earning bronze medals).
In this IMO, only one contestant achieved a perfect score of 46 points, namely Iosif
Bernstein from the Soviet Union.
When 34 π < x < 54 π, we obtain 2 cos x < 0 ≤ 2|sin x|. When 0 ≤ x < π
4
7
or 4π< x ≤ 2π, there holds 2 cos x > 2|sin x| ≥ 0.
The right inequality,
√ √ √ √ √
| 1 + sin 2x − 1 − sin 2x| ≤ max{ 1 + sin 2x, 1 − sin 2x} ≤ 2,
π 7π
is obvious. Therefore, the solution to the inequality is 4 ≤x≤ 4 .
Note. This problem is the same as the 6th problem in 1967 British Math-
ematical Olympiad Round 1.
Score Situation This particular problem saw the following distribution of scores among
contestants: 37 contestants scored 4 points, 10 contestants scored 3 points, 14 contestants
scored 2 points, 13 contestants scored 1 point, and 6 contestants scored 0 point. The average
score of this problem is 2.738, indicating that it had a certain level of difficulty.
240 IMO Problems, Theorems, and Methods: Algebra
Among the top five teams in the team scores, the scores of this problem are as follows:
the Soviet Union team scored 25 points (with a total team score of 281 points), the Hungary
team scored 29 points (with a total team score of 244 points), the Romania team scored 27
points (with a total team score of 222 points), the Poland team scored 28 points (with a
total team score of 178 points), and the German Democratic Republic team scored 23 points
(with a total team score of 175 points).
The gold medal cutoff for this IMO was set at 38 points (with 8 contestants earning gold
medals), the silver medal cutoff was 30 points (with 12 contestants earning silver medals),
and the bronze medal cutoff was 20 points (with 17 contestants earning bronze medals).
In this IMO, only two contestants achieved a perfect score of 40 points, namely László
Lovász from Hungary and Pavel Bleher from the Soviet Union.
⎧ 2
⎪(x1 − x3 x5 )(x22 − x3 x5 ) ≤ 0,
⎪
⎪ 2
⎪
⎪ 2
⎨(x2 − x4 x1 )(x3 − x4 x1 ) ≤ 0,
⎪
(x23 − x5 x2 )(x24 − x5 x2 ) ≤ 0,
⎪
⎪
⎪
⎪(x24 − x1 x3 )(x25 − x1 x3 ) ≤ 0,
⎪
⎪
⎩ 2
(x5 − x2 x4 )(x21 − x2 x4 ) ≤ 0,
5
(x2i − xi+2 xi+4 )(x2i+1 − xi+2 xi+4 )
i=1
5
= (x2i x2i+1 − x2i xi+2 xi+4 − x2i+1 xi+2 xi+4 + x2i+2 x2i+4 )
i=1
Inequality Problems 241
5
1 2
= (x x2 + x2i+1 x2i+2 + x2i x2i+2 + x2i+4 x2i+1
2 i=1 i+4 i
Since (x2i + x2i+1 )(xi+2 − xi+4 )2 ≥ 0 for i ∈ {1, 2, 3, 4, 5}, the above
inequality holds when x1 = x2 = x3 = x4 = x5 .
Therefore, the solution to the system is x1 = x2 = x3 = x4 = x5 .
Solution 2. Let (x1 , x2 , x3 , x4 , x5 ) be a solution to the system of inequal-
ities. Consider the case where
x1 ≤ x2 = x3 = x5 ≤ x4 .
x1 = x2 = x3 = x4 = x5 . (1)
Among the top five teams in the team scores, the scores of this problem are as follows:
the Soviet Union team scored 48 points (with a total team score of 270 points), the Hungary
team scored 52 points (with a total team score of 263 points), the German Democratic
Republic team scored 43 points (with a total team score of 239 points), the Romania team
scored 34 points (with a total team score of 208 points), and the United Kingdom team
scored 30 points (with a total team score of 179 points).
The gold medal cutoff for this IMO was set at 40 points (with 8 contestants earning gold
medals), the silver medal cutoff was 30 points (with 16 contestants earning silver medals),
and the bronze medal cutoff was 19 points (with 30 contestants earning bronze medals).
In this IMO, a total of eight contestants achieved a perfect score of 40 points.
Problem 4.5 (IMO 29-4, proposed by Ireland). Show that the set of
real numbers x which satisfy the inequality
70
k 5
≥
x−k 4
k=1
1 2 3 70
Solution. Consider the function y = x−1 + x−2 + x−3 + · · · + x−70 and its
5
graph intersecting with the line y = 4 . Let the solutions of the equation
1 2 3 70 5
+ + + ···+ =
x−1 x−2 x−3 x − 70 4
be denoted by x1 < x2 < · · · < x70 . The solution set for the inequality is
given by
1 2 3 70
The graphs of the function y = x−1 + x−2 + x−3 +···+ x−70 and the line
5
y = 4 are illustrated in Figure 4.1.
Inequality Problems 243
1 2 3 70
Figure 4.1 The Graphs of the Function y = x−1
+ x−2
+ x−3
+···+ x−70
and
5
the Line y = 4
1 2 m
+ + ···+ ≥ r (r > 0)
x−1 x−2 x−m
m(m+1)
is 2r .
Score Situation This particular problem saw the following distribution of scores among
contestants: 66 contestants scored 7 points, 6 contestants scored 6 points, 3 contestants
scored 5 points, 3 contestants scored 4 points, 17 contestants scored 3 points, 12 contestants
scored 2 points, 25 contestants scored 1 point, and 136 contestants scored 0 point. The
average score of this problem is 2.332, indicating that it had a certain level of difficulty.
Among the top five teams in the team scores, the scores of this problem are as follows:
the Soviet Union team scored 38 points (with a total team score of 217 points), the China
team scored 42 points (with a total team score of 201 points), the Romania team scored
42 points (with a total team score of 201 points), the Germany team scored 31 points (with
a total team score of 174 points), and the Vietnam team scored 40 points (with a total team
score of 166 points).
The gold medal cutoff for this IMO was set at 32 points (with 17 contestants earning gold
medals), the silver medal cutoff was 23 points (with 48 contestants earning silver medals),
and the bronze medal cutoff was 14 points (with 65 contestants earning bronze medals).
In this IMO, a total of five contestants achieved a perfect score of 42 points.
√
a2 + b2 + c2 ≥ 4 3S.
√
a2 + b2 + c2 − 4 3S
√
= 2a2 + 2c2 − 2ac cos B − 2 3ac sin B
π
= 2 a2 + c2 − 2ac sin B +
6
≥ 2(a2 + c2 − 2ac)
= 2(a − c)2 ≥ 0.
Thus, the inequality holds, and the equality holds if and only if
a = b = c.
Thus, the inequality holds, and the equality holds if and only if
a = b = c.
Note. This problem has a variety of solutions, such as using the AM-GM
inequality, or discussing cases based on the size of angle A. Additionally,
this inequality is known as Weitzenböck’s inequality, and it has several
generalizations:
√
a2 + b2 + c2 ≥ 4 3S + (a − b)2 + (b − c)2 + (c − a)2 .
246 IMO Problems, Theorems, and Methods: Algebra
≥ (a − b)2 + (b − c)2 .
Therefore,
√ √
a2 + b2 + c2 − 4 3S = 2a2 + 2c2 − 2ac cos B − 2 3ac sin B
π
= 2 a2 + c2 − 2ac sin B +
6
≥ 2(a2 + c2 − 2ac) = 2(a − c)2
x y z
with equality if and only if β+γ = γ+α = α+β .
Inequality Problems 247
Score Situation This particular problem saw the following distribution of scores among
contestants: 3 contestants scored 7 points, no contestant scored 6 points, 1 contestant
scored 5 points, no contestant scored 4 points, no contestant scored 3 points, 1 contestant
scored 2 points, no contestant scored 1 point, and 4 contestants scored 0 point. The average
score of this problem is 3.111, indicating that it was relatively straightforward.
Among the top five teams in the team scores, the Hungary team achieved a total score of
270 points, the Poland team achieved a total score of 203 points, the Romania team achieved
a total score of 197 points, the Czechoslovakia team achieved a total score of 159 points,
and the German Democratic Republic team achieved a total score of 146 points.
The gold medal cutoff for this IMO was set at 37 points (with 3 contestants earning gold
medals), the silver medal cutoff was 34 points (with 4 contestants earning silver medals), and
the bronze medal cutoff was 30 points (with 4 contestants earning bronze medals).
In this IMO, only one contestant achieved a perfect score of 40 points, namely Béla
Bollobás from Hungary.
a2 (b + c − a) + b2 (c + a − b) + c2 (a + b − c) ≤ 3abc.
2x(y + z)2 + 2y(z + x)2 + 2z(x + y)2 ≤ 3(x + y)(y + z)(z + x),
6xyz ≤ x2 y + y 2 z + z 2 x + x2 z + y 2 x + z 2 y. (1)
x2 y + y 2 z + z 2 x + x2 z + y 2 x + z 2 y
≥ 6 6 x2 y · y 2 z · z 2 x · x2 z · y 2 x · z 2 y
= 6xyz,
A
x
x
E
F
z
y
B y z C
D
a + b b + c c + a a 3 + b 3 + c3
2< + + − ≤ 3.
c a b abc
Score Situation This particular problem saw the following distribution of scores among
contestants: 7 contestants scored 7 points, no contestant scored 6 points, 1 contestant
scored 5 points, no contestant scored 4 points, no contestant scored 3 points, no contestant
scored 2 points, 2 contestants scored 1 point, and 7 contestants scored 0 point. The average
score of this problem is 3.294, indicating that it was relatively straightforward.
Among the top five teams in the team scores, the scores of this problem are as follows:
the Soviet Union team scored 54 points (with a total team score of 269 points), the Hungary
team scored 37 points (with a total team score of 253 points), the Romania team scored
35 points (with a total team score of 213 points), the Poland team scored 33 points (with a
total team score of 209 points), and the Bulgaria team scored 29 points (with a total team
score of 198 points).
The gold medal cutoff for this IMO was set at 38 points (with 7 contestants earning gold
medals), the silver medal cutoff was 31 points (with 9 contestants earning silver medals), and
the bronze medal cutoff was 27 points (with 19 contestants earning bronze medals).
In this IMO, only one contestant achieved a perfect score of 42 points, namely David
Bernstein from the Soviet Union.
8 1 1
≤ 2 +
(x1 + x2 )(y1 + y2 ) − (z1 + z2 )2 x1 y1 − z1 x2 y2 − z22
Proof 1. From x1 y1 > z12 ≥ 0 and x1 > 0, it follows that y1 > 0. Similarly,
y2 > 0. Therefore,
√
x1 y1 z22 + x2 y2 z12 ≥ 2z1 z2 x1 x2 y1 y2 ,
√ √
( x1 x2 y1 y2 − z1 z2 )2 = x1 x2 y1 y2 − 2z1 z2 x1 x2 y1 y2 + z12 z22
Also
which implies
8 2
≤
(x1 + x2 )(y1 + y2 ) − (z1 + z2 )2 (x1 y1 − z12 )(x2 y2 − z22 )
1 1
≤ 2 + .
x1 y1 − z1 x2 y2 − z22
The equality holds if and only if x1 = x2 , y1 = y2 , and z1 = z2 .
Proof 2. Let A1 = x1 y1 − z12 and A2 = x2 y2 − z22 , where A1 , A2 > 0. The
original inequality becomes
1 1
+ · (A1 + A2 + x1 y2 + x2 y1 − 2z1 z2 ) ≥ 8.
A1 A2
Also
1 1
+ · (A1 + A2 + x1 y2 + x2 y1 − 2z1 z2 )
A1 A2
1 1 √
≥ + · (A1 + A2 + 2 x1 y2 x2 y1 − 2z1 z2 )
A1 A2
252 IMO Problems, Theorems, and Methods: Algebra
1 1
= + · (A1 + A2 + 2 (A1 + z12 )(A2 + z22 ) − 2z1 z2 )
A1 A2
1 1
≥ + · (A1 + A2 + 2( A1 A2 + z1 z2 ) − 2z1 z2 )
A1 A2
(using the Cauchy–Schwarz inequality)
1 1
= + · (A1 + A2 + 2 A1 A2 )
A1 A2
1 1 A1 + A2
= + · (A1 + A2 ) + 2 √
A1 A2 A1 A2
2 √
1 1 2 A1 A2
≥ A1 + A2 + 2 √
A1 A2 A1 A2
= 4 + 2 · 2 = 8.
Hence, the original inequality holds. The equality holds if and only if x1 =
x2 , y1 = y2 , and z1 = z2 .
Note. The A1 and A2 in the problem can be represented as the determi-
nants of the following matrices M 1 and M 2 respectively, namely
x1 z1 x2 z2
M1 = and M 2 = ,
z1 y1 z2 y2
and the inequality to be proved can be written as
8 1 1
≤ + .
det(M 1 + M 2 ) det M 1 det M 2
If M 1 and M 2 are n×n positive definite matrices, then the above inequality
still holds.
Furthermore, there are several similar problems:
• (Austrian–Polish Mathematical Competition 1998, Problem 1).
Let x1 , x2 , y1 , y2 be real numbers such that x21 + x22 ≤ 1. Prove that
(x1 y1 + x2 y2 − 1)2 ≥ (x21 + x22 − 1)(y12 + y22 − 1).
• (From the Chinese journal High-School Mathematics, 1992(03):
26–27). Let a1 , a2 , b1 , b2 , c1 , c2 be real numbers such that a1 a2 > 0,
a1 c1 ≥ b21 , and a2 c2 ≥ b22 . Prove that
(a1 + a2 )(c1 + c2 ) ≥ (b1 + b2 )2 + 2 (a1 c1 − b21 )(a2 c2 − b22 ).
Inequality Problems 253
Score Situation This particular problem saw the following distribution of scores among con-
testants: 7 contestants scored 8 points, 2 contestants scored 7 points, 3 contestants scored
6 points, 9 contestants scored 5 points, 13 contestants scored 4 points, 13 contestants scored
3 points, 17 contestants scored 2 points, 15 contestants scored 1 point, and 33 contestants
scored 0 point. The average score of this problem is 2.438, indicating that it had a certain
level of difficulty.
Among the top five teams in the team scores, the scores of this problem are as follows:
the Hungary team scored 30 points (with a total team score of 247 points), the German
Democratic Republic team scored 29 points (with a total team score of 240 points), the
Soviet Union team scored 29 points (with a total team score of 231 points), the Romania
team scored 33 points (with a total team score of 219 points), and the United Kingdom team
scored 23 points (with a total team score of 193 points).
The gold medal cutoff for this IMO was set at 40 points (with 3 contestants earning gold
medals), the silver medal cutoff was 30 points (with 20 contestants earning silver medals),
and the bronze medal cutoff was 24 points (with 21 contestants earning bronze medals).
In this IMO, only three contestants achieved a perfect score of 40 points, namely Tibor
Fiala from Hungary, Vladimir Drinfeld from the Soviet Union, and Simon Phillips Norton from
the United Kingdom.
If n = 3, then
If n = 5, then
and similarly, the sum of the last two terms is also non-negative.
Additionally, the third term is non-negative, so A5 ≥ 0.
If n is odd and n ≥ 7, let a1 = a2 = a3 < a4 < a5 = a6 = · · · = an , then
+ (a5 − aj ),
j=5
Inequality Problems 255
a1 = a2 , a3 = a4 = a5
Score Situation This particular problem saw the following distribution of scores among con-
testants: 4 contestants scored 5 points, 2 contestants scored 4 points, 3 contestants scored
3 points, 11 contestants scored 2 points, 6 contestants scored 1 point, and 2 contestants
scored 0 point. The average score of this problem is 2.321, indicating that it had a certain
level of difficulty.
256 IMO Problems, Theorems, and Methods: Algebra
Among the top five teams in the team scores, the scores of this problem are as follows:
the Hungary team scored 35 points (with a total team score of 255 points), the Soviet Union
team scored 34 points (with a total team score of 205 points), the German Democratic
Republic team scored 20 points (with a total team score of 142 points), the Poland team
scored 29 points (with a total team score of 118 points), the Romania team scored 22 points
(with a total team score of 110 points), and the United Kingdom team scored 21 points (with
a total team score of 110 points).
The gold medal cutoff for this IMO was set at 35 points (with 7 contestants earning gold
medals), the silver medal cutoff was 23 points (with 12 contestants earning silver medals),
and the bronze medal cutoff was 11 points (with 29 contestants earning bronze medals).
In this IMO, only one contestant achieved a perfect score of 42 points, namely Imre
Ruzsa from Hungary.
x1 ≥ x2 ≥ · · · ≥ xn and y1 ≥ y2 ≥ · · · ≥ yn .
n
n
(xi − yi )2 ≤ (xi − zi )2 .
i=1 i=1
n
n
n
n
n
n
x2i −2 xi yi + yi2 ≤ x2i −2 xi zi + zi2
i=1 i=1 i=1 i=1 i=1 i=1
n
n
n
n
⇔ xi zi ≤ xi yi yi2 = zi2 ,
i=1 i=1 i=1 i=1
which is true according to the “mixed order sum” and “same order sum”
from the rearrangement inequality. Therefore, the original inequality is
valid.
Score Situation This particular problem saw the following distribution of scores among
contestants: 76 contestants scored 6 points, 12 contestants scored 5 points, 11 contestants
scored 4 points, 5 contestants scored 3 points, 9 contestants scored 2 points, 12 contestants
Inequality Problems 257
scored 1 point, and 3 contestants scored 0 point. The average score of this problem is 4.727,
indicating that it was simple.
Among the top five teams in the team scores, the scores of this problem are as follows:
the Hungary team scored 47 points (with a total team score of 258 points), the German
Democratic Republic team scored 46 points (with a total team score of 249 points), the
United States team scored 43 points (with a total team score of 247 points), the Soviet Union
team scored 48 points (with a total team score of 246 points), and the United Kingdom team
scored 45 points (with a total team score of 241 points).
The gold medal cutoff for this IMO was set at 38 points (with 8 contestants earning gold
medals), the silver medal cutoff was 32 points (with 25 contestants earning silver medals),
and the bronze medal cutoff was 23 points (with 36 contestants earning bronze medals).
In this IMO, a total of six contestants achieved a perfect score of 40 points.
a2 + b 2 ≤ 2 and A2 + B 2 ≤ 1.
so it follows that
π
f θ+ = 1 + a sin θ − b cos θ + A cos 2θ + B sin 2θ ≥ 0, (2)
2
f (θ + π) = 1 + a cos θ + b sin θ − A cos 2θ − B sin 2θ ≥ 0. (3)
2 + (a − b) sin θ − (a + b) cos θ ≥ 0,
2+ (a − b)2 + (a + b)2 sin(θ − ϕ1 ) ≥ 0,
258 IMO Problems, Theorems, and Methods: Algebra
a−b a+b
where cos ϕ1 = √ and sin ϕ1 = √ . Therefore,
(a−b)2 +(a+b)2 (a−b)2 +(a+b)2
2− (a − b)2 + (a + b)2 ≥ 0,
which implies a2 + b2 ≤ 2.
By (1) and (3),
1 − A cos 2θ − B sin 2θ ≥ 0,
1− A2 + B 2 cos(2θ − ϕ2 ) ≥ 0,
a2 a3 an 1 1 1
a1 + + + ···+ 2 ≥ 1 + + + ···+ .
4 9 n 2 3 n
n
n
n
n
ak bk k 1
≥ ≥ = .
k2 k 2 k 2 k
k=1 k=1 k=1 k=1
a1 + a2 + · · · + ak ≥ 1 + 2 + · · · + k.
Consequently,
a1 − 1 a2 − 2 an − n
2
+ 2
+ ··· +
1 2 n2
a1 + a2 − (1 + 2) a3 − 3 an − n
≥ 2
+ 2
+ ···+
2 3 n2
a1 + a2 + a3 − (1 + 2 + 3) a4 − 4 an − n
≥ 2
+ 2
+ ···+
3 4 n2
........................................................................
a1 + a2 + · · · + an − (1 + 2 + · · · + n)
≥ ≥ 0,
n2
a2 a3 an 1 1
and thus, a1 + 4 + 9 + ···+ n2 ≥1+ 2 + 3 + · · · + n1 .
260 IMO Problems, Theorems, and Methods: Algebra
Score Situation This particular problem saw the following distribution of scores among
contestants: 39 contestants scored 6 points, 1 contestant scored 5 points, no contestant
scored 4 points, 2 contestants scored 3 points, no contestant scored 2 points, 3 contestants
scored 1 point, and 3 contestants scored 0 point. The average score of this problem is 5.167,
indicating that it was simple.
Inequality Problems 261
Among the top five teams in the team scores, the scores of this problem are as follows:
the Romania team scored 48 points (with a total team score of 237 points), the United States
team scored 48 points (with a total team score of 225 points), the United Kingdom team
scored 45 points (with a total team score of 201 points), the Vietnam team scored 45 points
(with a total team score of 200 points), and the Czechoslovakia team scored 45 points (with
a total team score of 195 points).
The gold medal cutoff for this IMO was set at 35 points (with 5 contestants earning gold
medals), the silver medal cutoff was 27 points (with 20 contestants earning silver medals),
and the bronze medal cutoff was 22 points (with 38 contestants earning bronze medals).
In this IMO, only one contestant achieved a perfect score of 40 points, namely Mark
Kleiman from the United States.
a3 b + b3 c + c3 a ≥ abc(a + b + c)
Score Situation This particular problem saw the following distribution of scores among
contestants: 6 contestants scored 7 points, no contestant scored 6 points, 1 contestant
scored 5 points, 7 contestants scored 4 points, 2 contestants scored 3 points, no contestant
scored 2 points, 4 contestants scored 1 point, and 20 contestants scored 0 point. The average
score of this problem is 2.125, indicating that it had a certain level of difficulty.
Among the top five teams in the team scores, the Germany team achieved a total score
of 212 points, the United States team achieved a total score of 171 points, the Hungary team
achieved a total score of 170 points, the Soviet Union team achieved a total score of 169
points, and the Romania team achieved a total score of 161 points.
The gold medal cutoff for this IMO was set at 38 points (with 9 contestants earning gold
medals), the silver medal cutoff was 26 points (with 27 contestants earning silver medals),
and the bronze medal cutoff was 15 points (with 57 contestants earning bronze medals).
In this IMO, a total of four contestants achieved a perfect score of 42 points.
1
Let z = 3 − k, where 0 ≤ k ≤ 13 . Hence x + y = 2
3 + k. Further,
xy + yz + zx − 2xyz = xy(1 − 2z) + z(x + y)
1 1 2
= xy + 2k + −k +k
3 3 3
(x + y)2 1 1 2
≤ + 2k + −k +k
4 3 3 3
2
1 k 1 1 2
= + + 2k + −k +k
3 2 3 3 3
1 3 1 2 7
= k − k +
2 4 27
1 2 1 7 7
= k k− + ≤ ,
2 2 27 27
where the equality holds when k = 0 and x = y, i.e., when x = y = z = 13 .
Proof 2. Since 0 ≤ x, y, z ≤ 1,
xy + yz + zx − 2xyz ≥ xy + yz + zx − yz − zx = xy ≥ 0.
Also,
(1 − 2x)(1 − 2y)(1 − 2z) = 1 − 2(x + y + z) + 4(xy + yz + zx) − 8xyz
= −1 + 4(xy + yz + zx − 2xyz).
When 1 − 2x, 1 − 2y, and 1 − 2z are all non-negative real numbers,
3
(1 − 2x) + (1 − 2y) + (1 − 2z) 1
(1 − 2x)(1 − 2y)(1 − 2z) ≤ = . (1)
3 27
Since x + y + z = 1, at most one of 1 − 2x, 1 − 2y, and 1 − 2z is negative.
In this case, inequality (1) still holds. Therefore,
1
−1 + 4(xy + yz + zx − 2xyz) ≤ ,
27
7
implying xy + yz + zx − 2xyz ≤ 27 .
Note. The original inequality can also be proven by using the adjustment
method or pqr method. Furthermore, if xi (i = 1, 2, . . . , n) are non-negative
real numbers satisfying x1 + x2 + · · · + xn = 1, then
n−1 1 1 1
0≤ xi xj − xi xj xk ≤ 1− 2+ .
n−2 6 n n
1≤i<j≤n 1≤i<j<k≤n
264 IMO Problems, Theorems, and Methods: Algebra
|x3 + y 3 + z 3 − 3xyz| ≤ 1.
9
a3 + b3 + c3 + 4abc ≤ .
32
• (British Mathematical Olympiad 2002, 1st Round, Problem 3).
Let x, y, z be positive real numbers such that x2 + y 2 + z 2 = 1. Prove
that
1
x2 yz + xy 2 z + xyz 2 ≤ .
3
• (United States of America Mathematical Olympiad 2001, Prob-
lem 3). Let a, b, c be non-negative real numbers such that a2 + b2 + c2 +
abc = 4. Show that
0 ≤ ab + bc + ca − abc ≤ 2.
Score Situation This particular problem saw the following distribution of scores among
contestants: 86 contestants scored 7 points, 10 contestants scored 6 points, 7 contestants
scored 5 points, 9 contestants scored 4 points, 23 contestants scored 3 points, 35 contestants
scored 2 points, 3 contestants scored 1 point, and 19 contestants scored 0 point. The average
score of this problem is 4.557, indicating that it was simple.
Among the top five teams in the team scores, the scores of this problem are as follows:
the Soviet Union team scored 42 points (with a total team score of 235 points), the Bulgaria
team scored 42 points (with a total team score of 203 points), the Romania team scored 42
points (with a total team score of 199 points), the Hungary team scored 37 points (with a
total team score of 195 points), and the United States team scored 42 points (with a total
team score of 195 points).
The gold medal cutoff for this IMO was set at 40 points (with 14 contestants earning gold
medals), the silver medal cutoff was 26 points (with 35 contestants earning silver medals),
and the bronze medal cutoff was 17 points (with 49 contestants earning bronze medals).
In this IMO, a total of eight contestants achieved a perfect score of 42 points.
n n
(e1 , e2 , . . . , en ) such that i=1 ei xi =
i=1 ei xi , let ai = ei − ei for
i = 1, 2, . . . , n. Then ai are integers, not all zero, and |ai | ≤ k − 1, yielding
√
(k − 1) n
|a1 x1 + a2 x2 + · · · + an xn | = 0 < .
kn − 1
Thus, the original inequality holds.
If e1 x1 + e2 x2 + · · · + en xn are all distinct, then the set
n
A= ei xi |ei ∈ {0, 1, 2, . . . , k − 1}
i=1
n
contains k distinct numbers. By the Cauchy–Schwarz inequality,
n n n
√ √
0≤ ei xi ≤ (k − 1) xi ≤ (k − 1) x2i · n = (k − 1) n,
i=1 i=1 i=1
Score Situation This particular problem saw the following distribution of scores among
contestants: 59 contestants scored 7 points, 6 contestants scored 6 points, 2 contestants
scored 5 points, no contestant scored 4 points, 1 contestant scored 3 points, 5 contestants
Inequality Problems 267
scored 2 points, 39 contestants scored 1 point, and 125 contestants scored 0 point. The
average score of this problem is 2.156, indicating that it had a certain level of difficulty.
Among the top five teams in the team scores, the scores of this problem are as follows:
the Romania team scored 40 points (with a total team score of 250 points), the Germany
team scored 42 points (with a total team score of 248 points), the Soviet Union team scored
28 points (with a total team score of 235 points), the German Democratic Republic team
scored 42 points (with a total team score of 231 points), and the United States team scored
35 points (with a total team score of 220 points).
The gold medal cutoff for this IMO was set at 42 points (with 22 contestants earning gold
medals), the silver medal cutoff was 32 points (with 42 contestants earning silver medals),
and the bronze medal cutoff was 18 points (with 56 contestants earning bronze medals).
In this IMO, a total of 22 contestants achieved a perfect score of 42 points.
ai + am+1−i ≥ n + 1. (1)
2(a1 + a2 + · · · + am )
= (a1 + am ) + (a2 + am−1 ) + · · · + (am + a1 ) ≥ m(n + 1),
a1 +a2 +···+am n+1
implying m ≥ 2 .
268 IMO Problems, Theorems, and Methods: Algebra
Score Situation This particular problem saw the following distribution of scores among
contestants: 92 contestants scored 7 points, 11 contestants scored 6 points, 13 contestants
scored 5 points, 10 contestants scored 4 points, 15 contestants scored 3 points, 33 contestants
scored 2 points, 60 contestants scored 1 point, and 151 contestants scored 0 point. The
average score of this problem is 2.561, indicating that it had a certain level of difficulty.
Among the top five teams in the team scores, the scores of this problem are as follows:
the United States team scored 40 points (with a total team score of 252 points), the China
team scored 42 points (with a total team score of 229 points), the Russia team scored 29
points (with a total team score of 224 points), the Bulgaria team scored 36 points (with a
total team score of 223 points), and the Hungary team scored 35 points (with a total team
score of 221 points).
The gold medal cutoff for this IMO was set at 40 points (with 30 contestants earning gold
medals), the silver medal cutoff was 30 points (with 64 contestants earning silver medals),
and the bronze medal cutoff was 19 points (with 98 contestants earning bronze medals).
In this IMO, a total of 22 contestants achieved a perfect score of 42 points.
1 1 1 3
+ + ≥ .
a3 (b + c) b3 (c + a) c3 (a + b) 2
1 1 1
Proof 1. Since abc = 1, it follows that a + b + c = bc + ca + ab. By the
Cauchy–Schwarz inequality,
1 1 1
+ + (a(b + c) + b(a + c) + c(a + b))
a3 (b + c) b3 (a + c) c3 (a + b)
2
1 1 1
≥ + + ,
a b c
1 1 1
and thus, ( a3 (b+c) + b3 (a+c) + c3 (a+b) ) ≥ 12 ( a1 + 1
b + 1c ) ≥ 3
2 · 3 1
abc = 32 .
bc ca ab
ab ≤ ac ≤ bc, ≥ ≥ .
a(b + c) b(c + a) c(a + b)
Inequality Problems 269
Score Situation This particular problem saw the following distribution of scores among
contestants: 90 contestants scored 7 points, 5 contestants scored 6 points, 4 contestants
scored 5 points, 1 contestant scored 4 points, 1 contestant scored 3 points, 5 contestants
scored 2 points, 7 contestants scored 1 point, and 299 contestants scored 0 point. The
average score of this problem is 1.709, indicating that it was relatively challenging.
Among the top five teams in the team scores, the scores of this problem are as follows:
the China team scored 42 points (with a total team score of 236 points), the Romania team
scored 35 points (with a total team score of 230 points), the Russia team scored 42 points
(with a total team score of 227 points), the Vietnam team scored 42 points (with a total team
score of 220 points), and the Hungary team scored 21 points (with a total team score of
210 points).
The gold medal cutoff for this IMO was set at 37 points (with 30 contestants earning gold
medals), the silver medal cutoff was 29 points (with 71 contestants earning silver medals),
and the bronze medal cutoff was 19 points (with 100 contestants earning bronze medals).
In this IMO, a total of 14 contestants achieved a perfect score of 42 points.
Since the absolute values of S(π0 ) and S(π) both exceed r, their signs
must be opposite, with one greater than r and the other less than −r.
It is observed that starting from π0 , one can reach any permutation
by performing several exchanges of adjacent elements in the permutation
π0 . Thus, there exists a sequence of permutations π0 , π1 , . . . , πm such that
πm = π, and for each i ∈ {0, 1, 2, . . . , m − 1}, the permutation πi+1 is
obtained by swapping two adjacent elements in πi .
Inequality Problems 271
This indicates that in the sequence S(π0 ), S(π1 ), . . . , S(πm ), the dis-
tance between any two consecutive numbers on the number line does not
exceed 2r. Furthermore, S(π0 ) and S(πm ) are on opposite sides of the inter-
val [−r, r], so there must be at least one S(πi ) within this interval, implying
the existence of a permutation πi such that |S(πi )| ≤ r. This concludes the
proof.
is divisible by p.
• (William Lowell Putnam Mathematical Competition 2017,
B6). Find the number of ordered 64-tuples (x0 , x1 , . . . , x63 ) such that
x0 , x1 , . . . , x63 are distinct elements of {1, 2, . . . , 2017} and
x0 + x1 + 2x2 + 3x3 + · · · + 63x63
is divisible by 2017.
Score Situation This particular problem saw the following distribution of scores among
contestants: 88 contestants scored 7 points, 6 contestants scored 6 points, 2 contestants
scored 5 points, 4 contestants scored 4 points, 22 contestants scored 3 points, 5 contestants
272 IMO Problems, Theorems, and Methods: Algebra
scored 2 points, 64 contestants scored 1 point, and 269 contestants scored 0 point. The
average score of this problem is 1.778, indicating that it was relatively challenging.
Among the top five teams in the team scores, the scores of this problem are as follows:
the China team scored 38 points (with a total team score of 223 points), the Hungary team
scored 35 points (with a total team score of 219 points), the Iran team scored 42 points
(with a total team score of 217 points), the United States team scored 32 points (with a
total team score of 202 points), and the Russia team scored 29 points (with a total team
score of 202 points).
The gold medal cutoff for this IMO was set at 35 points (with 39 contestants earning gold
medals), the silver medal cutoff was 25 points (with 70 contestants earning silver medals),
and the bronze medal cutoff was 15 points (with 122 contestants earning bronze medals).
In this IMO, a total of four contestants achieved a perfect score of 42 points.
1 1 1
Proof 2. Since b − 1 + c = b(1 − b + bc ) = b(1 + a − 1b ),
1 1
a−1+ · b−1+
b c
2
1 1 2 1
= b a−1+ a+1− =b a − 1− ≤ ba2 .
b b b
1 1
b−1+ > 0, c−1+ > 0,
c a
indicating that the original inequality holds in this case.
If a − 1 + 1b , b − 1 + 1c , and c − 1 + a1 are all positive, then from the above
three inequalities,
2 2 2
1 1 1
a−1+ · b−1+ · c−1+ ≤ a3 b3 c3 = 1,
b c a
which implies (a − 1 + 1b ) · (b − 1 + 1c ) · (c − 1 + a1 ) ≤ 1.
1 1
v =b−1+ > 0, w =c−1+ > 0,
c a
implying uvw ≤ 0 < 1.
274 IMO Problems, Theorems, and Methods: Algebra
a aλ
√ ≥ λ .
a2 + 8bc a + b λ + cλ
λ λ
2 λ λ
a2 (aλ + bλ + cλ )2 ≥ a2 aλ + 2b 2 c 2 = a2 a2λ + 4aλ b 2 c 2 + 4bλ cλ
λ 3λ 3λ
≥ a2 a2λ + 8a 2 b 4 c 4
λ 3λ 3λ
= a2λ+2 + 8a 2 +2 b 4 c 4 .
λ 3λ 3λ
a2λ+2 + 8a 2 +2 b 4 c 4 = a2λ (a2 + 8bc).
4
Thus, √ a ≥ 4
a3
4 4 . Similarly,
a2 +8bc a3 +b 3 +c 3
4 4
b b3 c c3
√ ≥ 4 4 4 and √ ≥ 4 4 4 .
b2 + 8ca a3 + b3 + c3 c2 + 8bc a3 + b3 + c3
a b c
√ +√ +√ ≥ 1.
2
a + 8bc 2
b + 8ca 2
c + 8ab
3
a a
√ √ a(a2 + 8bc) ≥ a .
cyc a2 + 8bc cyc a2 + 8bc cyc cyc
Thus, we only need to prove ( cyc a)3 ≥ 2
cyc a(a + 8bc), which is
2
equivalent to proving cyc a(b − c) ≥ 0.
Therefore, the original inequality holds.
276 IMO Problems, Theorems, and Methods: Algebra
Thus, we only need to prove a(a2 + 8bc) + b(b2 + 8ca) + c(c2 + 8ab) ≤ 1,
which is equivalent to
a b c
x= √ , y=√ , z= √ ,
2
a + 8bc 2
b + 8ca 2
c + 8ab
and then use ( x12 − 1)( y12 − 1)( z12 − 1) = 512 to derive a contradiction.
Next, we can prove a more general proposition:
Let a1 , a2 , . . . , an be positive real numbers with a1 a2 · · · an = 1. Then
1 1 1
+ + ···+ ≥ 1.
1 + (n2 − 1)a1 1 + (n2 − 1)a2 1 + (n2 − 1)an
Let xi = √ 1
for i ∈ {1, 2, . . . , n}. We prove that if x1 + x2 +
1+(n2 −1)ai
· · · + xn < 1, then a1 a2 · · · an > 1. Denote x1 x2 · · · xn = P . Since ai =
1−x2i
(n2 −1)x2
,
i
n n
(1 − x2i ) = (n2 − 1)n P 2 ai .
i=1 i=1
Inequality Problems 277
From x1 +x2 +· · ·+xn < 1 and the AM-GM inequality, for j ∈ {1, 2, . . . , n},
P
1 − xj > x1 + · · · + xj−1 + xj+1 + · · · + xn ≥ (n − 1) n−1
,
xj
1 + xj > xj + x1 + x2 + · · · + xn ≥ (n + 1) n+1 xj P .
Thus,
2
− n2 −1
2n
1 − x2j > (n2 − 1)xj P n2 −1 ,
n
(1 − x2i ) > (n2 − 1)n P 2 .
i=1
n n
Hence, (n2 − 1)n P 2 ai =
i=1
2 2 n 2
i=1 (1 − xi ) > (n − 1) P , implying
a1 a2 · · · an > 1.
Furthermore, there are several related propositions:
a b c
√
3
+ √
3
+ √
3
≥ 1.
a + 2b b + 2c c + 2a
Score Situation This particular problem saw the following distribution of scores among
contestants: 77 contestants scored 7 points, 3 contestants scored 6 points, 7 contestants
scored 5 points, 11 contestants scored 4 points, 9 contestants scored 3 points, 15 contestants
scored 2 points, 40 contestants scored 1 point, and 311 contestants scored 0 point. The
average score of this problem is 1.550, indicating that it was relatively challenging.
Among the top five teams in the team scores, the scores of this problem are as follows:
the China team scored 40 points (with a total team score of 225 points), the United States
team scored 31 points (with a total team score of 196 points), the Russia team scored 28
points (with a total team score of 196 points), the South Korea team scored 42 points (with
a total team score of 185 points), and the Bulgaria team scored 19 points (with a total team
score of 185 points).
The gold medal cutoff for this IMO was set at 30 points (with 39 contestants earning gold
medals), the silver medal cutoff was 20 points (with 81 contestants earning silver medals),
and the bronze medal cutoff was 11 points (with 122 contestants earning bronze medals).
In this IMO, a total of four contestants achieved a perfect score of 42 points.
Inequality Problems 279
Also show that the equality is valid if and only if the sequence is an arith-
metic progression.
Proof. (i) Since subtracting the same number from all xi does not change
n
the inequality, without loss of generality, we assume i=1 xi = 0. From the
given condition,
n n n
|xi − xj | = 2 (xj − xi ) = 2 (2i − n − 1)xi ,
i=1 j=1 i<j i=1
n
n
≤4 (2i − n − 1)2 x2i
i=1 i=1
n
4n(n + 1)(n − 1) 2
= xi .
3 i=1
Furthermore,
n n n
n
n
n
n
(xi − xj )2 = n x2i − 2 xi xj + n x2j = 2n x2i .
i=1 j=1 i=1 i=1 j=1 j=1 i=1
n n 2 n n
Thus, ( i=1 j=1 |xi − xj |)2 ≤ 2(n 3−1) i=1 j=1 (xi − xj )2 .
(ii) From the conditions for the equality in the Cauchy–Schwarz inequal-
ity, the equality holds if and only if there exists a real number k such that
xi = k(2i − n − 1), i.e., xi = 2ki − k(n + 1). Thus, x1 , x2 , . . . , xn form an
arithmetic progression.
If x1 , x2 , . . . , xn form an arithmetic sequence with common difference d,
then xi = d2 (2i − n − 1) + x1 +x 2
n
. Subtracting x1 +x
2
n
from each xi yields
n
d
xi = (2i − n − 1), and xi = 0.
2 i=1
In this case, the equality holds in the given inequality.
280 IMO Problems, Theorems, and Methods: Algebra
0 ≤ xi ≤ 1, i = 1, 2, . . . , n,
Let S(n) denote the largest possible value of the sum (*). Find S(n).
Score Situation This particular problem saw the following distribution of scores among
contestants: 67 contestants scored 7 points, 4 contestants scored 6 points, 1 contestant
scored 5 points, 5 contestants scored 4 points, 6 contestants scored 3 points, 20 contestants
scored 2 points, 161 contestants scored 1 point, and 193 contestants scored 0 point. The
average score of this problem is 1.613, indicating that it was relatively challenging.
Inequality Problems 281
Among the top five teams in the team scores, the scores of this problem are as follows:
the Bulgaria team scored 37 points (with a total team score of 227 points), the China team
scored 42 points (with a total team score of 211 points), the United States team scored 36
points (with a total team score of 188 points), the Vietnam team scored 36 points (with a
total team score of 172 points), and the Russia team scored 30 points (with a total team
score of 167 points).
The gold medal cutoff for this IMO was set at 29 points (with 37 contestants earning gold
medals), the silver medal cutoff was 19 points (with 69 contestants earning silver medals),
and the bronze medal cutoff was 13 points (with 104 contestants earning bronze medals).
In this IMO, only three contestants achieved a perfect score of 42 points, namely
Bao Le Hùng Viet and Trong Canh Nguyen from Vietnam, and Yunhao Fu from China.
Show that ti , tj , tk are side lengths of a triangle for all i, j, k with 1 ≤ i <
j < k ≤ n.
t1 t3 t2 t3 ti tj
= + + + + + +n
t3 t1 t3 t2 tj ti
1≤i<j≤n
(i,j)∈{(1,3),(2,3)}
/
t 1 + t2 1 1
≥ + t3 + + 2+n
t3 t1 t2
1≤i<j≤n
(i,j)∈{(1,3),(2,3)}
/
282 IMO Problems, Theorems, and Methods: Algebra
t 1 + t2 4t3
≥ + + 2(C2n − 2) + n
t3 t 1 + t2
t 1 + t2 4t3
= + + n2 − 4. (1)
t3 t 1 + t2
t3
Let x = t1 +t2 . Then x ≥ 1, and
1 (x − 1)(4x − 1)
4x + −5= ≥ 0.
x x
From (1),
1 1 1
(t1 + t2 + · · · + tn ) + + ···+ ≥ 5 + n2 − 4 = n2 + 1,
t1 t2 tn
which leads to a contradiction. Therefore, the original proposition holds.
Proof 2. We employ mathematical induction to prove that if there exist
ti , tj , tk such that ti + tj ≤ tk , then
1 1 1
(t1 + t2 + · · · + tn ) + + ···+ ≥ n2 + 1.
t1 t2 tn
For n = 3, assume t1 + t2 ≤ t3 , and it is easy to prove that
1 1 1
(t1 + t2 + t3 ) + + ≥ 10.
t1 t2 t3
Suppose when n = l ≥ 3 and t1 + t2 ≤ t3 ,
1 1 1
(t1 + t2 + · · · + tl ) + + ···+ ≥ l2 + 1.
t1 t2 tl
For n = l + 1,
1 1 1
(t1 + t2 + · · · + tl+1 ) + + ···+
t1 t2 tl+1
1 1 1 1
= (t1 + · · · + tl ) + ··· + + tl+1 + ···+
t1 tl t1 tl
1
+ (t1 + · · · + tl ) + 1
tl+1
2 1 1 1
≥ l + 1 + 2 tl+1 + ··· + (t1 + · · · + tl ) + 1
t1 tl tl+1
≥ l2 + 1 + 2 l2 + 1 + 1
> l2 + 1 + 2l + 1
= (l + 1)2 + 1.
Inequality Problems 283
max{a1 , a2 , . . . , an } ≤ n · min{a1 , a2 , . . . , an },
there exist three that are the side lengths of an acute triangle.
284 IMO Problems, Theorems, and Methods: Algebra
Score Situation This particular problem saw the following distribution of scores among
contestants: 250 contestants scored 7 points, 6 contestants scored 6 points, 6 contestants
scored 5 points, 8 contestants scored 4 points, 16 contestants scored 3 points, 27 contestants
scored 2 points, 33 contestants scored 1 point, and 140 contestants scored 0 point. The
average score of this problem is 4.080, indicating that it was simple.
Among the top five teams in the team scores, the scores of this problem are as follows:
the China team scored 42 points (with a total team score of 220 points), the United States
team scored 42 points (with a total team score of 212 points), the Russia team scored 42
points (with a total team score of 205 points), the Vietnam team scored 42 points (with a
total team score of 196 points), and the Bulgaria team scored 42 points (with a total team
score of 194 points).
The gold medal cutoff for this IMO was set at 32 points (with 45 contestants earning gold
medals), the silver medal cutoff was 24 points (with 78 contestants earning silver medals),
and the bronze medal cutoff was 16 points (with 120 contestants earning bronze medals).
In this IMO, a total of four contestants achieved a perfect score of 42 points.
x2 +y 2 +z 2 yz+y 2 +z 2
Therefore, x5 +y 2 +z 2 ≤ x2 +y 2 +z 2 . Similarly,
x2 + y 2 + z 2 zx + z 2 + x2 x2 + y 2 + z 2 xy + x2 + y 2
≤ , ≤ .
y 5 + z 2 + x2 x2 + y 2 + z 2 z 5 + x2 + y 2 x2 + y 2 + z 2
Summing up these three inequalities and using x2 +y 2 +z 2 ≥ xy+yz+zx,
we obtain
x2 + y 2 + z 2 x2 + y 2 + z 2 x2 + y 2 + z 2 xy + yz + zx
+ + 5 ≤2+ 2 ≤ 3.
x5 + y 2 + z 2 y 5 + z 2 + x2 z + x2 + y 2 x + y2 + z 2
Furthermore, since
x5 + xyz(y 2 + z 2 ) ≤ xyz(x5 + y 2 + z 2 ),
x2 x2 · xyz
≤
cyc
x5 + y 2 + z 2 cyc
x5 + xyz(y 2 + z 2 )
x2 yz
= .
cyc
x4 + yz(y 2 + z 2 )
x4 + yz(y 2 + z 2 ) ≥ x2 yz + xy 2 z + xyz 2 .
Hence,
x2 yz x2 yz
≤ = 1.
cyc
x4 2 2
+ yz(y + z ) cyc
x yz + xy 2 z + xyz 2
2
Score Situation This particular problem saw the following distribution of scores among
contestants: 55 contestants scored 7 points, 9 contestants scored 6 points, no contestant
scored 5 points, no contestant scored 4 points, no contestant scored 3 points, 3 contestants
scored 2 points, 23 contestants scored 1 point, and 423 contestants scored 0 point. The
average score of this problem is 0.912, indicating that it was extremely difficult.
Among the top five teams in the team scores, the scores of this problem are as follows:
the China team scored 35 points (with a total team score of 235 points), the United States
team scored 35 points (with a total team score of 213 points), the Russia team scored
20 points (with a total team score of 212 points), the Iran team scored 22 points (with a
total team score of 201 points), and the South Korea team scored 27 points (with a total
team score of 200 points).
The gold medal cutoff for this IMO was set at 35 points (with 42 contestants earning gold
medals), the silver medal cutoff was 23 points (with 79 contestants earning silver medals),
and the bronze medal cutoff was 12 points (with 128 contestants earning bronze medals).
In this IMO, a total of 16 contestants achieved a perfect score of 42 points.
d
xk − ak ≤ for all 1 ≤ k ≤ n. (1)
2
For any 1 ≤ k ≤ n, let l ≤ k be the smallest index such that xk = xl .
Then either l = 1 or l ≥ 2 and xl > xl−1 . In both cases,
d
xk = xl = al − . (2)
2
Since al − ak ≤ max{aj : 1 ≤ j ≤ k} − min{aj : k ≤ j ≤ n} = dk ≤ d,
by (2),
d d d
xk − ak = al − ak − ≤d− = ,
2 2 2
which is (1). This shows that − d2 ≤ xk − ak ≤ d
2 for all 1 ≤ k ≤ n. Hence,
d
max{|xi − ai | : 1 ≤ i ≤ n} ≤ .
2
By (a), the sequence {xk } indeed makes the equality in (*).
Proof 2. (a) The same as Proof 1.
(b) For each i(1 ≤ i ≤ n), let
Then
indicating both sequences {Mi } and {mi } are non-decreasing, and by their
definition, mi ≤ ai ≤ Mi .
Let xi = Mi +m
2
i
, and from di = Mi − mi ,
di mi − M i M i − mi di
− = = xi − Mi ≤ xi − ai ≤ xi − mi = = .
2 2 2 2
290 IMO Problems, Theorems, and Methods: Algebra
Therefore,
"
di d
max{|xi − ai | : 1 ≤ i ≤ n} ≤ max :1≤i≤n = .
2 2
By (a), the sequence {xk } indeed makes the equality in (*).
Note. There are several similar problems:
Score Situation This particular problem saw the following distribution of scores among
contestants: 161 contestants scored 7 points, 21 contestants scored 6 points, 18 contestants
scored 5 points, 18 contestants scored 4 points, 105 contestants scored 3 points, 8 contestants
scored 2 points, 13 contestants scored 1 point, and 176 contestants scored 0 point. The
average score of this problem is 3.383, indicating that it was relatively straightforward.
Among the top five teams in the team scores, the scores of this problem are as follows:
the Russia team scored 42 points (with a total team score of 184 points), the China team
scored 36 points (with a total team score of 181 points), the Vietnam team scored 41 points
(with a total team score of 168 points), the South Korea team scored 38 points (with a total
team score of 168 points), and the United States team scored 41 points (with a total team
score of 155 points).
The gold medal cutoff for this IMO was set at 29 points (with 39 contestants earning gold
medals), the silver medal cutoff was 21 points (with 83 contestants earning silver medals),
and the bronze medal cutoff was 14 points (with 131 contestants earning bronze medals).
In this IMO, no contestant achieved a perfect score of 42 points.
x y z a b
Proof 1. (a) Let a = x−1 , b = y−1 , and c = z−1 . Then x = a−1 , y = b−1 ,
c
and z = c−1 . From xyz = 1, we have abc = (a − 1)(b − 1)(c − 1), i.e.,
a + b + c − 1 = ab + bc + ca.
Therefore,
= (a + b + c)2 − 2(a + b + c − 1)
= (a + b + c − 1)2 + 1 ≥ 1.
x2 y2 z2
Hence, (x−1)2 + (y−1)2 + (z−1)2 ≥ 1.
k 2 k−1
(b) Let (x, y, z) = (− (k−1) 2 , k − k , k2 ),
where k is a positive integer.
Then (x, y, z) is a triple of rational numbers, and x, y, z are all not equal to 1.
For different positive integers k, the triples of rational numbers (x, y, z) are
distinct. In this case,
x2 y2 z2
2
+ 2
+
(x − 1) (y − 1) (z − 1)2
k2 (k − k 2 )2 (k − 1)2
= + 2 + 2
(k 2 − k + 1)2 (k − k + 1) 2 (k − k + 1)2
k 4 − 2k 3 + 3k 2 − 2k + 1
= = 1.
(k 2 − k + 1)2
x2 y2 z2
+ + ≥1
(x − 1)2 (y − 1)2 (z − 1)2
p2 q2 r2
⇔ 2
+ 2
+ ≥ 1. (1)
(p − q) (q − r) (r − p)2
292 IMO Problems, Theorems, and Methods: Algebra
p q r
Let a = p−q ,b= q−r , and c = r−p . Then the inequality (1) becomes
2
a ≥ 1. Since
From (2), 1 − cyc a2 = −(a + b + c − 1)2 ≤ 0. Thus, cyc a2 ≥ 1, and
the inequality (1) holds.
2
(b) Let b = t2t+t+1+t
, c = t2t+1 bc
+t+1 , and a = − b+c , where t can take
any rational number except 0 and −1. Changing t results in an infinite
number of rational triples (a, b, c), where a, b, c are all not equal to 1 and
2
cyc a = cyc a = 1.
a b c
Therefore, (x, y, z) = ( a−1 , b−1 , c−1 ) makes (b) hold.
√ √ √
3
√ 3 a2
3 2
x
Proof 3. (a) Let a = x, b = 3 y, and c = z. Then = √ =
bc 3 yz
2 2
√ x b c
3 xyz = x, and similarly ca = y and ab = z. Thus, the inequality to be
proved becomes
a4 b4 c4
+ 2 + 2 ≥ 1.
(a2 − bc)2 (b − ca)2 (c − ab)2
a4 b4 c4
+ 2 + 2
(a2 − bc)2 (b − ca)2 (c − ab)2
(a2 + b2 + c2 )2
≥ ,
(a2 − bc)2 + (b2 − ca)2 + (c2 − ab)2
and further,
• (Iran Team Selection Test 2018, Problem 2). Determine the least
real number k such that the inequality
2 2 2
2a 2b 2c 2a 2b 2c
+ + +k ≥4 + +
a−b b−c c−a a−b b−c c−a
is satisfied for all real numbers a, b, c.
• (Canadian Mathematical Olympiad 2017, Problem 1). Let a, b,
and c be non-negative real numbers, no two of which are equal. Prove
that
a2 b2 c2
2
+ 2
+ > 2.
(b − c) (c − a) (a − b)2
• (German Mathematical Olympiad 2010, Problem 2). Let a, b, c
be distinct real numbers. Show that
2 2 2
2a − b 2b − c 2c − a
+ + ≥ 5.
a−b b−c c−a
• (From the book Secrets in Inequalities, 2007: 151). Let x, y, z be
distinct real numbers. Prove that
x2 y2 z2
+ + ≥ 1.
(x − y)2 (y − z)2 (z − x)2
• (United Kingdom Team Selection Test 2005, NST2, Problem 3).
Let n ≥ 3 be an integer and a1 , a2 , . . . , an be positive real numbers such
that a1 a2 . . . an = 1. Prove that
a1 + 3 a2 + 3 an + 3
+ + ··· + ≥ 3.
(a1 + 1)2 (a2 + 1)2 (an + 1)2
Score Situation This particular problem saw the following distribution of scores among
contestants: 94 contestants scored 7 points, 7 contestants scored 6 points, 53 contestants
scored 5 points, 35 contestants scored 4 points, 3 contestants scored 3 points, 24 contestants
scored 2 points, 209 contestants scored 1 point, and 110 contestants scored 0 point. The
average score of this problem is 2.563, indicating that it had a certain level of difficulty.
Among the top five teams in the team scores, the scores of this problem are as follows:
the China team scored 42 points (with a total team score of 217 points), the Russia team
294 IMO Problems, Theorems, and Methods: Algebra
scored 33 points (with a total team score of 199 points), the United States team scored 30
points (with a total team score of 190 points), the South Korea team scored 42 points (with
a total team score of 188 points), and the Iran team scored 37 points (with a total team
score of 181 points).
The gold medal cutoff for this IMO was set at 31 points (with 47 contestants earning gold
medals), the silver medal cutoff was 22 points (with 100 contestants earning silver medals),
and the bronze medal cutoff was 15 points (with 120 contestants earning bronze medals).
In this IMO, only three contestants achieved a perfect score of 42 points, namely
Xiaosheng Mu and Dongyi Wei from China, and Alex Zhai from the United States.
(1 + a2 )2 (1 + a3 )3 · · · (1 + an )n > nn .
Thus,
(1 + a2 )2 (1 + a3 )3 · · · (1 + an )n
3 3 a3 4 4 a4 nn an
≥ 2 2 a2 × × × · · · ×
22 33 (n − 1)n−1
= nn ,
1
and the equality holds when ak = k−1 for k ∈ {2, 3, . . . , n}.
However, this contradicts the condition a2 a3 · · · an = 1. Hence,
(1 + a2 )2 (1 + a3 )3 · · · (1 + an )n > nn .
(1 + x1 )(1 + x1 + x2 ) · · · (1 + x1 + x2 + · · · + xn )
√
≥ (n + 1)n+1 x1 x2 · · · xn .
a1 !a2 ! · · · an ! ≥ ( An !)n ,
Score Situation This particular problem saw the following distribution of scores among
contestants: 171 contestants scored 7 points, 7 contestants scored 6 points, 2 contestants
scored 5 points, 8 contestants scored 4 points, 5 contestants scored 3 points, 8 contestants
scored 2 points, 83 contestants scored 1 point, and 263 contestants scored 0 point. The
average score of this problem is 2.550, indicating that it had a certain level of difficulty.
Among the top five teams in the team scores, the scores of this problem are as follows:
the South Korea team scored 42 points (with a total team score of 209 points), the China
team scored 40 points (with a total team score of 195 points), the United States team scored
40 points (with a total team score of 194 points), the Russia team scored 35 points (with a
total team score of 177 points), the Thailand team scored 42 points (with a total team score
of 159 points), and the Canada team scored 32 points (with a total team score of 159 points).
The gold medal cutoff for this IMO was set at 28 points (with 51 contestants earning gold
medals), the silver medal cutoff was 21 points (with 88 contestants earning silver medals),
and the bronze medal cutoff was 14 points (with 137 contestants earning bronze medals).
In this IMO, only one contestant achieved a perfect score of 42 points, namely Jeck Lim
from Singapore.
Score Situation This particular problem saw the following distribution of scores among
contestants: 370 contestants scored 7 points, 23 contestants scored 6 points, 18 contestants
scored 5 points, 15 contestants scored 4 points, 22 contestants scored 3 points, 14 contestants
scored 2 points, 23 contestants scored 1 point, and 75 contestants scored 0 point. The
average score of this problem is 5.348, indicating that it was simple.
298 IMO Problems, Theorems, and Methods: Algebra
Among the top five teams in the team scores, the scores of this problem are as follows:
the China team scored 42 points (with a total team score of 201 points), the United States
team scored 42 points (with a total team score of 193 points), the Chinese Taiwan team
scored 42 points (with a total team score of 192 points), the Russia team scored 42 points
(with a total team score of 191 points), and the Japan team scored 38 points (with a total
team score of 177 points).
The gold medal cutoff for this IMO was set at 29 points (with 49 contestants earning gold
medals), the silver medal cutoff was 22 points (with 113 contestants earning silver medals),
and the bronze medal cutoff was 16 points (with 133 contestants earning bronze medals).
In this IMO, only three contestants achieved a perfect score of 42 points, namely Jiyang
Gao from China, Po-Sheng Wu from Chinese Taiwan, and Alexander Gunning from Australia.
(a + 2b + 3c + 4d)aa bb cc dd < 1.
aa bb cc dd ≤ a · a + b · b + c · c + d · d = a2 + b2 + c2 + d2 .
(a + 2b + 3c + 4d)(a2 + b2 + c2 + d2 ) < 1.
Given a ≥ b ≥ c ≥ d > 0,
(a + b + c + d)3 = a3 + 3 a2 b + 6 abc
≥ (a + 2b + 3c + 4d)(a2 + b2 + c2 + d2 ),
Proof 2. If a ≤ 12 , then
(a + 2b + 3c + 4d)aa bb cc dd ≤ (a + 3b + 3c + 3d)aa+b+c+d
= (3 − 2a)a
= 1 − (1 − a)(1 − 2a)
≤ 1.
1
The equality holds if and only if a = b = c = d = 2, which is a
contradiction.
If a > 12 , then,
(a + 2b + 3c + 4d)aa bb cc dd ≤ (3 − 2a)aa (1 − a)1−a .
By the weighted AM-GM inequality,
a 1−a
·a+ · (1 − a) ≥ aa (1 − a)1−a ,
a + (1 − a) a + (1 − a)
and thus,
(3 − 2a)aa (1 − a)1−a ≤ (3 − 2a)(a2 + (1 − a)2 )
= (3 − 2a)(1 − 2a + 2a2 )
< 1.
Hence, the original inequality is true.
Note. The key to Proof 1 is to use the weighted AM-GM inequality to
transform aa bb cc dd into a2 + b2 + c2 + d2 , and then to prove a four-variable
cubic inequality.
Score Situation This particular problem saw the following distribution of scores among
contestants: 138 contestants scored 7 points, 9 contestants scored 6 points, 7 contestants
scored 5 points, 4 contestants scored 4 points, 9 contestants scored 3 points, 129 contestants
scored 2 points, 29 contestants scored 1 point, and 291 contestants scored 0 point. The
average score of this problem is 2.248, indicating that it had a certain level of difficulty.
Among the top five teams in the team scores, the scores of this problem are as follows:
the China team scored 38 points (with a total team score of 215 points), the Russia team
scored 31 points (with a total team score of 185 points), the United States team scored 36
points (with a total team score of 183 points), the South Korea team scored 32 points (with
a total team score of 175 points), and the Thailand team scored 37 points (with a total team
score of 174 points).
300 IMO Problems, Theorems, and Methods: Algebra
The gold medal cutoff for this IMO was set at 31 points (with 49 contestants earning gold
medals), the silver medal cutoff was 24 points (with 112 contestants earning silver medals),
and the bronze medal cutoff was 15 points (with 155 contestants earning bronze medals).
In this IMO, only one contestant achieved a perfect score of 42 points, namely Jinmin Li
from China.
Assume the inequality (*) holds for fewer than n real numbers. If there
is some xi = 0, then the terms containing xi on both sides of inequality
(*) are equal. By removing xi from {x1 , x2 , . . . , xn }, we reduce it to the
inductive hypothesis for n − 1 real numbers.
Similarly, if xi = −xj = 0, then all terms on both sides of inequality (*)
involving xi or xj are
2 2|xi | + ( |xk − xi | + |xk + xi |),
k = 1, . . . , n
k = i, j
where the first inequality follows from the previous transformation to the
inductive hypothesis. This completes the inductive proof.
Note. If f is a non-decreasing concave function defined on [0, +∞) with
f (0) = 0, then for any real numbers x1 , x2 , . . . , xn ,
n
n n
n
f (|xi − xj |) ≤ f (|xi + xj |).
i=1 j=1 i=1 j=1
Bin Zhao provided the proof in his paper “A Generation of the 2021
IMO Problem 2 (2021 IMO ),” published on the
New Star Math Website (http://www.nsmath.cn/jszl).
Furthermore, there are several similar problems:
n !
!
! xi + xi+1 !
! !
! xi − xi+1 ! ≥ λ(n).
i=1
n
ai − b i
≤λ
i=1
ai + b i
a1 a2 a2018 a2019
+ + ···+ + > C.
|a2 − a3 | |a3 − a4 | |a2019 − a1 | |a1 − a2 |
Score Situation This particular problem saw the following distribution of scores among
contestants: 16 contestants scored 7 points, 2 contestants scored 6 points, 1 contestant
scored 5 points, 3 contestants scored 4 points, 2 contestants scored 3 points, 12 contestants
scored 2 points, 61 contestants scored 1 point, and 522 contestants scored 0 point. The
average score of this problem is 0.375, indicating that it was extremely difficult.
Among the top five teams in the team scores, the scores of this problem are as follows:
the China team scored 16 points (with a total team score of 208 points), the Russia team
scored 17 points (with a total team score of 183 points), the South Korea team scored
9 points (with a total team score of 172 points), the United States team scored 2 points
(with a total team score of 165 points), and the Canada team scored 8 points (with a total
team score of 151 points).
The gold medal cutoff for this IMO was set at 24 points (with 52 contestants earning gold
medals), the silver medal cutoff was 19 points (with 103 contestants earning silver medals),
and the bronze medal cutoff was 12 points (with 148 contestants earning bronze medals).
In this IMO, only one contestant achieved a perfect score of 42 points, namely Yichuan
Wang from China.
Inequality Problems 303
an+1 − an = an − an−1 = 1,
implying
n−1 n−1
i=1 xi i=1 xi + xn an−1 xn + xn
xn = , xn+1 = = = xn .
an−1 an an−1 + 1
However, this contradicts the assumption that x1 , x2 , . . . , x2023 are all
distinct. Therefore, an+1 − an−1 ≥ 3, and consequently
1001
a2023 = a1 + (a2i+1 − a2i−1 ) ≥ 1 + 1001 × 3 = 3034.
i=1
n n
1 1 1
≥ xi + (xn+1 + xn+2 ) +
i=1
x
i=1 i
xn+1 xn+2
1 1
= an + (xn+1 + xn+2 ) + .
xn+1 xn+2
Since x1 , x2 , . . . , x2023 are all distinct and a1 , a2 , . . . , a2023 are integers,
1 1
(xn+1 + xn+2 )( xn+1 + xn+2 ) > 4, so an+2 > an + 2, implying an+2 ≥ an + 3
for every n in the set {1, 2, . . . , 2021}.
Therefore, a2023 ≥ 3034.
Note. There are several similar problems, which are presented in
Problem 4.22.
Score Situation This particular problem saw the following distribution of scores among
contestants: 384 contestants scored 7 points, 3 contestants scored 6 points, 1 contestant
scored 5 points, 4 contestants scored 4 points, 8 contestants scored 3 points, 32 contestants
scored 2 points, 100 contestants scored 1 point, and 86 contestants scored 0 point. The
average score of this problem is 4.717, indicating that it was simple.
Among the top five teams in the team scores, the scores of this problem are as follows:
the China team scored 42 points (with a total team score of 240 points), the United States
team scored 42 points (with a total team score of 222 points), the South Korea team scored
42 points (with a total team score of 215 points), the Romania team scored 42 points (with
a total team score of 208 points), and the Canada team scored 42 points (with a total team
score of 183 points).
The gold medal cutoff for this IMO was set at 32 points (with 54 contestants earning gold
medals), the silver medal cutoff was 25 points (with 90 contestants earning silver medals),
and the bronze medal cutoff was 18 points (with 170 contestants earning bronze medals).
In this IMO, a total of five contestants achieved a perfect score of 42 points.
Solution 1. Since
a b a b
+ < + = 1,
d+a+b a+b+c a+b a+b
c d c d
+ < + = 1,
b+c+d c+d+a c+d c+d
it follows that S < 2. Furthermore,
a b c d
S> + + + = 1,
a+b+c+d a+b+c+d a+b+c+d a+b+c+d
implying 1 < S < 2.
For any value 1 + t(0 < t < 1) in the interval (1, 2), let a = c = 1. Then
S = 1 + t if
2 2 d 2 b 2
t=S−1= − + = + − . (1)
1+b+d b+2 d+2 1+b+d b+2 d+2
2
Choose b = b0 sufficiently small such that (b0 +1)(b0 +2)
> t > b0b+2
0
. Then
the function of d,
2 2 d
f (d) = (1 + b0 + d)(d + 2) − + −t
1 + b0 + d b0 + 2 d + 2
2 b0 2
= (1 + b0 + d)(d + 2) + − −t ,
1 + b0 + d b0 + 2 d + 2
has f (0) = 2( b02+2 − t(b0 + 1)) > 0, and f (d) becomes negative as d → +∞.
Therefore, there must exist d0 such that f (d0 ) = 0, i.e., there are b0 , d0
making (1) hold.
Thus, the range of S is the open interval (1, 2).
Solution 2. It is evident that the sum S is homogeneous in a, b, c, d, so we
can assume
x = a + c, y = b + d, and x + y = 1.
2
a c a c
Consider the sum S1 = +
d+a+b = 1−c
b+c+d + 1−a = 2ac+x−x
ac+1−x .
If x is fixed, then the product ac can take any real number from 0 to
x2 x2
( a+c 2
2 ) = 4 , i.e., 0 < ac ≤ 4 . Furthermore,
2ac + x − x2 3x − 2 − x2
S1 = =2+ ,
ac + 1 − x ac + 1 − x
2x
which is monotonic in ac. Thus, the range of S1 is (x, 2−x ].
b d 2y
Similarly, if y is fixed, then the range of S2 = a+b+c + c+d+a is (y, 2−y ].
2x 2y
Consequently, the range of the sum S is (x + y, 2−x + 2−y ], i.e.,
(1, 4−4xy
2+xy ].
306 IMO Problems, Theorems, and Methods: Algebra
4−4xy
Noting that the range of xy is (0, 14 ], we find the range of 2+xy to be
[ 43 , 2).
Therefore, the range of the sum S is (1, 2).
a3 b3 c3 d3 1
+ + + ≥ .
b+c+d c+d+a d+a+b a+b+c 3
a b c d 2
+ + + ≥ .
b + 2c + 3d c + 2d + 3a d + 2a + 3b a + 2b + 3c 3
a b c d 4
+ 2 + 2 + 2 ≥ .
b2 2 2 2 2 2 2
+c +d c +d +a d +a +b a +b +c2 2 a+b+c+d
a b c d e f
+ + + + + ≥ 3.
b+c c+d d+e e+f f +a a+b
Score Situation This particular problem saw the following distribution of scores among
contestants: 27 contestants scored 7 points, 8 contestants scored 6 points, 15 contestants
scored 5 points, 13 contestants scored 4 points, 8 contestants scored 3 points, 12 contestants
scored 2 points, 22 contestants scored 1 point, and 35 contestants scored 0 point. The
average score of this problem is 3.100, indicating that it was relatively straightforward.
Among the top five teams in the team scores, the scores of this problem are as follows:
the Soviet Union team scored 44 points (with a total team score of 256 points), the United
States team scored 39 points (with a total team score of 243 points), the Hungary team
scored 33 points (with a total team score of 237 points), the German Democratic Republic
team scored 28 points (with a total team score of 236 points), and the Yugoslavia team
scored 22 points (with a total team score of 216 points).
The gold medal cutoff for this IMO was set at 38 points (with 10 contestants earning gold
medals), the silver medal cutoff was 30 points (with 24 contestants earning silver medals),
and the bronze medal cutoff was 23 points (with 37 contestants earning bronze medals).
In this IMO, a total of six contestants achieved a perfect score of 40 points.
n 4
xi xj (x2i + x2j ) ≤C xi
1≤i<j≤n i=1
Since
⎛ ⎞2
n 4 n
xi =⎝ x2k + 2 xi xj ⎠
i=1 k=1 1≤i<j≤n
⎛ ⎞
n
≥4 x2k ⎝2 xi xj ⎠
k=1 1≤i<j≤n
n
=8 xi xj x2k
1≤i<j≤n k=1
≥8 xi xj (x2i + x2j ),
1≤i<j≤n
it follows that
n 4
1
xi xj (x2i + x2j ) ≤ xi .
8 i=1
1≤i<j≤n
Thus, the minimum value of C is 18 , and the equality is true if and only if
two of the xi are equal and the others are all equal to 0.
Score Situation This particular problem saw the following distribution of scores among
contestants: 59 contestants scored 7 points, 11 contestants scored 6 points, 4 contestants
scored 5 points, 5 contestants scored 4 points, 10 contestants scored 3 points, 37 contestants
scored 2 points, 129 contestants scored 1 point, and 195 contestants scored 0 point. The
average score of this problem is 1.671, indicating that it was relatively challenging.
Among the top five teams in the team scores, the scores of this problem are as follows:
the China team scored 41 points (with a total team score of 182 points), the Russia team
scored 37 points (with a total team score of 182 points), the Vietnam team scored 37 points
(with a total team score of 177 points), the Romania team scored 42 points (with a total
team score of 173 points), and the Bulgaria team scored 23 points (with a total team score
of 170 points).
The gold medal cutoff for this IMO was set at 28 points (with 38 contestants earning gold
medals), the silver medal cutoff was 19 points (with 70 contestants earning silver medals),
and the bronze medal cutoff was 12 points (with 118 contestants earning bronze medals).
In this IMO, no contestant achieved a perfect score of 42 points.
which is equivalent to
m(a3 + b3 + c3 ) ≥ 6(a2 + b2 + c2 ) + 1
Score Situation This particular problem saw the following distribution of scores among
contestants: 28 contestants scored 7 points, 2 contestants scored 6 points, 2 contestants
scored 5 points, 1 contestant scored 4 points, 1 contestant scored 3 points, 4 contestants
scored 2 points, 95 contestants scored 1 point, and 365 contestants scored 0 point. The
average score of this problem is 0.659, indicating that it was extremely difficult.
312 IMO Problems, Theorems, and Methods: Algebra
Among the top five teams in the team scores, the scores of this problem are as follows:
the China team scored 35 points (with a total team score of 214 points), the Russia team
scored 11 points (with a total team score of 174 points), the South Korea team scored 30
points (with a total team score of 170 points), the Germany team scored 9 points (with a
total team score of 157 points), and the United States team scored 7 points (with a total
team score of 154 points).
The gold medal cutoff for this IMO was set at 28 points (with 42 contestants earning gold
medals), the silver medal cutoff was 19 points (with 89 contestants earning silver medals),
and the bronze medal cutoff was 15 points (with 122 contestants earning bronze medals).
In this IMO, only three contestants achieved a perfect score of 42 points, namely Zhiyu
Liu from China, Iurie Boreico from Moldova, and Alexander Magazinov from Russia.
4.3 Summary
Inequalities are pervasive across various branches of mathematics, includ-
ing analysis, algebra, number theory, and combinatorics, providing crucial
tools for comparison, estimation, and bounding of mathematical expres-
sions. However, more important and challenging than proving inequalities
is the ability to identify and establish relationships of inequality between
quantities in problems.
In the first 64 IMOs, there were a total of 33 inequality problems.
These problems can be broadly categorized into three types, as depicted in
Figure 4.3. The score details for these problems are presented in Table 4.2.
Due to the smaller number of participating teams and missing contestant
score information in early IMOs, there are several blanks in Table 4.2.
Problems 4.1–4.5 focus on “solving inequalities;” among these five prob-
lems, the one with the lowest average score is Problem 4.5 (IMO 29-4),
proposed by Ireland. Problems 4.6–4.30 deal with “proving inequalities;”
among these 25 problems, the one with the lowest average score is Prob-
lem 4.29 (IMO 62-2), proposed by Canada. Problems 4.31–4.33 are about
“determining value ranges;” among these three problems, the one with the
lowest average score is Problem 4.33 (IMO 47-3), proposed by Ireland.
These 33 problems were proposed by 18 countries, with Hungary con-
tributing the most, totaling four problems. Ireland, South Korea, and the
Netherlands each proposed three problems, while the United States, Poland,
France, Germany, Russia, and Austria each contributed two problems.
From Table 4.2, it can be observed that in the first 64 IMOs, there were
three inequality problems with an average score of 0–1 point; five problems
Inequality Problems 313
0
1–10 11–20 21–30 31–40 41–50 51–60 61–64
314
Problem 4.1 4.2 4.3 4.4 4.5 4.6 4.7 4.8
Full points 6.000 6.000 4.000 7.000 7.000 7.000 7.000 8.000
Average score 2.600 4.529 2.738 4.909 2.332 3.111 3.294 2.438
Top five mean 3.300 5.175 6.433 4.700 3.600
6th–15th mean 3.661
16th–25th mean 2.293
Problem number in IMO 2-2 4-2 7-1 14-4 29-4 3-2 6-2 11-6
Full points 7.000 7.000 7.000 7.000 7.000 7.000 7.000 7.000
Average score 2.563 2.550 5.348 2.248 0.375 4.717 3.100 1.671
Top five mean 6.133 6.417 6.867 5.800 1.733 7.000 4.150 6.000
6th–15th mean 5.400 4.852 6.850 4.500 1.283 6.850 3.052 3.883
16th–25th mean 3.950 3.467 6.917 3.783 0.683 6.700 1.833
Inequality Problems
Problem number in IMO 49-2 53-2 55-1 61-2 62-2 64-4 16-5 40-2
Proposing Austria Australia Austria Belgium Canada The The Poland
country Netherlands Netherlands
Problem 4.33
Note. Top five mean = Total score of the top five teams ÷ Total number of contestants from the top five teams,
6th–15th mean = Total score of the 6th–15th teams ÷ Total number of contestants from the 6th–15th teams,
16th–25th mean = Total score of the 16th–25th teams ÷ Total number of contestants from the 16th–25th teams.
315
316 IMO Problems, Theorems, and Methods: Algebra
Problem Number
Number of Problems
Inequality Problem 1, 4 2, 5 3, 6 in the First 64 IMOs
Solving inequalities 1 0 0 5
Proving inequalities 6 8 4 25
Determining value ranges 0 1 1 3
Total 7 9 5 33
0
0 5 10 15 20 25
the average score of the problem, as seen in Problem 4.33 (IMO 47-3),
Problem 4.23 (IMO 46-3), and Problem 4.20 (IMO 42-2).
From Figure 4.4, it can also be observed that there are several inequal-
ity problems where the average score is higher than the average score
of the 16th–25th teams, such as Problem 4.15 (IMO 28-3), Problem 4.5
(IMO 29-4), and Problem 4.14 (IMO 25-1). This phenomenon is due to
the smaller number of participating teams in early IMOs. It was not until
the 30th IMO in 1989 that the number of participating teams exceeded 50.
Therefore, it is common to see situations where the average score is close
to or even higher than the average score of the 16th–25th teams during this
period.
Chapter 5
Algebra, deriving from the Arabic word “al-jabr,” evolved in ancient times.
As a plethora of problems concerning quantitative relationships arose,
the need for more general methods to solve these problems led to the
development of elementary algebra, with solving equations as its core
focus.
The basic components of elementary algebra include numbers, expres-
sions, and equations, which roughly align with the algebra curriculum cur-
rently established in secondary schools worldwide. However, the algebra
taught in secondary schools also encompasses additional topics such as func-
tions, sequences, inequalities, and polynomials.
In the first 64 IMOs, there had been a total of 10 other algebra prob-
lems, approximately accounting for 9.9% of all algebra problems. These
problems can be primarily categorized into three types: (1) proving trigono-
metric identities, totaling two problems; (2) finding polynomials, totaling
two problems; (3) proving properties of polynomials and sets, totaling six
problems. The statistical distribution of these three types of problems in
the previous IMOs is presented in Table 5.1.
It can be observed that other algebra problems were not frequent, with
only 1–3 appearing in every 10 IMOs.
Specifically, there were only two problems about proving trigonomet-
ric identities. On the one hand, students begin to engage with trigono-
metric functions from middle school, and have developed a comprehensive
understanding, so IMO problems seldom specifically focused on trigono-
metric concepts. Instead, these concepts were integrated into geometry,
where algebraic methods can also be applied. On the other hand, since
317
318 IMO Problems, Theorems, and Methods: Algebra
Session
Content 1–10 11–20 21–30 31–40 41–50 51–60 61–64 Total
Proving trigonometric 2 0 0 0 0 0 0 2
identities
Finding polynomials 0 1 0 0 1 0 0 2
Proving properties 0 1 1 1 2 0 1 6
Algebra problems 20 20 14 13 15 13 6 101
The percentage of 10.0% 10.0% 7.1% 7.7% 20.0% 0.0% 16.7% 9.9%
other algebra
problems among the
algebra problems
3
2 sin4 x + sin2 2x + 5 cos4 x − cos 3x cos x
4
2 2
1 − cos 2x 3 2 1 + cos 2x 1
=2 + sin 2x + 5 − (cos 4x + cos 2x)
2 4 2 2
7 + 6 cos 2x + 7 cos2 2x 3 1
= + 1 − cos2 2x − (2 cos2 2x − 1 + cos 2x)
4 4 2
= 3 + cos 2x
= 2(1 + cos2 x).
Proof. The purpose is to prove that tan2 C = tan A tan B. Convert the
sine function in the condition into the tangent function:
1 tan2 C
sin2 C 1 − cos2 C 1− 1+tan2 C 1+tan2 C
2 = 2
= 1 = tan2 A
,
sin A 1 − cos A 1− 1+tan2 A 1+tan2 A
tan(A − B) tan A − tan B tan B 1 + tan2 A
1− = 1− = .
tan A tan A(1 + tan A tan B) (1 + tan A tan B) tan A
2
tan C tan A tan B
Combining these two expressions yields 1+tan 2 C = 1+tan A tan B , establish-
Example 5.3. Given tan x1 tan x2 · · · tan x2022 = 1, find the maximum
value of
Thus, we need to find the maximum value of cos x1 cos x2 · · · cos x2022 .
Since
1
= 1 + tan2 xi ≥ 2 |tan xi | ,
cos2 xi
1
we have cos2 x1 cos2 x2 ··· cos2 x2022 ≥ 22022 |tan x1 tan x2 · · · tan x2022 | = 22022 ,
so
1
sin x1 sin x2 · · · sin x2022 = cos x1 cos x2 · · · cos x2022 ≤ ,
21011
π
and the equality holds when x1 = x2 = · · · = x2022 = 4.
x x+2π x+4π 3
Example 5.4. Prove that cos3 3 + cos3 3 + cos3 3 = 4 cos x.
Proof. Using the triple-angle formula cos 3θ = 4 cos3 θ − 3 cos θ for cosine,
we have
x 3 x 1
cos3 = cos + cos x,
3 4 3 4
x + 2π 3 x + 2π 1 3 x + 2π 1
cos3 = cos + cos(x + 2π) = cos + cos x,
3 4 3 4 4 3 4
x + 4π 3 x + 4π 1 3 x − 2π 1
cos3 = cos + cos(x + 4π) = cos + cos x.
3 4 3 4 4 3 4
x x + 2π x + 4π 3
cos3 + cos3 + cos3 = cos x.
3 3 3 4
Other Algebra Problems 323
1 1 1 cos 1◦
+ + · · · + = .
cos 0◦ cos 1◦ cos 1◦ cos 2◦ cos 88◦ cos 89◦ sin2 1◦
sin 1◦ sin((k+1)◦ −k◦ )
Proof. Since cos k◦ cos(k+1)◦ = cos k◦ cos(k+1)◦ = tan(k + 1)◦ − tan k ◦ ,
88 88
1
sin 1◦ = (tan(k + 1)◦ − tan k ◦ ) = tan 89◦ = cot 1◦ .
cos k ◦ cos(k + 1)◦
k=0 k=0
1 1 1 cos 1◦
Hence, cos 0◦ cos 1◦ + cos 1◦ cos 2◦ + ···+ cos 88◦ cos 89◦ = sin2 1◦
.
324 IMO Problems, Theorems, and Methods: Algebra
(i) p an ;
(ii) p|ai for i = 0, 1, . . . , m − 1;
(iii) p2 a0 ,
326 IMO Problems, Theorems, and Methods: Algebra
then f (x) has an irreducible factor with integer coefficients, whose degree is
not less than m.
Cin f (x + n − i).
Proposition 5.22. For all m ∈ N, T2m+1 (0) = 0 and T2m (0) = (−1)m .
Proposition 5.23. The polynomial Tn (x) has n distinct real roots in the
interval [−1, 1], given by
(2k − 1) π
xk = cos (k = 1, 2, . . . , n).
2n
Proposition 5.24. The polynomial Tn (x) has n + 1 points in the inter-
val [−1, 1], xk = cos kπ
n (k = 0, 1, 2, . . . , n), where it alternates between the
maximum value 1 and the minimum value −1.
2 = y +y
2 , for y ∈ C with y = 0.
Other Algebra Problems 331
Proposition 5.26. The generating function for the sequence {Tn (x)} is
1 − xt
Tn (x)tn = , for t ∈ R with |t| ≤ 1.
1 − 2xt + t2
n≥0
f (x) = (x − α)q(x).
Example 5.8. Let f (x) be an integer polynomial. Prove that if the abso-
lute values of f (x) are all 1 when x takes three distinct integers, then f (x)
has no integer roots.
Proof. Suppose f (x) has an integer root x0 . Then f (x) can be factored
as f (x) = (x − x0 )q(x), where q(x) is an integer polynomial. Let a, b, c be
three distinct integers such that |f (a)| = |f (b)| = |f (c)| = 1. Then
|a − x0 | = |b − x0 | = |c − x0 | = 1,
Proof. The equation has a rational root if and only if its discriminant
is a perfect square. If a, b, c are all odd, assume b = 2m + 1, then Δ =
4(m(m + 1) − ac) + 1. Clearly, m(m + 1) − ac is odd, denoted as 2n + 1,
so Δ = 8n + 5, implying Δ is an odd number and leaves a remainder of 5
when divided by 8.
332 IMO Problems, Theorems, and Methods: Algebra
Example 5.11. Find the polynomial f (x) such that f (x2 + 1) = f 2 (x) + 1
with f (0) = 0.
Example 5.12. Prove that for any positive integer n, the polynomial
(x + 1)2n+1 + xn+2 is divisible by the polynomial x2 + x + 1.
Example 5.14. Let a, b, c be real numbers with absolute values less than 1.
Prove that ab + bc + ca + 1 > 0.
2n+1 k
Solution. From Δ2n+1 p (0) = k=0 (−1) Ck2n+1 p (2n + 1 − k) = 0,
n
(−30) + C2j
2n+1 · 2 = 0,
j=1
1
which simplifies to (−30) + 2 2 · 22n+1 − 1 = 0. Thus, n = 2.
where r(x) = 0 or deg r(x) < deg g(x). The polynomial q(x) is called the
quotient of dividing f (x) by g(x), and r(x) is the remainder.
Corollary 5.6. Two polynomials f (x) and g(x) are coprime if and only if
there exist polynomials u(x) and v(x) such that f (x)u(x) + g(x)v(x) = 1.
where a is the leading coefficient of f (x) and p1 (x), p2 (x), . . . , pt (x) are dis-
tinct irreducible polynomials with leading coefficients of 1. Here, pi (x) (i =
1, 2, . . . , t) are the ki -fold factors of f (x).
Theorem 5.16 (Fundamental Theorem of Algebra). Any complex
polynomial of degree n(n > 0) has, counted with multiplicity, exactly n
roots in the complex field.
336 IMO Problems, Theorems, and Methods: Algebra
Corollary 5.7. Let f (x) be a complex polynomial of degree n(n > 0). Then
it can be uniquely factored in the complex field as
f (x) = a(x − α1 )m1 (x − α2 )m2 · · · (x − αt )mt ,
where αi (i = 1, 2, . . . , t) are distinct complex numbers and mi -fold roots of
t
f (x), with i=1 mi = n, and a is the leading coefficient of f (x).
Corollary 5.8. Any real polynomial of odd degree has at least one real
root.
π 2π 3π cos 7π π
14 + cos 14 1
cos − cos + cos = π = .
7 7 7 2 cos 14 2
π
Proof 2. As illustrated in Figure 5.1, construct ∠M ON = 7.
Figure 5.1 ∠M ON .
2π π 3π
1 + 2 cos = 2 cos + 2 cos ,
7 7 7
which implies cos π7 − cos 2π 3π 1
7 + cos 7 = 2 .
338 IMO Problems, Theorems, and Methods: Algebra
Since cos 6π π 4π 3π
7 = − cos 7 and cos 7 = − cos 7 ,
π 2π 3π 1
cos − cos + cos = .
7 7 7 2
In particular, 1 + ε1 + ε2 + · · · + εn−1 = 0.
By (i), we can prove the following proposition.
(ii) Let n be a positive integer. Then
π 3π (2n − 1) π
cosk + cosk + · · · + cosk
2n + 1 2n + 1 2n + 1
1
= for k = 1, 3, . . . , 2n − 1.
2
(n−1)π
(iii) sin πn sin 2π
n · · · sin n
n
= 2n−1 .
(n−1)π 1−(−1)n
(iv) cos πn cos 2πn · · · cos n = 2n .
√
π (n−1)π n
(v) sin 2n sin 2π
2n · · · sin 2n = 2n−1 .
Other Algebra Problems 339
π 2π nπ 1
(vi) cos 2n+1 cos 2n+1 · · · cos 2n+1 = 2n .
√
π 2π nπ 2n+1
(vii) sin 2n+1 sin 2n+1 · · · sin 2n+1 = 2n .
Score Situation This particular problem saw the following distribution of scores among
contestants: 3 contestants scored 6 points, 3 contestants scored 5 points, no contestant
scored 4 points, 1 contestant scored 3 points, no contestant scored 2 points, 2 contestants
scored 1 point, and 7 contestants scored 0 point. The average score of this problem is 2.375,
indicating that it had a certain level of difficulty.
Among the top five teams in the team scores, the Soviet Union team achieved a total
score of 271 points, the Hungary team achieved a total score of 234 points, the Romania
team achieved a total score of 191 points, the Yugoslavia team achieved a total score of 162
points, and the Czechoslovakia team achieved a total score of 151 points.
The gold medal cutoff for this IMO was set at 35 points (with 7 contestants earning gold
medals), the silver medal cutoff was 28 points (with 11 contestants earning silver medals),
and the bronze medal cutoff was 21 points (with 17 contestants earning bronze medals).
In this IMO, no contestant achieved a perfect score of 40 points.
1 1 1
+ + ··· + = cot x − cot 2n x.
sin 2x sin 4x sin 2n x
cos x cos 2x
cot x − cot 2x = −
sin x sin 2x
sin 2x cos x − cos 2x sin x
=
sin x sin 2x
sin x
=
sin x sin 2x
1
= .
sin 2x
Score Situation This particular problem saw the following distribution of scores among con-
testants: 23 contestant scored 5 points, 3 contestants scored 4 points, no contestant scored
3 points, no contestants scored 2 points, no contestants scored 1 point, and 1 contestant
scored 0 point. The average score of this problem is 4.704, indicating that it was simple.
Among the top five teams in the team scores, the Soviet Union team achieved a total
score of 293 points, the Hungary team achieved a total score of 281 points, the German
Democratic Republic team achieved a total score of 280 points, the Poland team achieved a
total score of 269 points, and the Romania team achieved a total score of 257 points.
The gold medal cutoff for this IMO was set at 39 points (with 13 contestants earning gold
medals), the silver medal cutoff was 34 points (with 15 contestants earning silver medals),
and the bronze medal cutoff was 31 points (with 11 contestants earning bronze medals).
In this IMO, a total of 11 contestants achieved a perfect score of 40 points.
Other Algebra Problems 341
P (b + c, a) + P (c + a, b) + P (a + b, c) = 0;
(iii) P (1, 0) = 1.
Solution 1. From conditions (ii) and (iii), P (x, y) is not a constant. Set-
ting a = b and c = −2a in (ii), we obtain
P (x, y) = (x + y) P1 (x, y)
= (x + y)2 P2 (x, y)
··················
n−1
= (x + y) Pn−1 (x, y).
Among the top five teams in the team scores, the scores of this problem are as follows:
the Hungary team scored 33 points (with a total team score of 258 points), the German
Democratic Republic team scored 33 points (with a total team score of 249 points), the
United States team scored 39 points (with a total team score of 247 points), the Soviet Union
team scored 28 points (with a total team score of 246 points), and the United Kingdom team
scored 44 points (with a total team score of 241 points).
The gold medal cutoff for this IMO was set at 38 points (with 8 contestants earning gold
medals), the silver medal cutoff was 32 points (with 25 contestants earning silver medals),
and the bronze medal cutoff was 23 points (with 36 contestants earning bronze medals).
In this IMO, a total of six contestants achieved a perfect score of 40 points.
Problem 5.4 (IMO 45-2, proposed by South Korea). Find all poly-
nomials f with real coefficients such that for all reals a, b, c satisfying
ab + bc + ca = 0, we have the following relation:
f (a − b) + f (b − c) + f (c − a) = 2f (a + b + c).
f (a − b) + f (b − c) + f (c − a) = 2f (a + b + c). (1)
for all a ∈ R.
Therefore, n ≤ 2. Let f (x) = αx4 + βx2 , where α, β ∈ R. It can be
easily verified that f (x) = αx4 + βx2 satisfies the given conditions.
344 IMO Problems, Theorems, and Methods: Algebra
√ √
Note. We can also set a = (1 − 3)x, b = x, and c = (1 + 3)x. It follows
that
√ √ √
f (− 3x) + f (− 3x) + f (2 3x) = 2f (3x).
√
√Forn the coefficient of xn to be nonzero, we must have 2(− 3)n +
(2 3) = 2 · 3n , which only holds for n = 2, 4.
There is a similar problem:
Score Situation This particular problem saw the following distribution of scores among
contestants: 83 contestants scored 7 points, 31 contestants scored 6 points, 23 contestants
scored 5 points, 23 contestants scored 4 points, 32 contestants scored 3 points, 57 contestants
scored 2 points, 158 contestants scored 1 point, and 79 contestants scored 0 point. The
average score for this problem is 2.761, indicating that it had a certain level of difficulty.
Among the top five teams in the team scores, the scores of this problem are as follows:
the China team scored 41 points (with a total team score of 220 points), the United States
team scored 41 points (with a total team score of 212 points), the Russia team scored
34 points (with a total team score of 205 points), the Vietnam team scored 28 points (with
a total team score of 196 points), and the Bulgaria team scored 36 points (with a total team
score of 194 points).
The gold medal cutoff for this IMO was set at 32 points (with 45 contestants earning gold
medals), the silver medal cutoff was 24 points (with 78 contestants earning silver medals),
and the bronze medal cutoff was 16 points (with 120 contestants earning bronze medals).
In this IMO, a total of four contestants achieved a perfect score of 42 points.
Proof 1. Since (P (x))2 − 1 = (P (x) − 1)(P (x) + 1), the integer roots of
(P (x))2 = 1 are all integer roots of either P (x) = 1 or P (x) = −1. We will
prove that one of the equations P (x) = 1 or P (x) = −1 must have at most
two integer roots, and by the Fundamental Theorem of Algebra, the other
equation has at most deg(P ) integer roots, thus proving the conclusion.
We use a proof by contradiction. Assume that both equations
P (x) + 1 = 0 and P (x)−1 = 0 have at least three distinct integer roots, and
these roots are different from each other. Among these six distinct integers,
let the smallest be a.
Without loss of generality, assume a is a root of P (x) + 1 = 0. Then
P (x) + 1 = (x − a)Q(x), where Q(x) is an integer polynomial.
Let b,c, and d be three distinct integer roots of P (x) − 1 = 0, all greater
than a. Since P (x) − 1 = (x − a) Q(x) − 2 and P (b) − 1 = P (c) − 1 =
P (d) − 1 = 0, it follows that
2 = (b − a)Q(b) = (c − a)Q(c) = (d − a)Q(d),
where b − a, c − a, and d − a are three distinct positive integers. Clearly,
at least one of them must be greater than 2 and cannot be a divisor of 2.
This leads to a contradiction.
Proof 2. First, we prove a lemma.
Lemma. If m is an integer root of the integer polynomial F (x). Then the
integer roots of the polynomial F (x) + p or F (x) − p can only be m − p,
m − 1, m + 1, or m + p, where p is a prime number.
Proof of Lemma. Suppose F (x) = (x − m)G(x). Then
F (x) ± p = (x − m) G(x) ± p,
Proof. Let R and S be two polynomials with integer coefficients, and let
m be a positive integer. There are the following two lemmas:
Case 1: i1 < k.
Suppose ir < k ≤ ir+1 . Then Q = R + (1 + x)k S, where
R = Qi1 + Qi2 + · · · + Qir , S = (1 + x)−k (Qir+1 + Qir+2 + · · · + Qin ).
The degrees of R and S are both less than k. By Lemma 1, in (1 + x)k ,
the terms that affect the number of odd coefficients are only 1 and xk .
Thus,
w(Q) = w(R + (1 + x)k S) = w(R + S + xk S)
= w(R + S) + w(S) ≥ w(R).
The last step follows from the triangle inequality. R has w (R) odd
coefficients, and after adding S, there are t of them that become even,
implying that S must have at least t odd coefficients, i.e.,
w(R + S) ≥ w(R) − t and w(S) ≥ t.
Adding these two inequalities yields w(R + S) + w(S) ≥ w(R).
By the induction hypothesis, w(R) ≥ w(Qi1 ), so w(Q) ≥ w(Qi1 ).
Case 2: i1 ≥ k.
Suppose Qi1 = (1 + x)k R and Q = (1 + x)k S, where R = (1 + x)−k Qi1 =
(1 + x)i1 −k and S = (1 + x)−k Q = (1 + x)i1 −k + (1 + x)i2 −k + · · · +
(1 + x)in −k .
Then, the degrees of R and S are both less than k. By Lemma 1 and
Lemma 2,
w(Q) = w((1 + x)k S) = w((1 + xk )S) = 2w(S),
w(Qi1 ) = w((1 + x)k R) = w((1 + xk )R) = 2w(R).
Furthermore, by the induction hypothesis, w(S) ≥ w(R), so w(Q) ≥
w(Qi1 ).
Score Situation This particular problem saw the following distribution of scores among
contestants: 12 contestants scored 7 points, 3 contestants scored 6 points, no contestant
scored 5 points, 1 contestant scored 4 points, 5 contestants scored 3 points, 8 contestants
scored 2 points, 27 contestants scored 1 point, and 153 contestants scored 0 point. The
average score for this problem is 0.785, indicating that it was extremely difficult.
Among the top five teams in the team scores, the scores of this problem are as follows:
the Romania team scored 16 points (with a total team score of 201 points), the United
States team scored 34 points (with a total team score of 180 points), the Hungary team
scored 17 points (with a total team score of 168 points), the Bulgaria team scored 10 points
348 IMO Problems, Theorems, and Methods: Algebra
(with a total team score of 165 points), and the Vietnam team scored 2 points (with a total
team score of 144 points).
The gold medal cutoff for this IMO was set at 34 points (with 14 contestants earning gold
medals), the silver medal cutoff was 22 points (with 35 contestants earning silver medals),
and the bronze medal cutoff was 15 points (with 52 contestants earning bronze medals).
In this IMO, only two contestants achieved a perfect score of 42 points, namely Géza
Kós from Hungary and Daniel Tătaru from Romania.
Proof 1. Suppose f (x) can be factored into the product of two integer
polynomials, i.e., f (x) = g(x)h(x), where
Proof 2. Similar to Proof 1, suppose f (x) can be factored into the product
of two polynomials with integer coefficients, both having leading coefficients
of 1, i.e., f (x) = g(x)h(x).
Since g(0)h(0) = f (0) = 3, either |g(0)| = 1 or |h(0)| = 1. With-
out loss of generality, assume |g(0)| = 1, and let g(x) have k roots
Other Algebra Problems 349
Assume without loss of generality that |ak +5| = min1≤i≤k |ai +5|. Then
|ak +5| ≤ 3. Also, since |−ak |+|ak +5| ≥ 5, we have |ak | ≥ 5−|ak +5| ≥ 2.
From (1), we know that k ≥ 2.
Assume without loss of generality that |a1 | = min1≤i≤k−1 |ai |. Then
1 1
|a1 |k−1 ≤ |a1 ||a2 | · · · |ak−1 | = ≤ .
|ak | 2
1 1 1
Hence, |a1 | ≤ √
k−1
2
, and |a1 + 5| ≤ 5 + √
k−1
2
< 6 and |a1 |n−1 ≤ n−1 <
2 k−1
1
2. Therefore, |a1 + 5| · |a1 |n−1 < 3, which contradicts (2).
In conclusion, f (x) cannot be factored into the product of two noncon-
stant polynomials with integer coefficients.
Note. A famous irreducibility criterion that requires no information on the
canonical decomposition of the coefficients of an integer polynomial is the
following result of Perron:
Let f (x) = xn + an−1 xn−1 + · · · + a1 x + a0 ∈ Z[x] with a0 = 0. If
|an−1 | > 1 + |an−2 | + · · · + |a1 | + |a0 |,
then f is irreducible in Z[x].
Furthermore, there are several similar problems:
Score Situation This particular problem saw the following distribution of scores among
contestants: 92 contestants scored 7 points, 13 contestants scored 6 points, 1 contestant
scored 5 points, 1 contestant scored 4 points, 1 contestant scored 3 points, 6 contestants
scored 2 points, 91 contestants scored 1 point, and 208 contestants scored 0 point. The
average score for this problem is 2,027, indicating that it had a certain level of difficulty.
Among the top five teams in the team scores, the scores of this problem are as follows:
the China team scored 35 points (with a total team score of 215 points), the Germany team
scored 29 points (with a total team score of 189 points), the Bulgaria team scored 24 points
(with a total team score of 178 points), the Russia team scored 28 points (with a total
team score of 177 points), and the Chinese Taiwan team scored 42 points (with a total team
score of 162 points).
The gold medal cutoff for this IMO was set at 30 points (with 35 contestants earning gold
medals), the silver medal cutoff was 20 points (with 66 contestants earning silver medals),
and the bronze medal cutoff was 11 points (with 97 contestants earning bronze medals).
In this IMO, only two contestants achieved a perfect score of 42 points, namely Hong
Zhou from China and Hung-Wu Wu from Chinese Taiwan.
Other Algebra Problems 351
Proof. If every integer fixed point of Q is also a fixed point of P and the
polynomial P (x) − x has at most n roots, then the conclusion holds.
If not, then there exists an integer x0 such that Q(x0 ) = x0 but
P (x0 ) = x0 . Define xi+1 = P (xi ) for i = 0, 1, 2, . . .. Then xk = x0 and
xi+1 = xi for i = 1, 2, . . . , k. Clearly, (u − v)|(P (u) − P (v)) for distinct
integers u and v. Thus, for the following (non-zero) differences, the previ-
ous term divides the subsequent one:
x0 − x1 , x1 − x2 , . . . , xk−1 − xk , xk − xk+1 .
Since the polynomial F (x) has the same degree as P (x), both being
polynomials of degree n, there are at most n distinct integer roots.
Score Situation This particular problem saw the following distribution of scores among
contestants: 48 contestants scored 7 points, 2 contestants scored 6 points, 5 contestants
scored 5 points, 6 contestants scored 4 points, 25 contestants scored 3 points, 8 contestants
scored 2 points, 101 contestants scored 1 point, and 303 contestants scored 0 point. The
average score for this problem is 1.183, indicating that it was relatively challenging.
Among the top five teams in the team scores, the scores of this problem are as follows:
the China team scored 38 points (with a total team score of 214 points), the Russia team
scored 28 points (with a total team score of 174 points), the South Korea team scored 29
points (with a total team score of 170 points), the Germany team scored 32 points (with a
total team score of 157 points), and the United States team scored 20 points (with a total
team score of 154 points).
The gold medal cutoff for this IMO was set at 28 points (with 42 contestants earning gold
medals), the silver medal cutoff was 19 points (with 89 contestants earning silver medals),
and the bronze medal cutoff was 15 points (with 122 contestants earning bronze medals).
In this IMO, only three contestants achieved a perfect score of 42 points, namely Zhiyu
Liu from China, Iurie Boreico from Moldova, and Alexander Magazinov from Russia.
If m < 3n, then Δnx Δny Δnz f (x, y, z) ≡ 0. Using the difference formula
for univariate polynomials, we have:
n
Δn P (x) = (−1)n−i Cin P (x + i),
i=0
points of B.
For a sequence of positive integers s = (s1 , s2 , . . . , sn ), let M (s, l) denote
the minimum of the product of n positive integers yi ≤ si the sum of which
n
is at least l. For l ≤ n, we have M = 1, and for l ≥ i=1 si we define
n
M = i=1 si .
Theorem 1.2 in the 1999 paper “Combinatorial Nullstellensatz” is:
Let F be an arbitrary field, and let f = f (x1 , x2 , . . . , xn ) be a polynomial
in F[x1 , x2 , . . . , xn ]. Suppose the degree deg(f ) of f is ni=1 ti , where each
ti is a non-negative integer, and suppose the coefficient of ni=1 xtii in f is
nonzero. If S1 , S2 , . . . , Sn are subsets of F with |Si | > ti , then there are
s1 ∈ S1 , s2 ∈ S2 , . . . , sn ∈ Sn such that f (s1 , s2 , . . . , sn ) = 0.
Other Algebra Problems 355
Score Situation This particular problem saw the following distribution of scores among
contestants: 5 contestants scored 7 points, no contestant scored 6 points, no contestant
scored 5 points, no contestant scored 4 points, no contestant scored 3 points, 2 contestants
scored 2 points, 40 contestants scored 1 point, and 473 contestants scored 0 point. The
average score for this problem is 0.152, indicating that it was extremely difficult.
Among the top five teams in the team scores, the scores of this problem are as follows:
the Russia team scored 9 points (with a total team score of 184 points), the China team
scored 3 points (with a total team score of 181 points), the South Korea team scored 2
points (with a total team score of 168 points), the Vietnam team scored 0 point (with a total
team score of 168 points), and the United States team scored 4 points (with a total team
score of 155 points).
The gold medal cutoff for this IMO was set at 29 points (with 39 contestants earning gold
medals), the silver medal cutoff was 21 points (with 83 contestants earning silver medals),
and the bronze medal cutoff was 14 points (with 131 contestants earning bronze medals).
In this IMO, no contestant achieved a perfect score of 42 points.
m
Proof. Denote A = {a1 , . . . , an }. By contradiction, assume n = |A| < 2.
Note that the following mm sums are distinct:
f (c1 , . . . , cm ) := c1 m + c2 m2 + · · · + cm mm ,
α1 a1 + α2 a2 · · · + αn an ,
such distinct expressions, which contradicts the earlier statement that all
f (c1 , . . . , cm ) are distinct. Therefore, n ≥ m
2.
Score Situation This particular problem saw the following distribution of scores among
contestants: 37 contestants scored 7 points, no contestant scored 6 points, 2 contestants
scored 5 points, 1 contestant scored 4 points, 3 contestants scored 3 points, 2 contestants
scored 2 points, 12 contestants scored 1 point, and 562 contestants scored 0 point. The
average score for this problem is 0.481, indicating that it was extremely difficult.
Among the top five teams in the team scores, the scores of this problem are as follows:
the China team scored 42 points (with a total team score of 208 points), the Russia team
scored 28 points (with a total team score of 183 points), the South Korea team scored 36
points (with a total team score of 172 points), the United States team scored 29 points (with
a total team score of 165 points), and the Canada team scored 14 points (with a total team
score of 151 points).
The gold medal cutoff for this IMO was set at 24 points (with 52 contestants earning gold
medals), the silver medal cutoff was 19 points (with 103 contestants earning silver medals),
and the bronze medal cutoff was 12 points (with 148 contestants earning bronze medals).
In this IMO, only one contestant achieved a perfect score of 42 points, namely Yichuan
Wang from China.
Other Algebra Problems 357
5.3 Summary
Trigonometry originated from the needs of navigation, astronomy, and sim-
ilar fields, and later evolved into an independent branch of mathematics.
Polynomial, on the other hand, is a classical subject in algebra. Therefore,
this chapter also introduces some knowledge of trigonometric identities and
polynomials.
In the first 64 IMOs, there were a total of 10 other algebra problems.
These problems can be broadly categorized into three types, as depicted in
Figure 5.2. The score details for these problems are presented in Table 5.2.
Due to the smaller number of participating teams and missing contestant
score information in early IMOs, there are several blanks in Table 5.2.
2.5
Proving Trigonometric Identities Finding Polynomials
Proving Properties
2
1.5
0.5
0
1–10 11–20 21–30 31–40 41–50 51–60 61–64
Full points 6.000 5.000 8.000 7.000 8.000 7.000 7.000 7.000
Note. Top five mean = Total score of the top five teams ÷ Total number of contestants from the top five teams,
6th–15th mean = Total score of the 6th–15th teams ÷ Total number of contestants from the 6th–15th teams,
16th–25th mean = Total score of the 16th–25th teams ÷ Total number of contestants from the 16th–25th teams.
Other Algebra Problems 359
From Table 5.2, it can be observed that in the first 64 IMOs, there
were three other algebra problems with an average score of 0–1 point; one
problem with an average score of 1–2 points; four problems with an average
score of 2–3 points; one problem with an average score of 3–4 points; one
problem with an average score above 4 points. Overall, the other algebra
problems were relatively difficult.
In the 24th–64th IMOs, there were a total of six other algebra problems.
Among these, three had an average score of 0–1 point; one had an average
score of 1–2 points; two had an average score of 2–3 points; no problem
had an average score of 3–4 points; no problem had an average score above
4 points. Further analysis of the problem numbers of these six other algebra
problems, as shown in Table 5.3, reveals that these problems frequently
appeared as the 3rd/6th problem. The majority of these problems, totaling
five, were of the type proving properties of polynomials and sets.
Proving trigonometric 0 0 0 2
identities
Finding polynomials 0 1 0 2
Proving properties 1 1 3 6
Total 1 2 3 10
(Continued)
361
362 IMO Problems, Theorems, and Methods: Algebra
(Continued)
(Continued)
(Continued)
364 IMO Problems, Theorems, and Methods: Algebra
(Continued)
Problem Problem
Number in Proposing Number in Page
the IMO Country Category the Book Number
365
366 IMO Problems, Theorems, and Methods: Algebra
(Continued)
Problem Problem
Number in Proposing Number in Page
the IMO Country Category the Book Number
(Continued)
Problem Problem
Number in Proposing Number in Page
the IMO Country Category the Book Number
(Continued)
Problem Problem
Number in Proposing Number in Page
the IMO Country Category the Book Number
(Continued)
Problem Problem
Number in Proposing Number in Page
the IMO Country Category the Book Number
(Continued)
Problem Problem
Number in Proposing Number in Page
the IMO Country Category the Book Number
(Continued)
Problem Problem
Number in Proposing Number in Page
the IMO Country Category the Book Number
(Continued)
Problem Problem
Number in Proposing Number in Page
the IMO Country Category the Book Number